50 soal ukg & pembahasan sesuai kisi-kisis.pdf

15
Beranda Soal UKG Agus Zainal M (SDN Gudang 2) [email protected] BERANDA SOAL UKG 2015 AGUS ZAINAL M, S.Pd (SDN GUDANG 2) 1. KD 1.1 Analisis Perkembangan Bahasa Anak Usia SD Berikut ini adalah Karakteristik Anak Usia SD dari segi Mental, yaitu... a. Anak sudah memiliki gerakan yang bebas dan aman. Hal ini berguna untuk melakukan berbagai gerakan motorik kasar (jasmani) seperti memanjat, berlari dan menaiki tangga. b. Anak dapat menunjukkan kreativitasnya dalam membentuk sesatu karya tertentu c. Anak mulai tidak suka terikat dengan orang dewasa d. Anak menunjukkan tenggang rasa dan penghargaan terhadap teman Uraian: Segi Mental - Anak sudah mulai memahami beberapa konsep abstrak seperti menghitung tanpa menggunakan benda - Anak sudah dapat menghubungkan suatu objek atau kejadian dengan konsep tertentu yang bersifat abstrak. Misalnya tentang luas dan volume - Anak dapat menunjukkan kreativitasnya dalam membentuk sesatu karya tertentu - Anak dapat menciptakan sesiatu bentuk/benda dengan menggunakan alat - Anak dapat membuat gambar-gambar dengan menggunakan sudut perspektif sederhana - Anak dapat menampilkan sifat ingin tahu - Anak dapat merumuskan dan menunjukkan pengertian terhadap sesuatu - Anak sudah dapat mengikuti peraturan yang berlaku umum - Anak dapat menyelesaikan tugas yang diberikan guru, baik sendiri maupun kerja sama - Anak dapat menunjukkan aktivitasnya dalam berbagai kegiatan sekolah maupun di lingkungannya - Anak dapat memperlihatkan insiatif dan alternative untuk memecahkan masalah-masalah tertentu 2. KD 1.1 Analisis Perkembangan Bahasa Anak Usia SD Berikut ini adalah Karakteristik Anak Usia SD dari segi Psikomotorik, yaitu... a. Anak sudah dapat memakai pakaian dengan rapi b. Anak dapat menciptakan sesiatu bentuk/benda dengan menggunakan alat c. Anak sudah mulai memahami beberapa konsep abstrak seperti menghitung tanpa menggunakan benda d. Anak menunjukkan kepedulian terhadap orang lain Uraian: Segi Psikomotorik - Anak sudah memiliki gerakan yang bebas dan aman. Hal ini berguna untuk melakukan berbagai gerakan motorik kasar (jasmani) seperti memanjat, berlari dan menaiki tangga. - Memiliki kemampuan dalam melakukan koordinasi dan keseimbangan badan. Misalnya ketika berjalan atau berlari dengan berbagai pola - Anak sudah dapat memperkirakan kegiatan/gerakan yang berbahaya dan tidak berbahaya - Anak sudah dapat memakai pakaian dengan rapi - Anak sudah bias menunjukkan kebersihan dalam berpakaian, badan dan alat-alat yang dibawa 3 KD 1.1 Analisis Perkembangan Bahasa Anak Usia SD Berikut ini adalah Karakteristik Anak Usia SD dari segi Emosionalitas, yaitu... a. Anak menunjukkan keceriaan dalam berbagai aktivitas bersama kelompok teman sebayanya b. Anak dapat memperlihatkan insiatif dan alternative untuk memecahkan masalah-masalah tertentu c. Anak sudah mulai memahami beberapa konsep abstrak seperti menghitung tanpa menggunakan benda d. Anak dapat menampilkan sifat ingin tahu Uraian: Segi Sosial Emosionalitas - Anak mulai tidak suka terikat dengan orang dewasa - Anak dapat menunjukkan penghargaan terhadap guru atau orang dewasa lainnya - Anak dapat menunjukkan sikap empati terhadap suatu kondisi - Anak menunjukkan keceriaan dalam berbagai aktivitas bersama kelompok teman sebayanya - Anak dapat menunjukkan sikap marah dalam kondisi yang wajar - Anak menunjukkan kepedulian terhadap orang lain - Anak menunjukkan tenggang rasa dan penghargaan terhadap teman - Anak menunjukkan rasa solidaritas terhadap teman sekelompoknya - Anak telah memiliki kemauan untuk menceritakan sesuatu kepada teman-temannya 4 KD 1.1 Analisis Perkembangan Bahasa Anak Usia SD Memiliki kemampuan dalam melakukan koordinasi dan keseimbangan badan. Misalnya ketika berjalan atau berlari dengan berbagai pola adalah karakteristik perkembangan bahasa anak usia SD ditinjau dari segi ... a. Mental b. Psikomotorik c. Sosial d. Emosional

Upload: pak-gambeng

Post on 04-Jan-2016

646 views

Category:

Documents


7 download

TRANSCRIPT

Page 1: 50 SOAL UKG & PEMBAHASAN SESUAI KISI-KISIS.pdf

Beranda Soal UKG Agus Zainal M (SDN Gudang 2)

Republik_ilalangyahoocoid

BERANDA SOAL UKG 2015 AGUS ZAINAL M SPd (SDN GUDANG 2)

1 KD 11 Analisis Perkembangan Bahasa Anak Usia SD

Berikut ini adalah Karakteristik Anak Usia SD dari segi Mental yaitu a Anak sudah memiliki gerakan yang bebas dan aman Hal ini berguna untuk melakukan berbagai gerakan motorik

kasar (jasmani) seperti memanjat berlari dan menaiki tangga

b Anak dapat menunjukkan kreativitasnya dalam membentuk sesatu karya tertentu

c Anak mulai tidak suka terikat dengan orang dewasa d Anak menunjukkan tenggang rasa dan penghargaan terhadap teman

Uraian Segi Mental

- Anak sudah mulai memahami beberapa konsep abstrak seperti menghitung tanpa menggunakan benda - Anak sudah dapat menghubungkan suatu objek atau kejadian dengan konsep tertentu yang bersifat abstrak Misalnya

tentang luas dan volume - Anak dapat menunjukkan kreativitasnya dalam membentuk sesatu karya tertentu - Anak dapat menciptakan sesiatu bentukbenda dengan menggunakan alat - Anak dapat membuat gambar-gambar dengan menggunakan sudut perspektif sederhana - Anak dapat menampilkan sifat ingin tahu - Anak dapat merumuskan dan menunjukkan pengertian terhadap sesuatu - Anak sudah dapat mengikuti peraturan yang berlaku umum - Anak dapat menyelesaikan tugas yang diberikan guru baik sendiri maupun kerja sama - Anak dapat menunjukkan aktivitasnya dalam berbagai kegiatan sekolah maupun di lingkungannya - Anak dapat memperlihatkan insiatif dan alternative untuk memecahkan masalah-masalah tertentu

2 KD 11 Analisis Perkembangan Bahasa Anak Usia SD Berikut ini adalah Karakteristik Anak Usia SD dari segi Psikomotorik yaitu a Anak sudah dapat memakai pakaian dengan rapi

b Anak dapat menciptakan sesiatu bentukbenda dengan menggunakan alat c Anak sudah mulai memahami beberapa konsep abstrak seperti menghitung tanpa menggunakan benda d Anak menunjukkan kepedulian terhadap orang lain

Uraian Segi Psikomotorik

- Anak sudah memiliki gerakan yang bebas dan aman Hal ini berguna untuk melakukan berbagai gerakan motorik kasar (jasmani) seperti memanjat berlari dan menaiki tangga

- Memiliki kemampuan dalam melakukan koordinasi dan keseimbangan badan Misalnya ketika berjalan atau berlari dengan berbagai pola

- Anak sudah dapat memperkirakan kegiatangerakan yang berbahaya dan tidak berbahaya - Anak sudah dapat memakai pakaian dengan rapi - Anak sudah bias menunjukkan kebersihan dalam berpakaian badan dan alat-alat yang dibawa

3 KD 11 Analisis Perkembangan Bahasa Anak Usia SD Berikut ini adalah Karakteristik Anak Usia SD dari segi Emosionalitas yaitu a Anak menunjukkan keceriaan dalam berbagai aktivitas bersama kelompok teman sebayanya

b Anak dapat memperlihatkan insiatif dan alternative untuk memecahkan masalah-masalah tertentu c Anak sudah mulai memahami beberapa konsep abstrak seperti menghitung tanpa menggunakan benda d Anak dapat menampilkan sifat ingin tahu

Uraian Segi Sosial ndash Emosionalitas

- Anak mulai tidak suka terikat dengan orang dewasa - Anak dapat menunjukkan penghargaan terhadap guru atau orang dewasa lainnya - Anak dapat menunjukkan sikap empati terhadap suatu kondisi - Anak menunjukkan keceriaan dalam berbagai aktivitas bersama kelompok teman sebayanya - Anak dapat menunjukkan sikap marah dalam kondisi yang wajar - Anak menunjukkan kepedulian terhadap orang lain - Anak menunjukkan tenggang rasa dan penghargaan terhadap teman - Anak menunjukkan rasa solidaritas terhadap teman sekelompoknya - Anak telah memiliki kemauan untuk menceritakan sesuatu kepada teman-temannya

4 KD 11 Analisis Perkembangan Bahasa Anak Usia SD Memiliki kemampuan dalam melakukan koordinasi dan keseimbangan badan Misalnya ketika berjalan atau berlari dengan berbagai pola adalah karakteristik perkembangan bahasa anak usia SD ditinjau dari segi a Mental b Psikomotorik

c Sosial d Emosional

Beranda Soal UKG Agus Zainal M (SDN Gudang 2)

Republik_ilalangyahoocoid

Uraian No 2 5 KD 11 Analisis Perkembangan Bahasa Anak Usia SD

Anak dapat menampilkan sifat ingin tahu adalah karakteristik perkembangan bahasa anak usia SD ditinjau dari segi a Mental

b Psikomotorik c Sosial d Emosional

Uraian No 1 6 KD 11 Analisis Perkembangan Bahasa Anak Usia SD

Anak dapat menunjukkan sikap marah dalam kondisi yang wajar adalah karakteristik perkembangan bahasa anak usia SD ditinjau dari segi a Mental b Psikomotorik c Kognitif d Emosional

Uraian No 3 7 KD 11 Analisis Perkembangan Bahasa Anak Usia SD

Faktor kendala yang mempengaruhi keterampilan berbahasa anak adalah sebagai berikut kecuali a Jenis Kelamin b Keluarga c Keinginan dan Dorongan Komunikasi d Kebiasaan

Uraian Faktor dan Kendala dalam Mempelajari Keterampilan Berbahasa

1 Kesehatan 2 Kecerdasan 3 Jenis kelamin 4 Keluarga 5 Keinginan dan dorongan untuk berkomunikasi serta hubungan dengan teman sebaya 6 Kepribadian

8 KD 112 Aspek Membaca di kelas rendah Tahapan proses belajar membaca bagi siswa sekolah dasar kelas awal biasanya menggunakan metode membaca a Ejaan per ejaan

b Skimming c Permulaan d Cepat

Uraian Metode membaca permulaan

1) Metode Ejaalfabet 2) Metode Bunyi 3) Metode Suku Kata 4) Metode Kata 5) Metode Global 6) Metode SAS

9 KD 112 Aspek Membaca di kelas rendah Dalam pembelajaran membaca permulaan ada beberapa metode yang dapat digunakan kecuali a metode kupas rangkai suku kata b metode kata lembaga

c metode SAS d metode eja

Uraian No 8 10 KD 112 Aspek Membaca di kelas rendah

Berikut diberikan studi kasus Mula-mula diberikan kalimat secara keseluruhan Kalimat itu diuraikan atas kata-kata yang mendukungnya Dari kata-kata itu kita ceraikan atas suku-suku katanya dan akhirnya atas huruf-hurufnya Kemudian huruf-huruf itu kita sintetiskan kembali menjadi suku kata suku kata menjadi kata dan kata menjadi kalimat Berdasarkan studi kasus metode membaca permulaan yang tepat digunakan adalah a Metode Alfabet b Metode Suku Kata c Metode SAS

Beranda Soal UKG Agus Zainal M (SDN Gudang 2)

Republik_ilalangyahoocoid

d Metode Cerita Uraian 11 KD 112 Aspek Membaca di kelas rendah

Kelebihan membaca permulaan adalah sebagai berikut a Mempunyai nilai strategis bagi pengembangan kepribadian dan kemampuan siswa

b Meningkatkan nilai siswa dalam pelajaran Bahasa Indonesia c Meningkatkan kemandirian siswa dalam membaca d Mempermudah menghafal kata-kata

Uraian Kelebihan membaca permulaan

- Dapat memperoleh informasi secara tepat dan lengkap - Mempunyai nilai strategis bagi pengembangan kepribadian dan kemampuan siswa - Membaca permulaan juga dapat mengembangkan nilai-nilai moral kemampuan bernalar dan kreatifitas seseorang Kekurangan membaca permulaan - Kemampuan membaca pada umumnya lebih rendah - Menghambat kemampuan untuk memahami kalimat atau cerita - Mengalami kesulitan menjawab mengenai isi cerita karena kesibukan siswa mengeja dan menyuarakan huruf-huruf - Kecepata membaca dan pemahaman siswa sangat rendah

12 KD 112 Aspek Membaca di kelas rendah Metode yang merupakan penyempurnaan metode alphabet dengan mengajarkan bunyi-bunyi bahasa sebagai pengganti huruf-huruf berdasarkan ucapan hurufnya adalah metode membaca permulaan a Metode Suku Kata b Metode Kata-kata c Metode Cerita d Metode Suara

Uraian Teknik Membaca Permulaan

1) Metode Alfabet Metode ini sering disebut metode harfiah Metode ldquoletter by letter methodrdquo atau ldquoABC methodrdquo Mula-mula dikenalkan abjad dari A sampai Z Setelah hafal beberapa huruf barulah huruf-huruf tersebut dirangkai menjadi suku kata Suku kata-suku kata tersebut setelah dikenalkan barulah dirangkai menjadi kata Dengan modal beberapa kata ituterciptalah kalimat 2) Metode Suara Metode suara juga disebut rdquoPhonic Methodrdquo Metode ini merupakan penyempurnaan metode alphabet Pada metode ini bukan abjadnya yang di ajarkan melainkan bunyi-bunyi bahasa sebagai pengganti huruf-huruf tersebutUcapan huruf-huruf tidak berdasarkan atas bunyi abjadnya melainkan ucapan hurufnya 3) Metode Suku Kata Metode ini biasa juga disebut rdquoSyllabic Methodrdquo Dalam metode ini suku kata merupakan kunci pokok dalam membuat kata Metode suku kata sebenarnya suatu metode pengajaran membaca permulaan yang di dasarkan atas kata-kata dianalisis menjadi suku kata-suku kataSuku kata0suku kata tersebut di intesiskan kembali menjadi kata-kata 4) Metode Kata-kata Pelaksanaan metode ini selalu di awali dengan kata-kata tertulis da ndash da gi ndash gi ku ndash ku dan lain-lain Setelah kata-kata itu dikenalkan kata-kata tersebut dianalisis atas suku katanya Selanjitnya suku kata itu di analisis lagi atas huruf-hurufnya Barulah huruf-huruf itu digabungkan kembali dalam bentuk kata semula 5) Metode Cerita Pelaksanaan metode cerita dalam mengajarkan membaca permulaan diawali dengan menghafalkan cerita atau sebuah puisi Cerita atau puisi itu diuraikan atas kalimat-kalimtnya sampai pada kata-katanya Dalam mengucapkan kata0kata metode ini menggunakan kata-kata fonetik 6) Metode Struktural Analitik Sintetik (SAS) Adapun contoh pelaksanaan metode SAS sebagai berikut Mula-mula diberikan kalimat secara keseluruhan Kalimat itu diuraikan atas kata-kata yang mendukungnya Dari kata-kata itu kita ceraikan atas suku-suku katanya dan akhirnya atas huruf-hurufnya Kemidian huruf-huruf itu kita sintetiskan kembali menjadi suku kata suku kata menjadi kata dan kata menjadi kalimat

13 KD 113 Aspek Menulis di kelas tinggi Pendekatan menekankan keterpaduan empat aspek keterampilan berbahasa (menyimak berbicara membaca dan menulis) dalam pembelajaran adalah salah satu Pendekatan yang disarankan dalam pembelajaran menulis yang disebut a pendekatan komunikatif b Pendekatan integratif

c Pendekatan keterampilan proses d Pendekatan tematis

Uraian Pendekatan yang disarankan dalam pembelajaran menulis

1) Pendekatan komunikatif memfokuskan pada keterampilan siswa mengimplementasikan fungsi bahasa (untuk berkomunikasi) dalam pembelajaran

Beranda Soal UKG Agus Zainal M (SDN Gudang 2)

Republik_ilalangyahoocoid

2) Pendekatan integratif menekankan keterpaduan empat aspek keterampilan berbahasa (menyimak berbicara membaca dan menulis) dalam pembelajaran

3) Pendekatan keterampilan proses memfokuskan keterampilan siswa dalam mengamati mengklasifikasi menginterpretasi dan mengkomunikasikan

4) Pendekatan tematis menekankan tema pembelajaran sebagai payungpemandu dalam pembelajaran 14 KD 113 Aspek Menulis di kelas tinggi

Teknik menulis cerita terdiri atas hal-hal sebagai berikut kecuali a menjawab pertanyaan b membuat kalimat c subtitusi d persuasi

Uraian Teknik menyusun cerita dapat dilakukan dengan menjawab pertanyaan melengkapi kalimat memperbaiki susunan kalimat

memperluas kalimat subtitusi transfomtasi dan membuat kalimat Persuasi adalah bujukan halus

15 KD 113 Aspek Menulis di kelas tinggi Model pembelajaran menulis ceritacerpen di SD meliputi hal-hal berikut kecuali a Menceritakan gambar b Melanjutkan cerita c Menceritakan pengalaman d Mendeskripsikan cerita

Uraian Model pembelajaran menulis ceritacerpen di SD meliputi menceritakan gambar melanjutkan ceria lain menceitakan

mimpi menceritakan pengalaman dan menceritakan cita-cita 16 KD 121 Memilih berbagai Metode Menulis Permulaan

1) Menulis huruf lepas 2) Merangkaikan huruf lepas menjadi suku kata 3) Merangkaikan suku kata menjadi kata 4) Menyusun kata menjadi kalimat (Djauzak 19964) Tahapan diatas adalah Teknik Menulis Permulaan dengan metode a Metode Eja

b Metode Kata Lembaga c Metode SAS d Metode Global

Uraian Metode dan pembelajaran menulis permulaan

a Metode Eja Metode eja di dasarkan pada pendekatan harfiah artinya belajar membaca dan menulis dimulai dari huruf-huruf yang dirangkaikan menjadi suku kata Oleh karena itu pengajaran dimulai dari pengenalan huruf-huruf Demikian halnya dengan pengajaran menulis di mulai dari huruf lepas dengan langka-langkah sebagai berikut 1) Menulis huruf lepas 2) Merangkaikan huruf lepas menjadi suku kata 3) Merangkaikan suku kata menjadi kata 4) Menyusun kata menjadi kalimat (Djauzak 19964) b Metode kata lembaga Metode kata lembaga di mulai mengajar dengan langkah-langkah sebagai berikut 1) Mengenalkan kata 2) Merangkaikan kata antar suku kata 3) Menguraikan suku kata atas huruf-hurufnya 4) Menggabungkan huruf menjadi kata (Djauzak 19965) c Metode Global Metode global memulai pengajaran membaca dan menulis permulaan dengan membaca kalimat secara utuh yang ada di bawah gambar Menguraikan kalimat dengan kata-kata menguraikan kata-kata menjadi suku kata (Djauzak 19966) d Metode SAS Menuryut (Supriyadi 1996 334-335) pengertian metode SAS adalah suatu pendekatan cerita di sertai dengan gambar yang didalamnya terkandung unsur analitik sintetik Metode SAS menurut (Djuzak19968) adalah suatu pembelajaran menulis permulaan yang didasarkan atas pendekatan cerita yakni cara memulai mengajar menulis dengan menampil cerita yang diambil dari dialog siswa dan guru atau siswa dengan siswa Teknik pelaksanaan pembelajaran metode SAS yakni keterampilan menulis kartu huruf kartu suku kata kartu kata dan kartu kalimat sementara sebagian siswa mencari huruf suku kata dan kata guru dan sebagian siswa menempel kata-kata yang tersusun sehingga menjadi kalimat yang berarti (Subana) Proses operasional metode SAS mempunyai langkah-lagkah dengan urutan sebagai berikut a Struktur yaitu menampilkan keseluruhan b Analitik yatu melakukan proses penguraian c Sintetik yaitu melakukan penggalan pada struktur semula

Beranda Soal UKG Agus Zainal M (SDN Gudang 2)

Republik_ilalangyahoocoid

17 KD 121 Memilih berbagai Metode Menulis Permulaan 1) Mengenalkan kata 2) Merangkaikan kata antar suku kata 3) Menguraikan suku kata atas huruf-hurufnya 4) Menggabungkan huruf menjadi kata (Djauzak 19965) Tahapan diatas adalah Teknik Menulis Permulaan dengan metode a Metode Eja b Metode Kata Lembaga

c Metode SAS d Metode Global

Uraian No 16 18 KD 121 Memilih berbagai Metode Menulis Permulaan

Metode dengan memulai pengajaran membaca dan menulis permulaan dengan membaca kalimat secara utuh yang ada di bawah gambar Menguraikan kalimat dengan kata-kata menguraikan kata-kata menjadi suku kata disebut dengan metode a Metode Eja b Metode Kata Lembaga c Metode SAS d Metode Global

Uraian No 16 19 KD 121 Memilih berbagai Metode Menulis Permulaan

Suatu pembelajaran menulis permulaan yang didasarkan atas pendekatan cerita yakni cara memulai mengajar menulis dengan menampil cerita yang diambil dari dialog siswa dan guru atau siswa dengan siswa disebut dengan metode a Metode Eja b Metode Kata Lembaga c Metode SAS

d Metode Global Uraian No 16 20 KD 121 Memilih berbagai Metode Menulis Permulaan

menulis kartu huruf kartu suku kata kartu kata dan kartu kalimat sementara sebagian siswa mencari huruf suku kata dan kata guru dan sebagian siswa menempel kata-kata yang tersusun sehingga menjadi kalimat yang berarti adalah contoh metode a Metode Eja b Metode Kata Lembaga c Metode SAS

d Metode Global Uraian No 16 21 KD 122 Merancang berbagai kegiatan menulis di kelas tinggi

Berikut adalah kegiatan menulis lanjutan di kelas tinggi kecuali a menulis tentang berbagai topik b menulis pengumuman c menulis pantun d menulis memo

Uraian 22 KD 123 Perencanaan dan Pelaksanaan evaluasi pembelajaran Bahasa dan Sastra Indonesia

Perencanaan Pengajaran meliputi hal-hal berikut kecuali a tujuan apa yang hendak dicapai b memilih bahan ajar c proses belajar mengajar d alat penilaian

Uraian Menurut Akhlan dan Rahman (199715) perencanaan pengajaran meliputi

a tujuan apa yang hendak dicapai b bahan pengajaran c proses belajar mengajardan d alat penilaian

23 KD 123 Perencanaan dan Pelaksanaan evaluasi pembelajaran Bahasa dan Sastra Indonesia karakteristik perencanaan pengajaran yang baik hendaknya mengandung prinsip sebagai berikut a Memiliki sikap objektif rasio (tepat dan masuk akal) komprehensif dan sistematis (menyeluruh dan tersusun rapi)

b Merupakan suatu wahana atau wadah untuk mengembangkan segala potensi yang ada dan dimiliki oleh anak

Beranda Soal UKG Agus Zainal M (SDN Gudang 2)

Republik_ilalangyahoocoid

didik c Mengendalikan kekuatan sendiri bukan didasarkan atas kekuatan orang lain d Melakukan studi kasus yang berkesinambungan

Uraian Menurut Akhlan dan Rahman (19977) karakteristik perencanaan pengajaran yang baik hendaknya mengandung prinsip

sebagai berikut a Mengembangkan hubungan interaksi yang baik di antara sesama manusia dalam hal ini siswa dan guru serta personal

terkait b Merupakan suatu wahana atau wadah untuk mengembangkan segala potensi yang ada dan dimiliki oleh anak didik c Memiliki sikap objektif rasio (tepat dan masuk akal) komprehensif dan sistematis (menyeluruh dan tersusun rapi) d Mengendalikan kekuatan sendiri bukan didasarkan atas kekuatan orang lain Didukung oleh fakta dan data yang

menunjang pencapaian tujuan yang telah di dirumuskan e Fleksibel dan dinamis artinya mudah disesuaikan dengan keadaan serta perkembangan ke arah yang lebih baik dan

maju 24 KD 123 Perencanaan dan Pelaksanaan evaluasi pembelajaran Bahasa dan Sastra Indonesia

tahap mengumpulkan informasi tentang keadaan objek evaluasi (siswa) dengan menggunakan teknik tes atau nontes disebut tahapan a Tahap Tindak Lanjut b Tahap Persiapan c Tahap Pelaksanaan

d Tahap Pengolahan Hasil Uraian Berikut ini penjelasan singkat tentang keempat tahap evaluasi pembelajaran tersebut

(1) Tahap Persiapan Menurut Damaianti (2007 8) tahap ini disebut juga tahap perencanaan dan perumusan kriterium Langkahnya meliputi (a) perumusan tujuan evaluasi (b) penetapan aspek-aspek yang akan dievaluasi (c) menetapkan metode dan bentuk evaluasi (tesnontes) (d) merencanakan waktu evaluasi (e) melakukan uji coba (untuk tes) agar dapat mengukur validitas dan reliabilitasnya Untuk evaluasi yang menggunakan tes hasil dari tahap ini adalah kisi-kisi soal dan seperangkat alat tes soal lembar jawaban (untuk tes tulis) kunci jawaban dan pedoman penilaian 2) Tahap Pelaksanaan Tahap pelaksanaan atau disebut juga dengan tahap pengukuran dan pengumpulan data adalah tahap untuk mengumpulkan informasi tentang keadaan objek evaluasi (siswa) dengan menggunakan teknik tes atau nontes Bila menggunakan teknik tes soal yang igunakan sebaiknya sudah teruji validitas dan reliabilitasnya Tes yang digunakan dapat berbentuk tes tulis lisan atau praktik 3) Tahap Pengolahan Hasil Tahap pengolahan hasil adalah tahap pemeriksaan hasil evaluasi dengan memberikan skor Skor yang diperoleh siswa selanjutnya diubah menjadi nilai Pada tes tulis pemeriksaan hasil dilakukan setelah tes selesai sedangkan pada tes lisan dan praktik pemberian nilai dilakukan bersamaan dengan waktu pelasanaan tes tersebut 4) Tahap Tindak Lanjut Tahap tindak lanjut atau disebut juga tahap penafsiran adalah tahap untuk mengambil keputusan berdasarkan nilai yang dihasilkan pada tahap pengolahan hasil misalnya a memperbaiki proses belajar mengajar b memperbaiki kesulitan belajar siswa c memperbaiki alat evaluasi d membuat laporan evaluasi (rapor)

25 KD 131 Merumuskan hakikat (pengertiantujuanjenismanfaat) membaca membaca yang mengutamakan isi bacaan sebagai ungkapan pikiran perasaan dan kehendak penulis Bila hanya ingin mengetahui isinya membaca cerdas bersifat lugas Akan tetapi bila maksudnya untuk memahami dan memilki isi bacaan maka tergolong kedalam membaca jenis a Membaca cerdas atau membaca dalam hati

b Membaca bahasa c Membaca teknis d Membaca bebas

Uraian Menurut Soedjono dalam Sue (200418-21) ada lima macam membaca yaitu membaca bahasa membaca cerdas atau

membaca dalam hati membaca teknis membaca emosional dan membaca bebas 1) Membaca bahasa Membaca bahasa adalah membaca yang mengutamakan bahasa bacaan Membaca bahasa mementingkan segi bahasa bacaan Hal-hal yang perlu diperhatikan dalam membaca bahasa adalah kesesuian pikir dengan bahasa perbendaharaan bahasa yang meliputi kosa kata struktur kalimat dan ejaan 2) Membaca cerdas atau membaca dalam hati

Beranda Soal UKG Agus Zainal M (SDN Gudang 2)

Republik_ilalangyahoocoid

Membaca cerdas adalah membaca yang mengutamakan isi bacaan sebagai ungkapan pikiran perasaan dan kehendak penulis Bila hanya ingin mengetahui isinya membaca cerdas bersifat lugas Akan tetapi bial maksudnya untuk memahami dan memilki isi bacaan maka disebut membaca belajar 3) Membaca teknis Membaca teknis adalah membaca dengan mengarahkan bacaan secara wajar Wajar maksudnya sesuai ucapan tekanan dan intonasinya Pikiran perasaan dan kemauan yang tersimpan dalam bacaan dapat diaktualisasikan dengan baik 4) Membaca emosional Membaca emosional adalah membaca sebagai sarana untuk memasuki perasaan yaitu keindahan isi dan keindahan bahasanya 5) Membaca bebas Membaca bebas adalah membaca sesuatu atas kehendak sendiri tanpa adanya unsur paksaan dari luar Unsur dari luar misalnya guru orang tua teman atau pihak-pihak lain

26 KD 131 Merumuskan hakikat (pengertiantujuanjenismanfaat) membaca membaca sesuatu atas kehendak sendiri tanpa adanya unsur paksaan dari luar Unsur dari luar misalnya guru orang tua teman atau pihak-pihak lain termasuk jenis membaca a Membaca cerdas atau membaca dalam hati b Membaca bahasa c Membaca teknis d Membaca bebas

Uraian No 25 27 KD 131 Merumuskan hakikat (pengertiantujuanjenismanfaat) membaca

1 menggunakan ucapan yang tepat 2 menggunakan frase yang tepat 3 menggunakan intonasi suara yang wajar 4 dalam posisi sikap yang baik 5 menguasai tanda-tanda baca 6 membaca dengan terang dan jelas keterampilan di atas harus dipunyai dalam jenis membaca a Membaca cerdas atau membaca dalam hati b Membaca bahasa c Membaca teknis d Membaca nyaring

Uraian JENIS-JENIS MEMBACA DAN KARAKTERISTIKNYA

Ditinjau dari segi terdengar atau tidaknya suara pembaca waktu melakukan kegiatan membaca maka proses membaca dapat dibedakan menjadi A Membaca Nyaring Membaca nyaring adalah kegiatan membaca dengan menyuarakan tulisan yang dibacanya dengan ucapan dan intonasi yang tepat agar pendengar dan pembaca dapat menangkap informasi yang disampaikan oleh penulis baik yang berupa pikiran perasaan sikap ataupun pengalaman penulis Ketrampilan yang dituntut dalam membaca nyaring adalah berbagai kemampuan diantaranya adalah 1 menggunakan ucapan yang tepat 2 menggunakan frase yang tepat 3 menggunakan intonasi suara yang wajar 4 dalam posisi sikap yang baik 5 menguasai tanda-tanda baca 6 membaca dengan terang dan jelas 7 membaca dengan penuh perasaan ekspresif 8 membaca dengan tidak terbata-bata 9 mengerti serta memahami bahan bacaan yang dibacanya 10 kecepatan bergantung pada bahan bacaan yang dibacanya 11 membaca dengan tanpa terus-menerus melihat bahan bacaan 12 membaca dengan penuh kepercayaan pada diri sendiri B Membaca Dalam Hati Membaca dalam hati adalah kegiatan membaca yang dilakukan dengan tanpa menyuarakan isi bacaan yang dibacanya Ketrampilan yang dituntut dalam membaca dalam hati antara lain sebagai berikut 1 membaca tanpa bersuara tanpa bibir bergerak tanpa ada desis apapun 2 membaca tanpa ada gerakan-gerakan kepala 3 membaca lebih cepat dibandingkan dengan membaca nyaring 4 tanpa menggunakan jari atau alat lain sebagai penunjuk 5 mengerti dan memahami bahan bacaan 6 dituntut kecepatan mata dalam membaca 7 membaca dengan pemahaman yang baik

Beranda Soal UKG Agus Zainal M (SDN Gudang 2)

Republik_ilalangyahoocoid

8 dapat menyesuaikan kecepatan dengan tingkat kesukaran yang terdapat dalam bacaan 28 KD 131 Merumuskan hakikat (pengertiantujuanjenismanfaat) membaca

Hal berikut dilakukan seseorang ketika membaca survai kecuali a memeriksa judul bacaanbuku kata pengantar daftar isi dan malihat abstrak(jika ada) b memeriksa bagian terahkir dari isi (kesimpulan) jika ada c memeriksa indeks dan apendiks(jika ada) d membaca biografi pengarang

Uraian Secara garis besar membaca dalam hati dapat dibedakan menjadi dua (I) MEMBACA EKSTENSIF amp (II) MEMBACA

INTENSIF Berikut penjelasan secara rinci kedua jenis membaca tersebut I Membaca Ekstensif membaca ekstensif adalah membaca secara luas Objeknya meliputi sebanyak mungkin teks dalam waktu yang sesingkat-singkatnya Membaca ekstensif meliputi 1 Membaca Survai (Survey Reading) Membaca survai adalah kegiatan membaca untuk mengetahui secara sekilas terhadap bahan bacaan yang akan dibaca lebih mendalam Kegiatan membaca survai merupakan pendahuluan dalam membaca ekstensif Yang dilakukan seseorang ketika membaca survai adalah sebagai berikut (a) memeriksa judul bacaanbuku kata pengantar daftar isi dan malihat abstrak(jika ada) (b) memeriksa bagian terahkir dari isi (kesimpulan) jika ada (c) memeriksa indeks dan apendiks(jika ada)

2 Membaca Sekilas Membaca sekilas atau membaca cepat adalah kegiatan membaca dengan mengandalakan kecepatan gerak mata dalam melihat dan memperhatikan bahan tertulis yang dibacanya dengan tujuan untuk mendapatkan informasi secara cepat Metode yang digunakan dalam melatihkan membaca cepat adalah (a) metode kosakata metode yang berusaha untuk menambah kosakata (b) Metode motivasi metode yang berusaha memotivasi pembaca(pemula) yang mengalami hambatan (c) Metode gerak mata metode yang mengembangkan kecepatan membaca dengan menigkatkan kecepatan gerak mata Hambatan-hambatan yang dapat mengurangi kecepatan mambaca (a) vokalisai atau berguman ketika membaca (b) membaca dengan menggerakan bibir tetapi tidak bersuara (c) kepala bergerak searah tulisan yang dibaca (d) subvokalisasi suara yang biasa ikut membaca di dalam pikiran kita (e) jari tangan selalu menunjuk tulisa yang sedang kit abaca (f) gerakan mata kembali pada kata-kata sebelumnya 3 Membaca Dangkal (Superficial Reading) membaca dangkal pada hakekatnya bertujuan untuk memperoleh pemahaman yang dangkal yang bersifat luaran yang tidak mendalam dari suatu bahan bacaan Membaca jenis ini biasanya dilakukan seseorang membaca demi kesenangan membaca bacaan ringan yang mendatangkan kesenangan kegembiraan sebagai pengisi waktu senggang

29 KD 131 Merumuskan hakikat (pengertiantujuanjenismanfaat) membaca Membaca jenis ini biasanya dilakukan seseorang membaca demi kesenangan membaca bacaan ringan yang mendatangkan kesenangan kegembiraan sebagai pengisi waktu senggang Berdasarkan karakteristik diatas kegiatan tersebut termasuk ke dalam membaca jenis a Membaca Survai (Survey Reading) b Membaca Sekilas c Membaca Dangkal (Superficial Reading)

d Membaca Nyaring Uraian No 28 30 KD 131 Merumuskan hakikat (pengertiantujuanjenismanfaat) membaca

Berikut adalah tujuan umum dalam aktifitas membaca kecuali a Membaca untuk memperoleh perincian-perincian atau fakta-fakta (reading for details or facts) Membaca tersebut

bertujuan untuk menemukan atau mengetahui penemuan-penemuan telah dilakukan oleh sang tokoh untuk memecahkan masalah-masalah yang dibuat oleh sang tokoh

b Membaca untuk mengetahui ukuran atau susunan organisasi cerita (reading for sequenceor organization) Membaca tersebut bertujuan untuk mengetahui bagian-bagian cerita dan hubungan antar bagian-bagian cerita

c Membaca untuk menyimpulkan atau membaca inferensi (reading for inference) d Membaca untuk memperoleh kekurangan suatu buku (finding mistakes)

Uraian Tujuan Membaca

Beranda Soal UKG Agus Zainal M (SDN Gudang 2)

Republik_ilalangyahoocoid

Berdasarkan maksud tujuan atau keintensifan serta cara dalam membaca di bawah ini Anderson dalam Tarigan (19799-10) mengemukakan beberapa tujuan membaca antara lain a Membaca untuk memperoleh perincian-perincian atau fakta-fakta (reading for details or facts) Membaca tersebut bertujuan untuk menemukan atau mengetahui penemuan-penemuan telah dilakukan oleh sang tokoh untuk memecahkan masalah-masalah yang dibuat oleh sang tokoh b Membaca untuk memperoleh ide-ide utama (reading for main ideas) Membaca untuk mengetahui topik atau masalah dalam bacaan Untuk menemukan ide pokok bacaan dengan membaca halamn demi halaman c Membaca untuk mengetahui ukuran atau susunan organisasi cerita (reading for sequenceor organization) Membaca tersebut bertujuan untuk mengetahui bagian-bagian cerita dan hubungan antar bagian-bagian cerita d Membaca untuk menyimpulkan atau membaca inferensi (reading for inference) Pembaca diharapkan dapat merasakan sesuatu yang dirasakan penulis e Membaca untuk mengelompokkan atau mengklasifikasikan (reading for classify) Membaca jenis ini bertujuan untuk menemukan hal-hal yang tidak wajar mengenai sesuatu hal (Anderson dalam Tarigan 197910) f Membaca untuk menilai atau mengevaluasai (reading to evaluate) Jenis membaca tersebut bertujuan menemukan suatu keberhasilan berdasarkan ukuran-ukuran tertentu Membaca jenis ini memerlukan ketelitian dengan membandingkan dan mengujinya kembali g Membaca untuk memperbandingkan atau mempertentangkan (reading to compare or contrast) Tujuan membaca tersebut adalah untuk menemukan bagaimana cara perbedaan atau persamaan dua hal atau lebih

31 KD 133 menemukan pesan pokok utama sebuah berita Simak penggalan berita berikut Hebat Siswa Indonesia Pertahankan Tradisi Emas di Olimpiade Fisika Tim Olimpiade Fisika Indonesia REPUBLIKACOID JAKARTA--Luar biasa Lima siswa Indonesia yang dikirim ke ajang Olimpiade Fisika atau International Physics Olympiad (IPhO) ke-41 di Zagreb Kroasia 17-25 Juli berhasil menyabet empat medali emas dan satu perak Pelajar yang menyumbang emas adalah Muhammad Sohibul Maromi (SMAN 1 Pamekasan Madura) Christian George Emor (SMA Lokon St Nikolaus Tomohon Sulawesi Utara) David Giovanni (SMAK Penabur Gading Serpong Banten) dan Kevin Soedyatmiko (SMAN 12 Jakarta) Sedangkan medali perak berhasil diraih oleh Ahmad Ataka Awwalur Rizqi (SMAN 1 Yogyakarta) Prestasi ini jauh lebih baik dibanding ajang Olimpiade Fisika ke-40 di Merida Yucatan Meksiko 2009 yang lalu Saat itu delegasi siswa Indonesia merebut satu medali emas dua medali perak dan satu perunggu Hasil empat medali emas dan satu perak ini hampir menyamai prestasi terbaik sebelumnya pada ajang Olimpiade Fisika ke-37 di Singapura Saat itu siswa Indonesia tidak hanya berhasil menyabet 4 medali emas namun juga meraih predikat `Absolute Winner` atas nama Mailoa Jonathan Pradana (SMAK 1 BPK Penabur Jakarta) Tapi yang terpenting lagi pelajar Indonesia berhasil mempertahankan tradisi emas di setiap ajang Olimpiade Fisika Red Endro Yuwanto Peristiwa yang diberitakan adalah a siswa Indonesia pertahankan tradisi emas di Olimpiade Matematika b siswa Indonesia pertahankan tradisi emas di Olimpiade Fisika c siswa Indonesia menyumbang emas di Olimpiade Matematika

d siswa Indonesia menyumbang perak di Olimpiade Fisika Uraian 32 KD 133 menemukan pesan pokok utama sebuah berita

Simak penggalan berita berikut Hebat Siswa Indonesia Pertahankan Tradisi Emas di Olimpiade Fisika Tim Olimpiade Fisika Indonesia REPUBLIKACOID JAKARTA--Luar biasa Lima siswa Indonesia yang dikirim ke ajang Olimpiade Fisika atau International Physics Olympiad (IPhO) ke-41 di Zagreb Kroasia 17-25 Juli berhasil menyabet empat medali emas dan satu perak Pelajar yang menyumbang emas adalah Muhammad Sohibul Maromi (SMAN 1 Pamekasan Madura) Christian George Emor (SMA Lokon St Nikolaus Tomohon Sulawesi Utara) David Giovanni (SMAK Penabur Gading Serpong Banten) dan Kevin Soedyatmiko (SMAN 12 Jakarta) Sedangkan medali perak berhasil diraih oleh Ahmad Ataka Awwalur Rizqi (SMAN 1 Yogyakarta) Prestasi ini jauh lebih baik dibanding ajang Olimpiade Fisika ke-40 di Merida Yucatan Meksiko 2009 yang lalu Saat itu delegasi siswa Indonesia merebut satu medali emas dua medali perak dan satu perunggu Hasil empat medali emas dan satu perak ini hampir menyamai prestasi terbaik sebelumnya pada ajang Olimpiade Fisika ke-37 di Singapura Saat itu siswa Indonesia tidak hanya berhasil menyabet 4 medali emas namun juga meraih predikat `Absolute Winner` atas nama Mailoa Jonathan Pradana (SMAK 1 BPK Penabur Jakarta) Tapi yang terpenting lagi pelajar Indonesia berhasil mempertahankan tradisi emas di setiap ajang Olimpiade Fisika Red Endro Yuwanto Lima siswa Indonesia berhasil mendapatkan a 4 emas 1 perak

b 1 emas 4 perak c 3 emas 2 perak d 2 perak 3 emas

Beranda Soal UKG Agus Zainal M (SDN Gudang 2)

Republik_ilalangyahoocoid

Uraian 33 KD 134 menemukan pesan pokok dalam wacana naratif seperti cerita rakyat puisi

Surat dari ibu Jika bayang telah pudar Dan elang laut pulang ke sarang Angin bertiup ke benua Tiang ndash tiang akan kering sendiri Dan nahkoda sudah tau pedoman Boleh engkau datang padaku Makna lambang dari nahkoda sudah tahu pedoman adalahhellip a Sudah mencari pedoman hidup b Sudah menemukan arah dan tujuan

c Sudah berilmu dan berpengalaman d Sudah mempunyai pasangan hidup

Uraian 34 KD 135 membandingkan berbagai jenis wacana bahasa indonesia (deskripsi narasi)

Pendekatan untuk mendapat tanggapan emosional pembaca ataupun kesan pembaca adalah contoh Pendekatan Deskripsi jenis a Pendekatan Ekspositoris b Pendekatan Impresionistik

c Pendekatan menurut sikap pengarang d Pendekatan Realistik

Uraian PENDEKATAN DESKRIPSI

Pendekatan dalam pendeskripsian dapat dibedakan menjadi beberapa kategori pendekatan yaitu

1 Pendekatan Ekspositoris

Dalam pendekatan ini kita berusaha agar deskripsi yang kita buat dapat memberi keterangan sesuai dengan keadan yang

sebenarnya sehingga pembaca dapat seolah-olah ikut melihat atau merasakan objek yang kita deskripsikan Karangan

jenis ini berisi daftar detail sesuatu secara lengkap sehingga pembaca dan penalarannya dapat memperoleh kesan

keseluruhan tentang sesuatu

2 Pendekatan Impresionistik

Tujuan deskripsi impresionistik ialah untuk mendapatkan tanggapan emosional pembaca ataupun kesan pembaca Corak

deskripsi ini diantaranya juga ditentukan oleh macam kesan apa yang diinginkan penulisnya

3 Pendekatan menurut Sikap Pengarang

Pendekatan ini sangat bergantung pada tujuan yang ingin dicapai sifat objek serta pembaca deskripsinya Dalam

menguraikan sebuah gagasan penulis mungkin mengharapkan agar pembaca merasa tidak puas terhadapa suatu

tindakan atau keadaan atau penulis menginginkan agar pembaca juga harus merasakan bahwa persoalan yang dihadapi

merupakan masalah yang gawat Penulis juga dapat membayangkan bahwa akan terjadi sesuatu yang tidak diinginkan

sehingga pembaca dari semula sudah disiapkan perasaan yang kurang enak seram takut dan sebagainya (Alkhaidah

1997) 35 kd 135 membandingkan berbagai jenis wacana bahasa indonesia (deskripsi narasi)

Prinsip-Prinsip karangan Narasi sebagai berikut kecuali a Alur b Penokohan c Amanat

d Sudut Pandang Uraian Prinsip-prinsip dasar narasi sebagai tumpuan berfikir adalah sebagai berikut

1 Alur (plot) biasa disebut dengan jalan cerita

2 Penokohan (rangkaian perbuatanaksi) 3 Latar (setting) yaitu tempat dan atau waktu terjadinya perbuatan tokoh atau peristiwa yang dialami tokoh 4 Sudut Pandang (Point of View)

a Narrator Serba Tahu (Omniscient point of view) Yaitu narrator bertindak sebagai pencipta segalanya b Narrator bertindak objektif (Objektive point of view) Pengarang tak member komentar apa pun dan pembaca bebas menafsirkan apa yang diceritakan pengarang c Narrator ikut aktif ( Narator acting) Narrator terlibat dalam cerita d Narrator sebagai peninjau

Beranda Soal UKG Agus Zainal M (SDN Gudang 2)

Republik_ilalangyahoocoid

Pelakunya adalah orang ketiga yang biasa disebut ldquodiardquo 36 KD 135 membandingkan berbagai jenis wacana bahasa indonesia (deskripsi narasi)

1 Menentukan tema atau amanat apa yang akan disampaikan 2 Menetapkan sasaran pembaca 3 Merancang peristiwa-peristiwa utama yang akan ditampilkan dalam bentuk skema perkembangandan akhir cerita 5 Memerinci peristiwa-peristiwa utama ke dalam detail-detail peristiwa sebagai pendukung cerita 6 Menyusun tokoh dan perwatakanserta latar dan sudut pandang Karakteristik diatas adalah pengembangan dari paragraf a Narasi

b Deskprisi c Ekspositoris d Sudut Pandang Persuasif

Uraian Pengembangan karangan narasi dapat dilakukan dengan langkah-langkah berikut

(1) menentukan tema atau amanat apa yang akan disampaikan (2) menetapkan sasaran pembaca (3) merancang peristiwa-peristiwa utama yang akan ditampilkan dalam bentuk skema alur (4) membagi peristiwa utama ke dalam ke dalam bagian awal perkembangan dan akhir cerita (5) memerinci peristiwa-peristiwa utama ke dalam detail-detail peristiwa sebagai pendukung cerita dan (6) menyusun tokoh dan perwatakan latar dan sudut pandang

37 KD 135 membandingkan berbagai jenis wacana bahasa indonesia (deskripsi narasi) Karangan narasi yang berusaha untuk memberikan suatu maksud tertentu menyampaikan suatu amanat terselubung kepada para pembaca atau pendengar sehingga tampak seolah-olah melihat disebut a Narasi Informatif b Narasi Ekspositoris c Narasi Sugestif d Narasi Artistik

Uraian Jenis-Jenis Narasi

Narasi Informatif Narasi informatif adalah narasi yang memiliki sasaran penyampaian informasi secara tepat tentang suatu peristiwa

dengan tujuan memperluas pengetahuan orang tentang kisah seseorang Narasi Ekspositorik

Narasi ekspositorik adalah narasi yang memiliki sasaran penyampaian informasi secara tepat tentang suatu peristiwa dengan tujuan memperluas pengetahuan orang tentang kisah seseorang Dalam narasi ekspositorik penulis menceritakan suatu peristiwa berdasarkan data yang sebenarnya Pelaku yang ditonjolkan biasanya satu orang Pelaku diceritakan mulai dari kecil sampai saat ini atau sampai terakhir dalam kehidupannya Karangan narasi ini diwarnai oleh eksposisi maka ketentuan eksposisi juga berlaku pada penulisan narasi ekspositprik Ketentuan ini berkaitan dengan penggunaan bahasa yang logis berdasarkan fakta yang ada tidak memasukan unsursugestif atau bersifat objektif

Narasi Artistik

Narasi artistik adalah narasi yang berusaha untuk memberikan suatu maksud tertentu menyampaikan suatu amanat terselubung kepada para pembaca atau pendengar sehingga tampak seolah-olah melihat Ketentuan ini berkaitan dengan penggunaan bahasa yang logis berdasarkan fakta yang ada tidak memasukan unsur sugestif atau bersifat objektif

Narasi Sugestif Narasi sugestif adalah narasi yang berusaha untuk memberikan suatu maksud tertentu menyampaikan suatu amanat

terselubung kepada para pembaca atau pendengar sehingga tampak seolah-olah melihat 38 KD 135 membandingkan berbagai jenis wacana bahasa indonesia (deskripsi narasi)

Ciri karangan Narasi yang benar kecuali a Menonjolkan unsur perbuatan atau tindakan Membuat pembaca atau pendengar merasakan sendiri atau

mengalami sendiri

b Ada konfiks menjawab pertanyaan apa yang terjadi c Dirangkai dalam urutan waktu menggambarkan dengan jelas suatu peristiwa d Berisi ajakan dirangkai dalam urutan waktu

Uraian Ciri-Ciri Karangan Narasi Menurut Gorys Keraf (2000136)

Menonjolkan unsur perbuatan atau tindakan Dirangkai dalam urutan waktu Berusaha menjawab pertanyaan apa yang terjadi Ada konfiks

39 kd 135 membandingkan berbagai jenis wacana bahasa indonesia (deskripsi narasi) Langkah menyusun paragraf deskripsi yang benar adalah (1)Tentukan objek atau tema yang akan dideskripsikan (2)Mengumpulkan data dengan mengamati objek yang akan dideskripsikan (3)Menyusun data tersebut ke dalam urutan yang baik (menyusun kerangka karangan) (4)Menguraikan kerangka karangan menjadi dekripsi yang sesuai dengan tema yang ditentukan

Beranda Soal UKG Agus Zainal M (SDN Gudang 2)

Republik_ilalangyahoocoid

(5)Tentukan tujuan a 1-5-2-3-4

b 1-5-4-3-2 c 1-5-3-2-4 d 1-3-4-2-5

Uraian Langkah menyusun deskripsi

1Tentukan objek atau tema yang akan dideskripsikan 2Tentukan tujuan 3Mengumpulkan data dengan mengamati objek yang akan dideskripsikan 4Menyusun data tersebut ke dalam urutan yang baik (menyusun kerangka karangan) 5Menguraikan kerangka karangan menjadi dekripsi yang sesuai dengan tema yang ditentukan

40 Kd 311 Merancang aktivitas pembelajaran berdasarkan prinsip dan teori pembelajaran matematika Dalam tahap ini penyajian yang dilakukan melalui tindakan anak secara langsung terlihat dalam memanipulasi (mengotak atik)objek Proses tersebut menurut Bruner dikategorikan ke dalam model a Tahap Enaktif

b Tahap Ikonik c Tahap Simbolik d Tahap Implikatif

Uraian 1 Model Tahap Enaktif

Dalam tahap ini penyajian yang dilakukan melalui tindakan anak secara langsung terlihat dalam memanipulasi (mengotak atik)objek Contoh Budi mempunyai 2 pensil kemudian ibunya memberikannya lagi 3 pinsil Berapa banyak pensil Budi sekarang

2 Model Tahap Ikonik Dalam tahap ini kegiatan penyajian dilakukan berdasarkan pada pikiran internal dimana pengetahuan disajikan melalui serangkaian gambar-gambar atau grafik yang dilakukan anak berhubungan dengan mental yang merupakan gambaran dari objek-objek yang dimanipulasinya Contoh + = hellip

3 Model Tahap Simbolis Dalam tahap ini bahasa adalah pola dasar simbolik anak memanipulasi Simbol-simbol atau lambang-lambang objek tertentu Contoh 2 pensil + 3 pensil = hellip pensil

41 Kd 311 Merancang aktivitas pembelajaran berdasarkan prinsip dan teori pembelajaran matematika Dalam tahap ini kegiatan penyajian dilakukan berdasarkan pada pikiran internal dimana pengetahuan disajikan melalui serangkaian gambar-gambar atau grafik yang dilakukan anak Proses tersebut menurut Bruner dikategorikan ke dalam model a Tahap Enaktif b Tahap Ikonik

c Tahap Simbolik d Tahap Implikatif

Uraian No 40 42 Kd 311 Merancang aktivitas pembelajaran berdasarkan prinsip dan teori pembelajaran matematika

Objek belajar matematika dibagi kedalam Objek Langsung dan Objek Tak Langsung Hal tersebut adalah teori belajar Matematika menurut a Robert M Gagne

b Jerome S Burner c Thorndike d Skinner

Uraian Teori yang diperkenalkan Robert MGagne pada tahun 1960-an pembelajaran harus dikondisikan untuk memunculkan

respons yang diharapkanMenurut Gagne (dalam Ismail 1998) belajar matematika terdiri dari objek langsung dan objek tak langsung

43 Kd 311 Merancang aktivitas pembelajaran berdasarkan prinsip dan teori pembelajaran matematika Perhatikan contoh berikut Budi mempunyai 2 pinsil kemudian ibunya memberikannya lagi 3 pinsilBerapa banyak pinsil Budi sekarang Hal tersebut dikemukakan Bruner dalam Proses Pembelajaran Matematika dalam tahap a Simbolik b Ikonik c Implikatif d Enaktif

Uraian No 40

Beranda Soal UKG Agus Zainal M (SDN Gudang 2)

Republik_ilalangyahoocoid

44 Kd 311 Merancang aktivitas pembelajaran berdasarkan prinsip dan teori pembelajaran matematika Perhatikan gambar disamping Ilustrasi di samping dikemukakan Bruner dalam Proses Pembelajaran Matematika dalam tahap

a Simbolik b Ikonik

c Implikatif d Enaktif

Uraian No 40 45 Kd 311 Merancang aktivitas pembelajaran berdasarkan prinsip dan teori pembelajaran matematika

Perhatikan contoh berikut Contoh 2 pinsil + 3 pinsil = hellippinsil Contoh tersebut dikemukakan Bruner dalam Proses Pembelajaran Matematika dalam tahap a Simbolik

b Ikonik c Implikatif d Enaktif

Uraian No 40 46 Kd 311 Merancang aktivitas pembelajaran berdasarkan prinsip dan teori pembelajaran matematika

Perhatikan contoh berikut penjumlahan bilangan positif dan negatif siswa mencoba sendiri dengan menggunakan garis bilangan Contoh tersebut dikemukakan Bruner dalam Teorema Pembelajaran Matematika a Penyusunan

b Notasi c Pengkontrasan dan Keanekaragaman d Pengaitan

Uraian Berdasarkan hasil pengamatannya Brunner merumuskan 5 teorema dalam pembelajaran matematika yaitu

1) Teorema Penyusunan Menerangkan bahwa cara yang terbaik memulai belajar suatu konsep matematika dalil defenisi dan semacamnya adalah dengan cara menyusun penyajiannya Misalnya dalam mempelajari penjumlahan bilangan positif dan negatif siswa mencoba sendiri dengan menggunakan garis bilangan 2) Teorema Notasi Menerangkan bahwa dalam pengajaran suatu konsep penggunaan notasi-notasi matematika harus diberikan secara bertahap dari yang sederhana ke yang lebih kompleks 3) Teorema Pengkontrasan dan Keanekaragaman Menerangkan bahwa pengontrasan dan keanekaragaman sangat penting dalam melakukan pengubahan konsep matematika dari yang konkrit ke yang lebih abstrak Dalam hal ini diperlukan banyak contoh Contoh yang diberikan harus sesuai dengan rumusan yang diberikan Misalnya menjelaskan persegi panjang disertai juga kemungkinan jajaran genjang dan segi empat lainnya selain persegi panjnag Dengan demikian siswa dapat membedakan apakah segi empat yang diberikan padanya termasuk persegi panjang atau tidak 4) Teorema Pengaitan Menerangkan bahwa dalam matematika terdapat hubungan yang berkaitan antara satu konsep dengan konsep yang lain Di mana materi yang satu merupakan prasyarat yang harus diketahui untuk mempelajari materi yang lain

47 Kd 311 Merancang aktivitas pembelajaran berdasarkan prinsip dan teori pembelajaran matematika Perhatikan contoh berikut Guru menjelaskan persegi panjang disertai juga kemungkinan jajaran genjang dan segi empat lainnya selain persegi panjnag Dengan demikian siswa dapat membedakan apakah segi empat yang diberikan padanya termasuk persegi panjang atau tidak Contoh tersebut dikemukakan Bruner dalam Teorema Pembelajaran Matematika a Penyusunan b Notasi c Pengkontrasan dan Keanekaragaman

d Pengaitan Uraian No 46 48 Kd 311 Merancang aktivitas pembelajaran berdasarkan prinsip dan teori pembelajaran matematika

Tahap pembelajaran Matematika menurut Van Halle adalah sebagai berikut kecuali a Tahap Pengenalan b Tahap Pengurutan c Tahap Analisis d Tahap Penyimpulan

Beranda Soal UKG Agus Zainal M (SDN Gudang 2)

Republik_ilalangyahoocoid

Uraian Van Hiele menyatakan bahwa terdapat 5 tahap belajar siswa dalam belajar geometri yaitu

a Tahap Pengenalan Pada tahap ini siswa mulai belajar mengenal suatu bangun geometri secara keseluruhan namun belum mampu mengetahui adanya sifat-sifat dari bangun geometri yang dilihatnya b Tahap Analisis Pada tahap ini siswa sudah mulai mengenal sifat-sifat yang dimiliki bangun geometri yang diamatinya c Tahap Pengurutan Pada tahap ini siswa sudah mengenal dan memahami sifat-sifat suatu bangun geometri serta sudah dapat mengurutkan bangun-bangun geometri yang satu sama yang lainnya saling berhubungan d Tahap Deduksi Pada tahap ini siswa telah mampu menarik kesimpulan secara deduktif yaitu menarik kesimpulan yang bersifat umum dan menuju ke hal yang bersifat khusus serta dapat mengambil kesimpulan e Tahap Akurasi Pada tahap ini siswa mulai menyadari pentingnya ketepatan prinsip-prinsip dasar yang melandasi suatu pembuktian Tahap berfikir ini merupakan tahap berfikir yang paling tinggi rumit dan kompleks karena di luar jangkauan usia anak-anak SD sampai tingakat SMP

49 Kd 311 Merancang aktivitas pembelajaran berdasarkan prinsip dan teori pembelajaran matematika middot Matematika untuk tujuan pembelajaran dianalisis sebagai kumpulan fakta yang berdiri sendiri dan tidak saling berkaitan middot Anak diharuskan menguasai unsur-unsur yang banyak sekali tanpa diperhatikan pengertiannya middot Anak mempelajari unsur-unsur dalam bentuk seperti yang akan digunakan nanti dalam kesempatan lain middot Anak akan mencapai tujuan ini secara efektif dan efisien dengan melalui pengulangan Teori Pembelajaran Matematika tersebut dikemukakan oleh a Skinner b Piaget c Van Brownell

d Thorndike Uraian Brownell mengemukakan tentang Teori Makna (Meaning Theory) sebagai pengganti Teori Latihan HafalUlangan (Drill

Theory) Intisari dari teori Drill adalah - Matematika untuk tujuan pembelajaran dianalisis sebagai kumpulan fakta yang berdiri sendiri dan tidak saling berkaitan - Anak diharuskan menguasai unsur-unsur yang banyak sekali tanpa diperhatikan pengertiannya - Anak mempelajari unsur-unsur dalam bentuk seperti yang akan digunakan nanti dalam kesempatan lain - Anak akan mencapai tujuan ini secara efektif dan efisien dengan melalui pengulangan

50 Kd 311 Merancang aktivitas pembelajaran berdasarkan prinsip dan teori pembelajaran matematika Tahap operasionaloperasi konkrit adalah teori Pembelajaran Matematika yang dikemukakan Peaget pada tahap usia a dibawah 2 tahun b 2-7 Tahun c 7-8 Tahun d 7-12 Tahun

Uraian Piaget membagi skema yang digunakan anak untuk memahami dunianya melalui empat periode utama yang berkorelasi

dengan dan semakin canggih seiring pertambahan usia Periode sensorimotor (usia 0ndash2 tahun) Periode praoperasional (usia 2ndash7 tahun) Periode operasional konkrit (usia 7ndash12 tahun) Periode operasional formal (usia 12 tahun sampai dewasa)

51 313 Memilih media pembelajaran yang tepat untuk pembelajaran operasi bilangan bulat Untuk operasi bilangan bulat perkalian media yang paling tepat digunakan adalah a sapu lidi

b kerikil c daun pakis d koin

Uraian 52 313 Memilih media pembelajaran yang tepat untuk pembelajaran operasi bilangan bulat

Untuk operasi bilangan bulat negatif kita bisa menggunakan media a uang kertas b kerikil c mistar d koin

Uraian 53 313 Memilih media pembelajaran yang tepat untuk pembelajaran operasi bilangan bulat

Beranda Soal UKG Agus Zainal M (SDN Gudang 2)

Republik_ilalangyahoocoid

Untuk operasi bilangan bulat penjumlahan media yang paling tepat digunakan adalah a uang kertas b kerikil c daun pakis d koin

Uraian 54 314 Memilih media pembelajaran yang tepat untuk pembelajaran operasi bilangan pecahan

Untuk operasi bilangan pecahan kita bisa menggunakan media berikut kecuali a kartu bilangan b garis bilangan c gambar bidang d blok pecahan

Uraian 55 316 Memilih media pembelajaran yang tepat untuk pembelajaran geometri dan pengukuran

Untuk menghitung luas dan keliling bangun datar media yang paling tepat digunakan adalah a Penggaris b Kertas Folio Bergaris

c Neraca d Kalkulator

Uraian

Page 2: 50 SOAL UKG & PEMBAHASAN SESUAI KISI-KISIS.pdf

Beranda Soal UKG Agus Zainal M (SDN Gudang 2)

Republik_ilalangyahoocoid

Uraian No 2 5 KD 11 Analisis Perkembangan Bahasa Anak Usia SD

Anak dapat menampilkan sifat ingin tahu adalah karakteristik perkembangan bahasa anak usia SD ditinjau dari segi a Mental

b Psikomotorik c Sosial d Emosional

Uraian No 1 6 KD 11 Analisis Perkembangan Bahasa Anak Usia SD

Anak dapat menunjukkan sikap marah dalam kondisi yang wajar adalah karakteristik perkembangan bahasa anak usia SD ditinjau dari segi a Mental b Psikomotorik c Kognitif d Emosional

Uraian No 3 7 KD 11 Analisis Perkembangan Bahasa Anak Usia SD

Faktor kendala yang mempengaruhi keterampilan berbahasa anak adalah sebagai berikut kecuali a Jenis Kelamin b Keluarga c Keinginan dan Dorongan Komunikasi d Kebiasaan

Uraian Faktor dan Kendala dalam Mempelajari Keterampilan Berbahasa

1 Kesehatan 2 Kecerdasan 3 Jenis kelamin 4 Keluarga 5 Keinginan dan dorongan untuk berkomunikasi serta hubungan dengan teman sebaya 6 Kepribadian

8 KD 112 Aspek Membaca di kelas rendah Tahapan proses belajar membaca bagi siswa sekolah dasar kelas awal biasanya menggunakan metode membaca a Ejaan per ejaan

b Skimming c Permulaan d Cepat

Uraian Metode membaca permulaan

1) Metode Ejaalfabet 2) Metode Bunyi 3) Metode Suku Kata 4) Metode Kata 5) Metode Global 6) Metode SAS

9 KD 112 Aspek Membaca di kelas rendah Dalam pembelajaran membaca permulaan ada beberapa metode yang dapat digunakan kecuali a metode kupas rangkai suku kata b metode kata lembaga

c metode SAS d metode eja

Uraian No 8 10 KD 112 Aspek Membaca di kelas rendah

Berikut diberikan studi kasus Mula-mula diberikan kalimat secara keseluruhan Kalimat itu diuraikan atas kata-kata yang mendukungnya Dari kata-kata itu kita ceraikan atas suku-suku katanya dan akhirnya atas huruf-hurufnya Kemudian huruf-huruf itu kita sintetiskan kembali menjadi suku kata suku kata menjadi kata dan kata menjadi kalimat Berdasarkan studi kasus metode membaca permulaan yang tepat digunakan adalah a Metode Alfabet b Metode Suku Kata c Metode SAS

Beranda Soal UKG Agus Zainal M (SDN Gudang 2)

Republik_ilalangyahoocoid

d Metode Cerita Uraian 11 KD 112 Aspek Membaca di kelas rendah

Kelebihan membaca permulaan adalah sebagai berikut a Mempunyai nilai strategis bagi pengembangan kepribadian dan kemampuan siswa

b Meningkatkan nilai siswa dalam pelajaran Bahasa Indonesia c Meningkatkan kemandirian siswa dalam membaca d Mempermudah menghafal kata-kata

Uraian Kelebihan membaca permulaan

- Dapat memperoleh informasi secara tepat dan lengkap - Mempunyai nilai strategis bagi pengembangan kepribadian dan kemampuan siswa - Membaca permulaan juga dapat mengembangkan nilai-nilai moral kemampuan bernalar dan kreatifitas seseorang Kekurangan membaca permulaan - Kemampuan membaca pada umumnya lebih rendah - Menghambat kemampuan untuk memahami kalimat atau cerita - Mengalami kesulitan menjawab mengenai isi cerita karena kesibukan siswa mengeja dan menyuarakan huruf-huruf - Kecepata membaca dan pemahaman siswa sangat rendah

12 KD 112 Aspek Membaca di kelas rendah Metode yang merupakan penyempurnaan metode alphabet dengan mengajarkan bunyi-bunyi bahasa sebagai pengganti huruf-huruf berdasarkan ucapan hurufnya adalah metode membaca permulaan a Metode Suku Kata b Metode Kata-kata c Metode Cerita d Metode Suara

Uraian Teknik Membaca Permulaan

1) Metode Alfabet Metode ini sering disebut metode harfiah Metode ldquoletter by letter methodrdquo atau ldquoABC methodrdquo Mula-mula dikenalkan abjad dari A sampai Z Setelah hafal beberapa huruf barulah huruf-huruf tersebut dirangkai menjadi suku kata Suku kata-suku kata tersebut setelah dikenalkan barulah dirangkai menjadi kata Dengan modal beberapa kata ituterciptalah kalimat 2) Metode Suara Metode suara juga disebut rdquoPhonic Methodrdquo Metode ini merupakan penyempurnaan metode alphabet Pada metode ini bukan abjadnya yang di ajarkan melainkan bunyi-bunyi bahasa sebagai pengganti huruf-huruf tersebutUcapan huruf-huruf tidak berdasarkan atas bunyi abjadnya melainkan ucapan hurufnya 3) Metode Suku Kata Metode ini biasa juga disebut rdquoSyllabic Methodrdquo Dalam metode ini suku kata merupakan kunci pokok dalam membuat kata Metode suku kata sebenarnya suatu metode pengajaran membaca permulaan yang di dasarkan atas kata-kata dianalisis menjadi suku kata-suku kataSuku kata0suku kata tersebut di intesiskan kembali menjadi kata-kata 4) Metode Kata-kata Pelaksanaan metode ini selalu di awali dengan kata-kata tertulis da ndash da gi ndash gi ku ndash ku dan lain-lain Setelah kata-kata itu dikenalkan kata-kata tersebut dianalisis atas suku katanya Selanjitnya suku kata itu di analisis lagi atas huruf-hurufnya Barulah huruf-huruf itu digabungkan kembali dalam bentuk kata semula 5) Metode Cerita Pelaksanaan metode cerita dalam mengajarkan membaca permulaan diawali dengan menghafalkan cerita atau sebuah puisi Cerita atau puisi itu diuraikan atas kalimat-kalimtnya sampai pada kata-katanya Dalam mengucapkan kata0kata metode ini menggunakan kata-kata fonetik 6) Metode Struktural Analitik Sintetik (SAS) Adapun contoh pelaksanaan metode SAS sebagai berikut Mula-mula diberikan kalimat secara keseluruhan Kalimat itu diuraikan atas kata-kata yang mendukungnya Dari kata-kata itu kita ceraikan atas suku-suku katanya dan akhirnya atas huruf-hurufnya Kemidian huruf-huruf itu kita sintetiskan kembali menjadi suku kata suku kata menjadi kata dan kata menjadi kalimat

13 KD 113 Aspek Menulis di kelas tinggi Pendekatan menekankan keterpaduan empat aspek keterampilan berbahasa (menyimak berbicara membaca dan menulis) dalam pembelajaran adalah salah satu Pendekatan yang disarankan dalam pembelajaran menulis yang disebut a pendekatan komunikatif b Pendekatan integratif

c Pendekatan keterampilan proses d Pendekatan tematis

Uraian Pendekatan yang disarankan dalam pembelajaran menulis

1) Pendekatan komunikatif memfokuskan pada keterampilan siswa mengimplementasikan fungsi bahasa (untuk berkomunikasi) dalam pembelajaran

Beranda Soal UKG Agus Zainal M (SDN Gudang 2)

Republik_ilalangyahoocoid

2) Pendekatan integratif menekankan keterpaduan empat aspek keterampilan berbahasa (menyimak berbicara membaca dan menulis) dalam pembelajaran

3) Pendekatan keterampilan proses memfokuskan keterampilan siswa dalam mengamati mengklasifikasi menginterpretasi dan mengkomunikasikan

4) Pendekatan tematis menekankan tema pembelajaran sebagai payungpemandu dalam pembelajaran 14 KD 113 Aspek Menulis di kelas tinggi

Teknik menulis cerita terdiri atas hal-hal sebagai berikut kecuali a menjawab pertanyaan b membuat kalimat c subtitusi d persuasi

Uraian Teknik menyusun cerita dapat dilakukan dengan menjawab pertanyaan melengkapi kalimat memperbaiki susunan kalimat

memperluas kalimat subtitusi transfomtasi dan membuat kalimat Persuasi adalah bujukan halus

15 KD 113 Aspek Menulis di kelas tinggi Model pembelajaran menulis ceritacerpen di SD meliputi hal-hal berikut kecuali a Menceritakan gambar b Melanjutkan cerita c Menceritakan pengalaman d Mendeskripsikan cerita

Uraian Model pembelajaran menulis ceritacerpen di SD meliputi menceritakan gambar melanjutkan ceria lain menceitakan

mimpi menceritakan pengalaman dan menceritakan cita-cita 16 KD 121 Memilih berbagai Metode Menulis Permulaan

1) Menulis huruf lepas 2) Merangkaikan huruf lepas menjadi suku kata 3) Merangkaikan suku kata menjadi kata 4) Menyusun kata menjadi kalimat (Djauzak 19964) Tahapan diatas adalah Teknik Menulis Permulaan dengan metode a Metode Eja

b Metode Kata Lembaga c Metode SAS d Metode Global

Uraian Metode dan pembelajaran menulis permulaan

a Metode Eja Metode eja di dasarkan pada pendekatan harfiah artinya belajar membaca dan menulis dimulai dari huruf-huruf yang dirangkaikan menjadi suku kata Oleh karena itu pengajaran dimulai dari pengenalan huruf-huruf Demikian halnya dengan pengajaran menulis di mulai dari huruf lepas dengan langka-langkah sebagai berikut 1) Menulis huruf lepas 2) Merangkaikan huruf lepas menjadi suku kata 3) Merangkaikan suku kata menjadi kata 4) Menyusun kata menjadi kalimat (Djauzak 19964) b Metode kata lembaga Metode kata lembaga di mulai mengajar dengan langkah-langkah sebagai berikut 1) Mengenalkan kata 2) Merangkaikan kata antar suku kata 3) Menguraikan suku kata atas huruf-hurufnya 4) Menggabungkan huruf menjadi kata (Djauzak 19965) c Metode Global Metode global memulai pengajaran membaca dan menulis permulaan dengan membaca kalimat secara utuh yang ada di bawah gambar Menguraikan kalimat dengan kata-kata menguraikan kata-kata menjadi suku kata (Djauzak 19966) d Metode SAS Menuryut (Supriyadi 1996 334-335) pengertian metode SAS adalah suatu pendekatan cerita di sertai dengan gambar yang didalamnya terkandung unsur analitik sintetik Metode SAS menurut (Djuzak19968) adalah suatu pembelajaran menulis permulaan yang didasarkan atas pendekatan cerita yakni cara memulai mengajar menulis dengan menampil cerita yang diambil dari dialog siswa dan guru atau siswa dengan siswa Teknik pelaksanaan pembelajaran metode SAS yakni keterampilan menulis kartu huruf kartu suku kata kartu kata dan kartu kalimat sementara sebagian siswa mencari huruf suku kata dan kata guru dan sebagian siswa menempel kata-kata yang tersusun sehingga menjadi kalimat yang berarti (Subana) Proses operasional metode SAS mempunyai langkah-lagkah dengan urutan sebagai berikut a Struktur yaitu menampilkan keseluruhan b Analitik yatu melakukan proses penguraian c Sintetik yaitu melakukan penggalan pada struktur semula

Beranda Soal UKG Agus Zainal M (SDN Gudang 2)

Republik_ilalangyahoocoid

17 KD 121 Memilih berbagai Metode Menulis Permulaan 1) Mengenalkan kata 2) Merangkaikan kata antar suku kata 3) Menguraikan suku kata atas huruf-hurufnya 4) Menggabungkan huruf menjadi kata (Djauzak 19965) Tahapan diatas adalah Teknik Menulis Permulaan dengan metode a Metode Eja b Metode Kata Lembaga

c Metode SAS d Metode Global

Uraian No 16 18 KD 121 Memilih berbagai Metode Menulis Permulaan

Metode dengan memulai pengajaran membaca dan menulis permulaan dengan membaca kalimat secara utuh yang ada di bawah gambar Menguraikan kalimat dengan kata-kata menguraikan kata-kata menjadi suku kata disebut dengan metode a Metode Eja b Metode Kata Lembaga c Metode SAS d Metode Global

Uraian No 16 19 KD 121 Memilih berbagai Metode Menulis Permulaan

Suatu pembelajaran menulis permulaan yang didasarkan atas pendekatan cerita yakni cara memulai mengajar menulis dengan menampil cerita yang diambil dari dialog siswa dan guru atau siswa dengan siswa disebut dengan metode a Metode Eja b Metode Kata Lembaga c Metode SAS

d Metode Global Uraian No 16 20 KD 121 Memilih berbagai Metode Menulis Permulaan

menulis kartu huruf kartu suku kata kartu kata dan kartu kalimat sementara sebagian siswa mencari huruf suku kata dan kata guru dan sebagian siswa menempel kata-kata yang tersusun sehingga menjadi kalimat yang berarti adalah contoh metode a Metode Eja b Metode Kata Lembaga c Metode SAS

d Metode Global Uraian No 16 21 KD 122 Merancang berbagai kegiatan menulis di kelas tinggi

Berikut adalah kegiatan menulis lanjutan di kelas tinggi kecuali a menulis tentang berbagai topik b menulis pengumuman c menulis pantun d menulis memo

Uraian 22 KD 123 Perencanaan dan Pelaksanaan evaluasi pembelajaran Bahasa dan Sastra Indonesia

Perencanaan Pengajaran meliputi hal-hal berikut kecuali a tujuan apa yang hendak dicapai b memilih bahan ajar c proses belajar mengajar d alat penilaian

Uraian Menurut Akhlan dan Rahman (199715) perencanaan pengajaran meliputi

a tujuan apa yang hendak dicapai b bahan pengajaran c proses belajar mengajardan d alat penilaian

23 KD 123 Perencanaan dan Pelaksanaan evaluasi pembelajaran Bahasa dan Sastra Indonesia karakteristik perencanaan pengajaran yang baik hendaknya mengandung prinsip sebagai berikut a Memiliki sikap objektif rasio (tepat dan masuk akal) komprehensif dan sistematis (menyeluruh dan tersusun rapi)

b Merupakan suatu wahana atau wadah untuk mengembangkan segala potensi yang ada dan dimiliki oleh anak

Beranda Soal UKG Agus Zainal M (SDN Gudang 2)

Republik_ilalangyahoocoid

didik c Mengendalikan kekuatan sendiri bukan didasarkan atas kekuatan orang lain d Melakukan studi kasus yang berkesinambungan

Uraian Menurut Akhlan dan Rahman (19977) karakteristik perencanaan pengajaran yang baik hendaknya mengandung prinsip

sebagai berikut a Mengembangkan hubungan interaksi yang baik di antara sesama manusia dalam hal ini siswa dan guru serta personal

terkait b Merupakan suatu wahana atau wadah untuk mengembangkan segala potensi yang ada dan dimiliki oleh anak didik c Memiliki sikap objektif rasio (tepat dan masuk akal) komprehensif dan sistematis (menyeluruh dan tersusun rapi) d Mengendalikan kekuatan sendiri bukan didasarkan atas kekuatan orang lain Didukung oleh fakta dan data yang

menunjang pencapaian tujuan yang telah di dirumuskan e Fleksibel dan dinamis artinya mudah disesuaikan dengan keadaan serta perkembangan ke arah yang lebih baik dan

maju 24 KD 123 Perencanaan dan Pelaksanaan evaluasi pembelajaran Bahasa dan Sastra Indonesia

tahap mengumpulkan informasi tentang keadaan objek evaluasi (siswa) dengan menggunakan teknik tes atau nontes disebut tahapan a Tahap Tindak Lanjut b Tahap Persiapan c Tahap Pelaksanaan

d Tahap Pengolahan Hasil Uraian Berikut ini penjelasan singkat tentang keempat tahap evaluasi pembelajaran tersebut

(1) Tahap Persiapan Menurut Damaianti (2007 8) tahap ini disebut juga tahap perencanaan dan perumusan kriterium Langkahnya meliputi (a) perumusan tujuan evaluasi (b) penetapan aspek-aspek yang akan dievaluasi (c) menetapkan metode dan bentuk evaluasi (tesnontes) (d) merencanakan waktu evaluasi (e) melakukan uji coba (untuk tes) agar dapat mengukur validitas dan reliabilitasnya Untuk evaluasi yang menggunakan tes hasil dari tahap ini adalah kisi-kisi soal dan seperangkat alat tes soal lembar jawaban (untuk tes tulis) kunci jawaban dan pedoman penilaian 2) Tahap Pelaksanaan Tahap pelaksanaan atau disebut juga dengan tahap pengukuran dan pengumpulan data adalah tahap untuk mengumpulkan informasi tentang keadaan objek evaluasi (siswa) dengan menggunakan teknik tes atau nontes Bila menggunakan teknik tes soal yang igunakan sebaiknya sudah teruji validitas dan reliabilitasnya Tes yang digunakan dapat berbentuk tes tulis lisan atau praktik 3) Tahap Pengolahan Hasil Tahap pengolahan hasil adalah tahap pemeriksaan hasil evaluasi dengan memberikan skor Skor yang diperoleh siswa selanjutnya diubah menjadi nilai Pada tes tulis pemeriksaan hasil dilakukan setelah tes selesai sedangkan pada tes lisan dan praktik pemberian nilai dilakukan bersamaan dengan waktu pelasanaan tes tersebut 4) Tahap Tindak Lanjut Tahap tindak lanjut atau disebut juga tahap penafsiran adalah tahap untuk mengambil keputusan berdasarkan nilai yang dihasilkan pada tahap pengolahan hasil misalnya a memperbaiki proses belajar mengajar b memperbaiki kesulitan belajar siswa c memperbaiki alat evaluasi d membuat laporan evaluasi (rapor)

25 KD 131 Merumuskan hakikat (pengertiantujuanjenismanfaat) membaca membaca yang mengutamakan isi bacaan sebagai ungkapan pikiran perasaan dan kehendak penulis Bila hanya ingin mengetahui isinya membaca cerdas bersifat lugas Akan tetapi bila maksudnya untuk memahami dan memilki isi bacaan maka tergolong kedalam membaca jenis a Membaca cerdas atau membaca dalam hati

b Membaca bahasa c Membaca teknis d Membaca bebas

Uraian Menurut Soedjono dalam Sue (200418-21) ada lima macam membaca yaitu membaca bahasa membaca cerdas atau

membaca dalam hati membaca teknis membaca emosional dan membaca bebas 1) Membaca bahasa Membaca bahasa adalah membaca yang mengutamakan bahasa bacaan Membaca bahasa mementingkan segi bahasa bacaan Hal-hal yang perlu diperhatikan dalam membaca bahasa adalah kesesuian pikir dengan bahasa perbendaharaan bahasa yang meliputi kosa kata struktur kalimat dan ejaan 2) Membaca cerdas atau membaca dalam hati

Beranda Soal UKG Agus Zainal M (SDN Gudang 2)

Republik_ilalangyahoocoid

Membaca cerdas adalah membaca yang mengutamakan isi bacaan sebagai ungkapan pikiran perasaan dan kehendak penulis Bila hanya ingin mengetahui isinya membaca cerdas bersifat lugas Akan tetapi bial maksudnya untuk memahami dan memilki isi bacaan maka disebut membaca belajar 3) Membaca teknis Membaca teknis adalah membaca dengan mengarahkan bacaan secara wajar Wajar maksudnya sesuai ucapan tekanan dan intonasinya Pikiran perasaan dan kemauan yang tersimpan dalam bacaan dapat diaktualisasikan dengan baik 4) Membaca emosional Membaca emosional adalah membaca sebagai sarana untuk memasuki perasaan yaitu keindahan isi dan keindahan bahasanya 5) Membaca bebas Membaca bebas adalah membaca sesuatu atas kehendak sendiri tanpa adanya unsur paksaan dari luar Unsur dari luar misalnya guru orang tua teman atau pihak-pihak lain

26 KD 131 Merumuskan hakikat (pengertiantujuanjenismanfaat) membaca membaca sesuatu atas kehendak sendiri tanpa adanya unsur paksaan dari luar Unsur dari luar misalnya guru orang tua teman atau pihak-pihak lain termasuk jenis membaca a Membaca cerdas atau membaca dalam hati b Membaca bahasa c Membaca teknis d Membaca bebas

Uraian No 25 27 KD 131 Merumuskan hakikat (pengertiantujuanjenismanfaat) membaca

1 menggunakan ucapan yang tepat 2 menggunakan frase yang tepat 3 menggunakan intonasi suara yang wajar 4 dalam posisi sikap yang baik 5 menguasai tanda-tanda baca 6 membaca dengan terang dan jelas keterampilan di atas harus dipunyai dalam jenis membaca a Membaca cerdas atau membaca dalam hati b Membaca bahasa c Membaca teknis d Membaca nyaring

Uraian JENIS-JENIS MEMBACA DAN KARAKTERISTIKNYA

Ditinjau dari segi terdengar atau tidaknya suara pembaca waktu melakukan kegiatan membaca maka proses membaca dapat dibedakan menjadi A Membaca Nyaring Membaca nyaring adalah kegiatan membaca dengan menyuarakan tulisan yang dibacanya dengan ucapan dan intonasi yang tepat agar pendengar dan pembaca dapat menangkap informasi yang disampaikan oleh penulis baik yang berupa pikiran perasaan sikap ataupun pengalaman penulis Ketrampilan yang dituntut dalam membaca nyaring adalah berbagai kemampuan diantaranya adalah 1 menggunakan ucapan yang tepat 2 menggunakan frase yang tepat 3 menggunakan intonasi suara yang wajar 4 dalam posisi sikap yang baik 5 menguasai tanda-tanda baca 6 membaca dengan terang dan jelas 7 membaca dengan penuh perasaan ekspresif 8 membaca dengan tidak terbata-bata 9 mengerti serta memahami bahan bacaan yang dibacanya 10 kecepatan bergantung pada bahan bacaan yang dibacanya 11 membaca dengan tanpa terus-menerus melihat bahan bacaan 12 membaca dengan penuh kepercayaan pada diri sendiri B Membaca Dalam Hati Membaca dalam hati adalah kegiatan membaca yang dilakukan dengan tanpa menyuarakan isi bacaan yang dibacanya Ketrampilan yang dituntut dalam membaca dalam hati antara lain sebagai berikut 1 membaca tanpa bersuara tanpa bibir bergerak tanpa ada desis apapun 2 membaca tanpa ada gerakan-gerakan kepala 3 membaca lebih cepat dibandingkan dengan membaca nyaring 4 tanpa menggunakan jari atau alat lain sebagai penunjuk 5 mengerti dan memahami bahan bacaan 6 dituntut kecepatan mata dalam membaca 7 membaca dengan pemahaman yang baik

Beranda Soal UKG Agus Zainal M (SDN Gudang 2)

Republik_ilalangyahoocoid

8 dapat menyesuaikan kecepatan dengan tingkat kesukaran yang terdapat dalam bacaan 28 KD 131 Merumuskan hakikat (pengertiantujuanjenismanfaat) membaca

Hal berikut dilakukan seseorang ketika membaca survai kecuali a memeriksa judul bacaanbuku kata pengantar daftar isi dan malihat abstrak(jika ada) b memeriksa bagian terahkir dari isi (kesimpulan) jika ada c memeriksa indeks dan apendiks(jika ada) d membaca biografi pengarang

Uraian Secara garis besar membaca dalam hati dapat dibedakan menjadi dua (I) MEMBACA EKSTENSIF amp (II) MEMBACA

INTENSIF Berikut penjelasan secara rinci kedua jenis membaca tersebut I Membaca Ekstensif membaca ekstensif adalah membaca secara luas Objeknya meliputi sebanyak mungkin teks dalam waktu yang sesingkat-singkatnya Membaca ekstensif meliputi 1 Membaca Survai (Survey Reading) Membaca survai adalah kegiatan membaca untuk mengetahui secara sekilas terhadap bahan bacaan yang akan dibaca lebih mendalam Kegiatan membaca survai merupakan pendahuluan dalam membaca ekstensif Yang dilakukan seseorang ketika membaca survai adalah sebagai berikut (a) memeriksa judul bacaanbuku kata pengantar daftar isi dan malihat abstrak(jika ada) (b) memeriksa bagian terahkir dari isi (kesimpulan) jika ada (c) memeriksa indeks dan apendiks(jika ada)

2 Membaca Sekilas Membaca sekilas atau membaca cepat adalah kegiatan membaca dengan mengandalakan kecepatan gerak mata dalam melihat dan memperhatikan bahan tertulis yang dibacanya dengan tujuan untuk mendapatkan informasi secara cepat Metode yang digunakan dalam melatihkan membaca cepat adalah (a) metode kosakata metode yang berusaha untuk menambah kosakata (b) Metode motivasi metode yang berusaha memotivasi pembaca(pemula) yang mengalami hambatan (c) Metode gerak mata metode yang mengembangkan kecepatan membaca dengan menigkatkan kecepatan gerak mata Hambatan-hambatan yang dapat mengurangi kecepatan mambaca (a) vokalisai atau berguman ketika membaca (b) membaca dengan menggerakan bibir tetapi tidak bersuara (c) kepala bergerak searah tulisan yang dibaca (d) subvokalisasi suara yang biasa ikut membaca di dalam pikiran kita (e) jari tangan selalu menunjuk tulisa yang sedang kit abaca (f) gerakan mata kembali pada kata-kata sebelumnya 3 Membaca Dangkal (Superficial Reading) membaca dangkal pada hakekatnya bertujuan untuk memperoleh pemahaman yang dangkal yang bersifat luaran yang tidak mendalam dari suatu bahan bacaan Membaca jenis ini biasanya dilakukan seseorang membaca demi kesenangan membaca bacaan ringan yang mendatangkan kesenangan kegembiraan sebagai pengisi waktu senggang

29 KD 131 Merumuskan hakikat (pengertiantujuanjenismanfaat) membaca Membaca jenis ini biasanya dilakukan seseorang membaca demi kesenangan membaca bacaan ringan yang mendatangkan kesenangan kegembiraan sebagai pengisi waktu senggang Berdasarkan karakteristik diatas kegiatan tersebut termasuk ke dalam membaca jenis a Membaca Survai (Survey Reading) b Membaca Sekilas c Membaca Dangkal (Superficial Reading)

d Membaca Nyaring Uraian No 28 30 KD 131 Merumuskan hakikat (pengertiantujuanjenismanfaat) membaca

Berikut adalah tujuan umum dalam aktifitas membaca kecuali a Membaca untuk memperoleh perincian-perincian atau fakta-fakta (reading for details or facts) Membaca tersebut

bertujuan untuk menemukan atau mengetahui penemuan-penemuan telah dilakukan oleh sang tokoh untuk memecahkan masalah-masalah yang dibuat oleh sang tokoh

b Membaca untuk mengetahui ukuran atau susunan organisasi cerita (reading for sequenceor organization) Membaca tersebut bertujuan untuk mengetahui bagian-bagian cerita dan hubungan antar bagian-bagian cerita

c Membaca untuk menyimpulkan atau membaca inferensi (reading for inference) d Membaca untuk memperoleh kekurangan suatu buku (finding mistakes)

Uraian Tujuan Membaca

Beranda Soal UKG Agus Zainal M (SDN Gudang 2)

Republik_ilalangyahoocoid

Berdasarkan maksud tujuan atau keintensifan serta cara dalam membaca di bawah ini Anderson dalam Tarigan (19799-10) mengemukakan beberapa tujuan membaca antara lain a Membaca untuk memperoleh perincian-perincian atau fakta-fakta (reading for details or facts) Membaca tersebut bertujuan untuk menemukan atau mengetahui penemuan-penemuan telah dilakukan oleh sang tokoh untuk memecahkan masalah-masalah yang dibuat oleh sang tokoh b Membaca untuk memperoleh ide-ide utama (reading for main ideas) Membaca untuk mengetahui topik atau masalah dalam bacaan Untuk menemukan ide pokok bacaan dengan membaca halamn demi halaman c Membaca untuk mengetahui ukuran atau susunan organisasi cerita (reading for sequenceor organization) Membaca tersebut bertujuan untuk mengetahui bagian-bagian cerita dan hubungan antar bagian-bagian cerita d Membaca untuk menyimpulkan atau membaca inferensi (reading for inference) Pembaca diharapkan dapat merasakan sesuatu yang dirasakan penulis e Membaca untuk mengelompokkan atau mengklasifikasikan (reading for classify) Membaca jenis ini bertujuan untuk menemukan hal-hal yang tidak wajar mengenai sesuatu hal (Anderson dalam Tarigan 197910) f Membaca untuk menilai atau mengevaluasai (reading to evaluate) Jenis membaca tersebut bertujuan menemukan suatu keberhasilan berdasarkan ukuran-ukuran tertentu Membaca jenis ini memerlukan ketelitian dengan membandingkan dan mengujinya kembali g Membaca untuk memperbandingkan atau mempertentangkan (reading to compare or contrast) Tujuan membaca tersebut adalah untuk menemukan bagaimana cara perbedaan atau persamaan dua hal atau lebih

31 KD 133 menemukan pesan pokok utama sebuah berita Simak penggalan berita berikut Hebat Siswa Indonesia Pertahankan Tradisi Emas di Olimpiade Fisika Tim Olimpiade Fisika Indonesia REPUBLIKACOID JAKARTA--Luar biasa Lima siswa Indonesia yang dikirim ke ajang Olimpiade Fisika atau International Physics Olympiad (IPhO) ke-41 di Zagreb Kroasia 17-25 Juli berhasil menyabet empat medali emas dan satu perak Pelajar yang menyumbang emas adalah Muhammad Sohibul Maromi (SMAN 1 Pamekasan Madura) Christian George Emor (SMA Lokon St Nikolaus Tomohon Sulawesi Utara) David Giovanni (SMAK Penabur Gading Serpong Banten) dan Kevin Soedyatmiko (SMAN 12 Jakarta) Sedangkan medali perak berhasil diraih oleh Ahmad Ataka Awwalur Rizqi (SMAN 1 Yogyakarta) Prestasi ini jauh lebih baik dibanding ajang Olimpiade Fisika ke-40 di Merida Yucatan Meksiko 2009 yang lalu Saat itu delegasi siswa Indonesia merebut satu medali emas dua medali perak dan satu perunggu Hasil empat medali emas dan satu perak ini hampir menyamai prestasi terbaik sebelumnya pada ajang Olimpiade Fisika ke-37 di Singapura Saat itu siswa Indonesia tidak hanya berhasil menyabet 4 medali emas namun juga meraih predikat `Absolute Winner` atas nama Mailoa Jonathan Pradana (SMAK 1 BPK Penabur Jakarta) Tapi yang terpenting lagi pelajar Indonesia berhasil mempertahankan tradisi emas di setiap ajang Olimpiade Fisika Red Endro Yuwanto Peristiwa yang diberitakan adalah a siswa Indonesia pertahankan tradisi emas di Olimpiade Matematika b siswa Indonesia pertahankan tradisi emas di Olimpiade Fisika c siswa Indonesia menyumbang emas di Olimpiade Matematika

d siswa Indonesia menyumbang perak di Olimpiade Fisika Uraian 32 KD 133 menemukan pesan pokok utama sebuah berita

Simak penggalan berita berikut Hebat Siswa Indonesia Pertahankan Tradisi Emas di Olimpiade Fisika Tim Olimpiade Fisika Indonesia REPUBLIKACOID JAKARTA--Luar biasa Lima siswa Indonesia yang dikirim ke ajang Olimpiade Fisika atau International Physics Olympiad (IPhO) ke-41 di Zagreb Kroasia 17-25 Juli berhasil menyabet empat medali emas dan satu perak Pelajar yang menyumbang emas adalah Muhammad Sohibul Maromi (SMAN 1 Pamekasan Madura) Christian George Emor (SMA Lokon St Nikolaus Tomohon Sulawesi Utara) David Giovanni (SMAK Penabur Gading Serpong Banten) dan Kevin Soedyatmiko (SMAN 12 Jakarta) Sedangkan medali perak berhasil diraih oleh Ahmad Ataka Awwalur Rizqi (SMAN 1 Yogyakarta) Prestasi ini jauh lebih baik dibanding ajang Olimpiade Fisika ke-40 di Merida Yucatan Meksiko 2009 yang lalu Saat itu delegasi siswa Indonesia merebut satu medali emas dua medali perak dan satu perunggu Hasil empat medali emas dan satu perak ini hampir menyamai prestasi terbaik sebelumnya pada ajang Olimpiade Fisika ke-37 di Singapura Saat itu siswa Indonesia tidak hanya berhasil menyabet 4 medali emas namun juga meraih predikat `Absolute Winner` atas nama Mailoa Jonathan Pradana (SMAK 1 BPK Penabur Jakarta) Tapi yang terpenting lagi pelajar Indonesia berhasil mempertahankan tradisi emas di setiap ajang Olimpiade Fisika Red Endro Yuwanto Lima siswa Indonesia berhasil mendapatkan a 4 emas 1 perak

b 1 emas 4 perak c 3 emas 2 perak d 2 perak 3 emas

Beranda Soal UKG Agus Zainal M (SDN Gudang 2)

Republik_ilalangyahoocoid

Uraian 33 KD 134 menemukan pesan pokok dalam wacana naratif seperti cerita rakyat puisi

Surat dari ibu Jika bayang telah pudar Dan elang laut pulang ke sarang Angin bertiup ke benua Tiang ndash tiang akan kering sendiri Dan nahkoda sudah tau pedoman Boleh engkau datang padaku Makna lambang dari nahkoda sudah tahu pedoman adalahhellip a Sudah mencari pedoman hidup b Sudah menemukan arah dan tujuan

c Sudah berilmu dan berpengalaman d Sudah mempunyai pasangan hidup

Uraian 34 KD 135 membandingkan berbagai jenis wacana bahasa indonesia (deskripsi narasi)

Pendekatan untuk mendapat tanggapan emosional pembaca ataupun kesan pembaca adalah contoh Pendekatan Deskripsi jenis a Pendekatan Ekspositoris b Pendekatan Impresionistik

c Pendekatan menurut sikap pengarang d Pendekatan Realistik

Uraian PENDEKATAN DESKRIPSI

Pendekatan dalam pendeskripsian dapat dibedakan menjadi beberapa kategori pendekatan yaitu

1 Pendekatan Ekspositoris

Dalam pendekatan ini kita berusaha agar deskripsi yang kita buat dapat memberi keterangan sesuai dengan keadan yang

sebenarnya sehingga pembaca dapat seolah-olah ikut melihat atau merasakan objek yang kita deskripsikan Karangan

jenis ini berisi daftar detail sesuatu secara lengkap sehingga pembaca dan penalarannya dapat memperoleh kesan

keseluruhan tentang sesuatu

2 Pendekatan Impresionistik

Tujuan deskripsi impresionistik ialah untuk mendapatkan tanggapan emosional pembaca ataupun kesan pembaca Corak

deskripsi ini diantaranya juga ditentukan oleh macam kesan apa yang diinginkan penulisnya

3 Pendekatan menurut Sikap Pengarang

Pendekatan ini sangat bergantung pada tujuan yang ingin dicapai sifat objek serta pembaca deskripsinya Dalam

menguraikan sebuah gagasan penulis mungkin mengharapkan agar pembaca merasa tidak puas terhadapa suatu

tindakan atau keadaan atau penulis menginginkan agar pembaca juga harus merasakan bahwa persoalan yang dihadapi

merupakan masalah yang gawat Penulis juga dapat membayangkan bahwa akan terjadi sesuatu yang tidak diinginkan

sehingga pembaca dari semula sudah disiapkan perasaan yang kurang enak seram takut dan sebagainya (Alkhaidah

1997) 35 kd 135 membandingkan berbagai jenis wacana bahasa indonesia (deskripsi narasi)

Prinsip-Prinsip karangan Narasi sebagai berikut kecuali a Alur b Penokohan c Amanat

d Sudut Pandang Uraian Prinsip-prinsip dasar narasi sebagai tumpuan berfikir adalah sebagai berikut

1 Alur (plot) biasa disebut dengan jalan cerita

2 Penokohan (rangkaian perbuatanaksi) 3 Latar (setting) yaitu tempat dan atau waktu terjadinya perbuatan tokoh atau peristiwa yang dialami tokoh 4 Sudut Pandang (Point of View)

a Narrator Serba Tahu (Omniscient point of view) Yaitu narrator bertindak sebagai pencipta segalanya b Narrator bertindak objektif (Objektive point of view) Pengarang tak member komentar apa pun dan pembaca bebas menafsirkan apa yang diceritakan pengarang c Narrator ikut aktif ( Narator acting) Narrator terlibat dalam cerita d Narrator sebagai peninjau

Beranda Soal UKG Agus Zainal M (SDN Gudang 2)

Republik_ilalangyahoocoid

Pelakunya adalah orang ketiga yang biasa disebut ldquodiardquo 36 KD 135 membandingkan berbagai jenis wacana bahasa indonesia (deskripsi narasi)

1 Menentukan tema atau amanat apa yang akan disampaikan 2 Menetapkan sasaran pembaca 3 Merancang peristiwa-peristiwa utama yang akan ditampilkan dalam bentuk skema perkembangandan akhir cerita 5 Memerinci peristiwa-peristiwa utama ke dalam detail-detail peristiwa sebagai pendukung cerita 6 Menyusun tokoh dan perwatakanserta latar dan sudut pandang Karakteristik diatas adalah pengembangan dari paragraf a Narasi

b Deskprisi c Ekspositoris d Sudut Pandang Persuasif

Uraian Pengembangan karangan narasi dapat dilakukan dengan langkah-langkah berikut

(1) menentukan tema atau amanat apa yang akan disampaikan (2) menetapkan sasaran pembaca (3) merancang peristiwa-peristiwa utama yang akan ditampilkan dalam bentuk skema alur (4) membagi peristiwa utama ke dalam ke dalam bagian awal perkembangan dan akhir cerita (5) memerinci peristiwa-peristiwa utama ke dalam detail-detail peristiwa sebagai pendukung cerita dan (6) menyusun tokoh dan perwatakan latar dan sudut pandang

37 KD 135 membandingkan berbagai jenis wacana bahasa indonesia (deskripsi narasi) Karangan narasi yang berusaha untuk memberikan suatu maksud tertentu menyampaikan suatu amanat terselubung kepada para pembaca atau pendengar sehingga tampak seolah-olah melihat disebut a Narasi Informatif b Narasi Ekspositoris c Narasi Sugestif d Narasi Artistik

Uraian Jenis-Jenis Narasi

Narasi Informatif Narasi informatif adalah narasi yang memiliki sasaran penyampaian informasi secara tepat tentang suatu peristiwa

dengan tujuan memperluas pengetahuan orang tentang kisah seseorang Narasi Ekspositorik

Narasi ekspositorik adalah narasi yang memiliki sasaran penyampaian informasi secara tepat tentang suatu peristiwa dengan tujuan memperluas pengetahuan orang tentang kisah seseorang Dalam narasi ekspositorik penulis menceritakan suatu peristiwa berdasarkan data yang sebenarnya Pelaku yang ditonjolkan biasanya satu orang Pelaku diceritakan mulai dari kecil sampai saat ini atau sampai terakhir dalam kehidupannya Karangan narasi ini diwarnai oleh eksposisi maka ketentuan eksposisi juga berlaku pada penulisan narasi ekspositprik Ketentuan ini berkaitan dengan penggunaan bahasa yang logis berdasarkan fakta yang ada tidak memasukan unsursugestif atau bersifat objektif

Narasi Artistik

Narasi artistik adalah narasi yang berusaha untuk memberikan suatu maksud tertentu menyampaikan suatu amanat terselubung kepada para pembaca atau pendengar sehingga tampak seolah-olah melihat Ketentuan ini berkaitan dengan penggunaan bahasa yang logis berdasarkan fakta yang ada tidak memasukan unsur sugestif atau bersifat objektif

Narasi Sugestif Narasi sugestif adalah narasi yang berusaha untuk memberikan suatu maksud tertentu menyampaikan suatu amanat

terselubung kepada para pembaca atau pendengar sehingga tampak seolah-olah melihat 38 KD 135 membandingkan berbagai jenis wacana bahasa indonesia (deskripsi narasi)

Ciri karangan Narasi yang benar kecuali a Menonjolkan unsur perbuatan atau tindakan Membuat pembaca atau pendengar merasakan sendiri atau

mengalami sendiri

b Ada konfiks menjawab pertanyaan apa yang terjadi c Dirangkai dalam urutan waktu menggambarkan dengan jelas suatu peristiwa d Berisi ajakan dirangkai dalam urutan waktu

Uraian Ciri-Ciri Karangan Narasi Menurut Gorys Keraf (2000136)

Menonjolkan unsur perbuatan atau tindakan Dirangkai dalam urutan waktu Berusaha menjawab pertanyaan apa yang terjadi Ada konfiks

39 kd 135 membandingkan berbagai jenis wacana bahasa indonesia (deskripsi narasi) Langkah menyusun paragraf deskripsi yang benar adalah (1)Tentukan objek atau tema yang akan dideskripsikan (2)Mengumpulkan data dengan mengamati objek yang akan dideskripsikan (3)Menyusun data tersebut ke dalam urutan yang baik (menyusun kerangka karangan) (4)Menguraikan kerangka karangan menjadi dekripsi yang sesuai dengan tema yang ditentukan

Beranda Soal UKG Agus Zainal M (SDN Gudang 2)

Republik_ilalangyahoocoid

(5)Tentukan tujuan a 1-5-2-3-4

b 1-5-4-3-2 c 1-5-3-2-4 d 1-3-4-2-5

Uraian Langkah menyusun deskripsi

1Tentukan objek atau tema yang akan dideskripsikan 2Tentukan tujuan 3Mengumpulkan data dengan mengamati objek yang akan dideskripsikan 4Menyusun data tersebut ke dalam urutan yang baik (menyusun kerangka karangan) 5Menguraikan kerangka karangan menjadi dekripsi yang sesuai dengan tema yang ditentukan

40 Kd 311 Merancang aktivitas pembelajaran berdasarkan prinsip dan teori pembelajaran matematika Dalam tahap ini penyajian yang dilakukan melalui tindakan anak secara langsung terlihat dalam memanipulasi (mengotak atik)objek Proses tersebut menurut Bruner dikategorikan ke dalam model a Tahap Enaktif

b Tahap Ikonik c Tahap Simbolik d Tahap Implikatif

Uraian 1 Model Tahap Enaktif

Dalam tahap ini penyajian yang dilakukan melalui tindakan anak secara langsung terlihat dalam memanipulasi (mengotak atik)objek Contoh Budi mempunyai 2 pensil kemudian ibunya memberikannya lagi 3 pinsil Berapa banyak pensil Budi sekarang

2 Model Tahap Ikonik Dalam tahap ini kegiatan penyajian dilakukan berdasarkan pada pikiran internal dimana pengetahuan disajikan melalui serangkaian gambar-gambar atau grafik yang dilakukan anak berhubungan dengan mental yang merupakan gambaran dari objek-objek yang dimanipulasinya Contoh + = hellip

3 Model Tahap Simbolis Dalam tahap ini bahasa adalah pola dasar simbolik anak memanipulasi Simbol-simbol atau lambang-lambang objek tertentu Contoh 2 pensil + 3 pensil = hellip pensil

41 Kd 311 Merancang aktivitas pembelajaran berdasarkan prinsip dan teori pembelajaran matematika Dalam tahap ini kegiatan penyajian dilakukan berdasarkan pada pikiran internal dimana pengetahuan disajikan melalui serangkaian gambar-gambar atau grafik yang dilakukan anak Proses tersebut menurut Bruner dikategorikan ke dalam model a Tahap Enaktif b Tahap Ikonik

c Tahap Simbolik d Tahap Implikatif

Uraian No 40 42 Kd 311 Merancang aktivitas pembelajaran berdasarkan prinsip dan teori pembelajaran matematika

Objek belajar matematika dibagi kedalam Objek Langsung dan Objek Tak Langsung Hal tersebut adalah teori belajar Matematika menurut a Robert M Gagne

b Jerome S Burner c Thorndike d Skinner

Uraian Teori yang diperkenalkan Robert MGagne pada tahun 1960-an pembelajaran harus dikondisikan untuk memunculkan

respons yang diharapkanMenurut Gagne (dalam Ismail 1998) belajar matematika terdiri dari objek langsung dan objek tak langsung

43 Kd 311 Merancang aktivitas pembelajaran berdasarkan prinsip dan teori pembelajaran matematika Perhatikan contoh berikut Budi mempunyai 2 pinsil kemudian ibunya memberikannya lagi 3 pinsilBerapa banyak pinsil Budi sekarang Hal tersebut dikemukakan Bruner dalam Proses Pembelajaran Matematika dalam tahap a Simbolik b Ikonik c Implikatif d Enaktif

Uraian No 40

Beranda Soal UKG Agus Zainal M (SDN Gudang 2)

Republik_ilalangyahoocoid

44 Kd 311 Merancang aktivitas pembelajaran berdasarkan prinsip dan teori pembelajaran matematika Perhatikan gambar disamping Ilustrasi di samping dikemukakan Bruner dalam Proses Pembelajaran Matematika dalam tahap

a Simbolik b Ikonik

c Implikatif d Enaktif

Uraian No 40 45 Kd 311 Merancang aktivitas pembelajaran berdasarkan prinsip dan teori pembelajaran matematika

Perhatikan contoh berikut Contoh 2 pinsil + 3 pinsil = hellippinsil Contoh tersebut dikemukakan Bruner dalam Proses Pembelajaran Matematika dalam tahap a Simbolik

b Ikonik c Implikatif d Enaktif

Uraian No 40 46 Kd 311 Merancang aktivitas pembelajaran berdasarkan prinsip dan teori pembelajaran matematika

Perhatikan contoh berikut penjumlahan bilangan positif dan negatif siswa mencoba sendiri dengan menggunakan garis bilangan Contoh tersebut dikemukakan Bruner dalam Teorema Pembelajaran Matematika a Penyusunan

b Notasi c Pengkontrasan dan Keanekaragaman d Pengaitan

Uraian Berdasarkan hasil pengamatannya Brunner merumuskan 5 teorema dalam pembelajaran matematika yaitu

1) Teorema Penyusunan Menerangkan bahwa cara yang terbaik memulai belajar suatu konsep matematika dalil defenisi dan semacamnya adalah dengan cara menyusun penyajiannya Misalnya dalam mempelajari penjumlahan bilangan positif dan negatif siswa mencoba sendiri dengan menggunakan garis bilangan 2) Teorema Notasi Menerangkan bahwa dalam pengajaran suatu konsep penggunaan notasi-notasi matematika harus diberikan secara bertahap dari yang sederhana ke yang lebih kompleks 3) Teorema Pengkontrasan dan Keanekaragaman Menerangkan bahwa pengontrasan dan keanekaragaman sangat penting dalam melakukan pengubahan konsep matematika dari yang konkrit ke yang lebih abstrak Dalam hal ini diperlukan banyak contoh Contoh yang diberikan harus sesuai dengan rumusan yang diberikan Misalnya menjelaskan persegi panjang disertai juga kemungkinan jajaran genjang dan segi empat lainnya selain persegi panjnag Dengan demikian siswa dapat membedakan apakah segi empat yang diberikan padanya termasuk persegi panjang atau tidak 4) Teorema Pengaitan Menerangkan bahwa dalam matematika terdapat hubungan yang berkaitan antara satu konsep dengan konsep yang lain Di mana materi yang satu merupakan prasyarat yang harus diketahui untuk mempelajari materi yang lain

47 Kd 311 Merancang aktivitas pembelajaran berdasarkan prinsip dan teori pembelajaran matematika Perhatikan contoh berikut Guru menjelaskan persegi panjang disertai juga kemungkinan jajaran genjang dan segi empat lainnya selain persegi panjnag Dengan demikian siswa dapat membedakan apakah segi empat yang diberikan padanya termasuk persegi panjang atau tidak Contoh tersebut dikemukakan Bruner dalam Teorema Pembelajaran Matematika a Penyusunan b Notasi c Pengkontrasan dan Keanekaragaman

d Pengaitan Uraian No 46 48 Kd 311 Merancang aktivitas pembelajaran berdasarkan prinsip dan teori pembelajaran matematika

Tahap pembelajaran Matematika menurut Van Halle adalah sebagai berikut kecuali a Tahap Pengenalan b Tahap Pengurutan c Tahap Analisis d Tahap Penyimpulan

Beranda Soal UKG Agus Zainal M (SDN Gudang 2)

Republik_ilalangyahoocoid

Uraian Van Hiele menyatakan bahwa terdapat 5 tahap belajar siswa dalam belajar geometri yaitu

a Tahap Pengenalan Pada tahap ini siswa mulai belajar mengenal suatu bangun geometri secara keseluruhan namun belum mampu mengetahui adanya sifat-sifat dari bangun geometri yang dilihatnya b Tahap Analisis Pada tahap ini siswa sudah mulai mengenal sifat-sifat yang dimiliki bangun geometri yang diamatinya c Tahap Pengurutan Pada tahap ini siswa sudah mengenal dan memahami sifat-sifat suatu bangun geometri serta sudah dapat mengurutkan bangun-bangun geometri yang satu sama yang lainnya saling berhubungan d Tahap Deduksi Pada tahap ini siswa telah mampu menarik kesimpulan secara deduktif yaitu menarik kesimpulan yang bersifat umum dan menuju ke hal yang bersifat khusus serta dapat mengambil kesimpulan e Tahap Akurasi Pada tahap ini siswa mulai menyadari pentingnya ketepatan prinsip-prinsip dasar yang melandasi suatu pembuktian Tahap berfikir ini merupakan tahap berfikir yang paling tinggi rumit dan kompleks karena di luar jangkauan usia anak-anak SD sampai tingakat SMP

49 Kd 311 Merancang aktivitas pembelajaran berdasarkan prinsip dan teori pembelajaran matematika middot Matematika untuk tujuan pembelajaran dianalisis sebagai kumpulan fakta yang berdiri sendiri dan tidak saling berkaitan middot Anak diharuskan menguasai unsur-unsur yang banyak sekali tanpa diperhatikan pengertiannya middot Anak mempelajari unsur-unsur dalam bentuk seperti yang akan digunakan nanti dalam kesempatan lain middot Anak akan mencapai tujuan ini secara efektif dan efisien dengan melalui pengulangan Teori Pembelajaran Matematika tersebut dikemukakan oleh a Skinner b Piaget c Van Brownell

d Thorndike Uraian Brownell mengemukakan tentang Teori Makna (Meaning Theory) sebagai pengganti Teori Latihan HafalUlangan (Drill

Theory) Intisari dari teori Drill adalah - Matematika untuk tujuan pembelajaran dianalisis sebagai kumpulan fakta yang berdiri sendiri dan tidak saling berkaitan - Anak diharuskan menguasai unsur-unsur yang banyak sekali tanpa diperhatikan pengertiannya - Anak mempelajari unsur-unsur dalam bentuk seperti yang akan digunakan nanti dalam kesempatan lain - Anak akan mencapai tujuan ini secara efektif dan efisien dengan melalui pengulangan

50 Kd 311 Merancang aktivitas pembelajaran berdasarkan prinsip dan teori pembelajaran matematika Tahap operasionaloperasi konkrit adalah teori Pembelajaran Matematika yang dikemukakan Peaget pada tahap usia a dibawah 2 tahun b 2-7 Tahun c 7-8 Tahun d 7-12 Tahun

Uraian Piaget membagi skema yang digunakan anak untuk memahami dunianya melalui empat periode utama yang berkorelasi

dengan dan semakin canggih seiring pertambahan usia Periode sensorimotor (usia 0ndash2 tahun) Periode praoperasional (usia 2ndash7 tahun) Periode operasional konkrit (usia 7ndash12 tahun) Periode operasional formal (usia 12 tahun sampai dewasa)

51 313 Memilih media pembelajaran yang tepat untuk pembelajaran operasi bilangan bulat Untuk operasi bilangan bulat perkalian media yang paling tepat digunakan adalah a sapu lidi

b kerikil c daun pakis d koin

Uraian 52 313 Memilih media pembelajaran yang tepat untuk pembelajaran operasi bilangan bulat

Untuk operasi bilangan bulat negatif kita bisa menggunakan media a uang kertas b kerikil c mistar d koin

Uraian 53 313 Memilih media pembelajaran yang tepat untuk pembelajaran operasi bilangan bulat

Beranda Soal UKG Agus Zainal M (SDN Gudang 2)

Republik_ilalangyahoocoid

Untuk operasi bilangan bulat penjumlahan media yang paling tepat digunakan adalah a uang kertas b kerikil c daun pakis d koin

Uraian 54 314 Memilih media pembelajaran yang tepat untuk pembelajaran operasi bilangan pecahan

Untuk operasi bilangan pecahan kita bisa menggunakan media berikut kecuali a kartu bilangan b garis bilangan c gambar bidang d blok pecahan

Uraian 55 316 Memilih media pembelajaran yang tepat untuk pembelajaran geometri dan pengukuran

Untuk menghitung luas dan keliling bangun datar media yang paling tepat digunakan adalah a Penggaris b Kertas Folio Bergaris

c Neraca d Kalkulator

Uraian

Page 3: 50 SOAL UKG & PEMBAHASAN SESUAI KISI-KISIS.pdf

Beranda Soal UKG Agus Zainal M (SDN Gudang 2)

Republik_ilalangyahoocoid

d Metode Cerita Uraian 11 KD 112 Aspek Membaca di kelas rendah

Kelebihan membaca permulaan adalah sebagai berikut a Mempunyai nilai strategis bagi pengembangan kepribadian dan kemampuan siswa

b Meningkatkan nilai siswa dalam pelajaran Bahasa Indonesia c Meningkatkan kemandirian siswa dalam membaca d Mempermudah menghafal kata-kata

Uraian Kelebihan membaca permulaan

- Dapat memperoleh informasi secara tepat dan lengkap - Mempunyai nilai strategis bagi pengembangan kepribadian dan kemampuan siswa - Membaca permulaan juga dapat mengembangkan nilai-nilai moral kemampuan bernalar dan kreatifitas seseorang Kekurangan membaca permulaan - Kemampuan membaca pada umumnya lebih rendah - Menghambat kemampuan untuk memahami kalimat atau cerita - Mengalami kesulitan menjawab mengenai isi cerita karena kesibukan siswa mengeja dan menyuarakan huruf-huruf - Kecepata membaca dan pemahaman siswa sangat rendah

12 KD 112 Aspek Membaca di kelas rendah Metode yang merupakan penyempurnaan metode alphabet dengan mengajarkan bunyi-bunyi bahasa sebagai pengganti huruf-huruf berdasarkan ucapan hurufnya adalah metode membaca permulaan a Metode Suku Kata b Metode Kata-kata c Metode Cerita d Metode Suara

Uraian Teknik Membaca Permulaan

1) Metode Alfabet Metode ini sering disebut metode harfiah Metode ldquoletter by letter methodrdquo atau ldquoABC methodrdquo Mula-mula dikenalkan abjad dari A sampai Z Setelah hafal beberapa huruf barulah huruf-huruf tersebut dirangkai menjadi suku kata Suku kata-suku kata tersebut setelah dikenalkan barulah dirangkai menjadi kata Dengan modal beberapa kata ituterciptalah kalimat 2) Metode Suara Metode suara juga disebut rdquoPhonic Methodrdquo Metode ini merupakan penyempurnaan metode alphabet Pada metode ini bukan abjadnya yang di ajarkan melainkan bunyi-bunyi bahasa sebagai pengganti huruf-huruf tersebutUcapan huruf-huruf tidak berdasarkan atas bunyi abjadnya melainkan ucapan hurufnya 3) Metode Suku Kata Metode ini biasa juga disebut rdquoSyllabic Methodrdquo Dalam metode ini suku kata merupakan kunci pokok dalam membuat kata Metode suku kata sebenarnya suatu metode pengajaran membaca permulaan yang di dasarkan atas kata-kata dianalisis menjadi suku kata-suku kataSuku kata0suku kata tersebut di intesiskan kembali menjadi kata-kata 4) Metode Kata-kata Pelaksanaan metode ini selalu di awali dengan kata-kata tertulis da ndash da gi ndash gi ku ndash ku dan lain-lain Setelah kata-kata itu dikenalkan kata-kata tersebut dianalisis atas suku katanya Selanjitnya suku kata itu di analisis lagi atas huruf-hurufnya Barulah huruf-huruf itu digabungkan kembali dalam bentuk kata semula 5) Metode Cerita Pelaksanaan metode cerita dalam mengajarkan membaca permulaan diawali dengan menghafalkan cerita atau sebuah puisi Cerita atau puisi itu diuraikan atas kalimat-kalimtnya sampai pada kata-katanya Dalam mengucapkan kata0kata metode ini menggunakan kata-kata fonetik 6) Metode Struktural Analitik Sintetik (SAS) Adapun contoh pelaksanaan metode SAS sebagai berikut Mula-mula diberikan kalimat secara keseluruhan Kalimat itu diuraikan atas kata-kata yang mendukungnya Dari kata-kata itu kita ceraikan atas suku-suku katanya dan akhirnya atas huruf-hurufnya Kemidian huruf-huruf itu kita sintetiskan kembali menjadi suku kata suku kata menjadi kata dan kata menjadi kalimat

13 KD 113 Aspek Menulis di kelas tinggi Pendekatan menekankan keterpaduan empat aspek keterampilan berbahasa (menyimak berbicara membaca dan menulis) dalam pembelajaran adalah salah satu Pendekatan yang disarankan dalam pembelajaran menulis yang disebut a pendekatan komunikatif b Pendekatan integratif

c Pendekatan keterampilan proses d Pendekatan tematis

Uraian Pendekatan yang disarankan dalam pembelajaran menulis

1) Pendekatan komunikatif memfokuskan pada keterampilan siswa mengimplementasikan fungsi bahasa (untuk berkomunikasi) dalam pembelajaran

Beranda Soal UKG Agus Zainal M (SDN Gudang 2)

Republik_ilalangyahoocoid

2) Pendekatan integratif menekankan keterpaduan empat aspek keterampilan berbahasa (menyimak berbicara membaca dan menulis) dalam pembelajaran

3) Pendekatan keterampilan proses memfokuskan keterampilan siswa dalam mengamati mengklasifikasi menginterpretasi dan mengkomunikasikan

4) Pendekatan tematis menekankan tema pembelajaran sebagai payungpemandu dalam pembelajaran 14 KD 113 Aspek Menulis di kelas tinggi

Teknik menulis cerita terdiri atas hal-hal sebagai berikut kecuali a menjawab pertanyaan b membuat kalimat c subtitusi d persuasi

Uraian Teknik menyusun cerita dapat dilakukan dengan menjawab pertanyaan melengkapi kalimat memperbaiki susunan kalimat

memperluas kalimat subtitusi transfomtasi dan membuat kalimat Persuasi adalah bujukan halus

15 KD 113 Aspek Menulis di kelas tinggi Model pembelajaran menulis ceritacerpen di SD meliputi hal-hal berikut kecuali a Menceritakan gambar b Melanjutkan cerita c Menceritakan pengalaman d Mendeskripsikan cerita

Uraian Model pembelajaran menulis ceritacerpen di SD meliputi menceritakan gambar melanjutkan ceria lain menceitakan

mimpi menceritakan pengalaman dan menceritakan cita-cita 16 KD 121 Memilih berbagai Metode Menulis Permulaan

1) Menulis huruf lepas 2) Merangkaikan huruf lepas menjadi suku kata 3) Merangkaikan suku kata menjadi kata 4) Menyusun kata menjadi kalimat (Djauzak 19964) Tahapan diatas adalah Teknik Menulis Permulaan dengan metode a Metode Eja

b Metode Kata Lembaga c Metode SAS d Metode Global

Uraian Metode dan pembelajaran menulis permulaan

a Metode Eja Metode eja di dasarkan pada pendekatan harfiah artinya belajar membaca dan menulis dimulai dari huruf-huruf yang dirangkaikan menjadi suku kata Oleh karena itu pengajaran dimulai dari pengenalan huruf-huruf Demikian halnya dengan pengajaran menulis di mulai dari huruf lepas dengan langka-langkah sebagai berikut 1) Menulis huruf lepas 2) Merangkaikan huruf lepas menjadi suku kata 3) Merangkaikan suku kata menjadi kata 4) Menyusun kata menjadi kalimat (Djauzak 19964) b Metode kata lembaga Metode kata lembaga di mulai mengajar dengan langkah-langkah sebagai berikut 1) Mengenalkan kata 2) Merangkaikan kata antar suku kata 3) Menguraikan suku kata atas huruf-hurufnya 4) Menggabungkan huruf menjadi kata (Djauzak 19965) c Metode Global Metode global memulai pengajaran membaca dan menulis permulaan dengan membaca kalimat secara utuh yang ada di bawah gambar Menguraikan kalimat dengan kata-kata menguraikan kata-kata menjadi suku kata (Djauzak 19966) d Metode SAS Menuryut (Supriyadi 1996 334-335) pengertian metode SAS adalah suatu pendekatan cerita di sertai dengan gambar yang didalamnya terkandung unsur analitik sintetik Metode SAS menurut (Djuzak19968) adalah suatu pembelajaran menulis permulaan yang didasarkan atas pendekatan cerita yakni cara memulai mengajar menulis dengan menampil cerita yang diambil dari dialog siswa dan guru atau siswa dengan siswa Teknik pelaksanaan pembelajaran metode SAS yakni keterampilan menulis kartu huruf kartu suku kata kartu kata dan kartu kalimat sementara sebagian siswa mencari huruf suku kata dan kata guru dan sebagian siswa menempel kata-kata yang tersusun sehingga menjadi kalimat yang berarti (Subana) Proses operasional metode SAS mempunyai langkah-lagkah dengan urutan sebagai berikut a Struktur yaitu menampilkan keseluruhan b Analitik yatu melakukan proses penguraian c Sintetik yaitu melakukan penggalan pada struktur semula

Beranda Soal UKG Agus Zainal M (SDN Gudang 2)

Republik_ilalangyahoocoid

17 KD 121 Memilih berbagai Metode Menulis Permulaan 1) Mengenalkan kata 2) Merangkaikan kata antar suku kata 3) Menguraikan suku kata atas huruf-hurufnya 4) Menggabungkan huruf menjadi kata (Djauzak 19965) Tahapan diatas adalah Teknik Menulis Permulaan dengan metode a Metode Eja b Metode Kata Lembaga

c Metode SAS d Metode Global

Uraian No 16 18 KD 121 Memilih berbagai Metode Menulis Permulaan

Metode dengan memulai pengajaran membaca dan menulis permulaan dengan membaca kalimat secara utuh yang ada di bawah gambar Menguraikan kalimat dengan kata-kata menguraikan kata-kata menjadi suku kata disebut dengan metode a Metode Eja b Metode Kata Lembaga c Metode SAS d Metode Global

Uraian No 16 19 KD 121 Memilih berbagai Metode Menulis Permulaan

Suatu pembelajaran menulis permulaan yang didasarkan atas pendekatan cerita yakni cara memulai mengajar menulis dengan menampil cerita yang diambil dari dialog siswa dan guru atau siswa dengan siswa disebut dengan metode a Metode Eja b Metode Kata Lembaga c Metode SAS

d Metode Global Uraian No 16 20 KD 121 Memilih berbagai Metode Menulis Permulaan

menulis kartu huruf kartu suku kata kartu kata dan kartu kalimat sementara sebagian siswa mencari huruf suku kata dan kata guru dan sebagian siswa menempel kata-kata yang tersusun sehingga menjadi kalimat yang berarti adalah contoh metode a Metode Eja b Metode Kata Lembaga c Metode SAS

d Metode Global Uraian No 16 21 KD 122 Merancang berbagai kegiatan menulis di kelas tinggi

Berikut adalah kegiatan menulis lanjutan di kelas tinggi kecuali a menulis tentang berbagai topik b menulis pengumuman c menulis pantun d menulis memo

Uraian 22 KD 123 Perencanaan dan Pelaksanaan evaluasi pembelajaran Bahasa dan Sastra Indonesia

Perencanaan Pengajaran meliputi hal-hal berikut kecuali a tujuan apa yang hendak dicapai b memilih bahan ajar c proses belajar mengajar d alat penilaian

Uraian Menurut Akhlan dan Rahman (199715) perencanaan pengajaran meliputi

a tujuan apa yang hendak dicapai b bahan pengajaran c proses belajar mengajardan d alat penilaian

23 KD 123 Perencanaan dan Pelaksanaan evaluasi pembelajaran Bahasa dan Sastra Indonesia karakteristik perencanaan pengajaran yang baik hendaknya mengandung prinsip sebagai berikut a Memiliki sikap objektif rasio (tepat dan masuk akal) komprehensif dan sistematis (menyeluruh dan tersusun rapi)

b Merupakan suatu wahana atau wadah untuk mengembangkan segala potensi yang ada dan dimiliki oleh anak

Beranda Soal UKG Agus Zainal M (SDN Gudang 2)

Republik_ilalangyahoocoid

didik c Mengendalikan kekuatan sendiri bukan didasarkan atas kekuatan orang lain d Melakukan studi kasus yang berkesinambungan

Uraian Menurut Akhlan dan Rahman (19977) karakteristik perencanaan pengajaran yang baik hendaknya mengandung prinsip

sebagai berikut a Mengembangkan hubungan interaksi yang baik di antara sesama manusia dalam hal ini siswa dan guru serta personal

terkait b Merupakan suatu wahana atau wadah untuk mengembangkan segala potensi yang ada dan dimiliki oleh anak didik c Memiliki sikap objektif rasio (tepat dan masuk akal) komprehensif dan sistematis (menyeluruh dan tersusun rapi) d Mengendalikan kekuatan sendiri bukan didasarkan atas kekuatan orang lain Didukung oleh fakta dan data yang

menunjang pencapaian tujuan yang telah di dirumuskan e Fleksibel dan dinamis artinya mudah disesuaikan dengan keadaan serta perkembangan ke arah yang lebih baik dan

maju 24 KD 123 Perencanaan dan Pelaksanaan evaluasi pembelajaran Bahasa dan Sastra Indonesia

tahap mengumpulkan informasi tentang keadaan objek evaluasi (siswa) dengan menggunakan teknik tes atau nontes disebut tahapan a Tahap Tindak Lanjut b Tahap Persiapan c Tahap Pelaksanaan

d Tahap Pengolahan Hasil Uraian Berikut ini penjelasan singkat tentang keempat tahap evaluasi pembelajaran tersebut

(1) Tahap Persiapan Menurut Damaianti (2007 8) tahap ini disebut juga tahap perencanaan dan perumusan kriterium Langkahnya meliputi (a) perumusan tujuan evaluasi (b) penetapan aspek-aspek yang akan dievaluasi (c) menetapkan metode dan bentuk evaluasi (tesnontes) (d) merencanakan waktu evaluasi (e) melakukan uji coba (untuk tes) agar dapat mengukur validitas dan reliabilitasnya Untuk evaluasi yang menggunakan tes hasil dari tahap ini adalah kisi-kisi soal dan seperangkat alat tes soal lembar jawaban (untuk tes tulis) kunci jawaban dan pedoman penilaian 2) Tahap Pelaksanaan Tahap pelaksanaan atau disebut juga dengan tahap pengukuran dan pengumpulan data adalah tahap untuk mengumpulkan informasi tentang keadaan objek evaluasi (siswa) dengan menggunakan teknik tes atau nontes Bila menggunakan teknik tes soal yang igunakan sebaiknya sudah teruji validitas dan reliabilitasnya Tes yang digunakan dapat berbentuk tes tulis lisan atau praktik 3) Tahap Pengolahan Hasil Tahap pengolahan hasil adalah tahap pemeriksaan hasil evaluasi dengan memberikan skor Skor yang diperoleh siswa selanjutnya diubah menjadi nilai Pada tes tulis pemeriksaan hasil dilakukan setelah tes selesai sedangkan pada tes lisan dan praktik pemberian nilai dilakukan bersamaan dengan waktu pelasanaan tes tersebut 4) Tahap Tindak Lanjut Tahap tindak lanjut atau disebut juga tahap penafsiran adalah tahap untuk mengambil keputusan berdasarkan nilai yang dihasilkan pada tahap pengolahan hasil misalnya a memperbaiki proses belajar mengajar b memperbaiki kesulitan belajar siswa c memperbaiki alat evaluasi d membuat laporan evaluasi (rapor)

25 KD 131 Merumuskan hakikat (pengertiantujuanjenismanfaat) membaca membaca yang mengutamakan isi bacaan sebagai ungkapan pikiran perasaan dan kehendak penulis Bila hanya ingin mengetahui isinya membaca cerdas bersifat lugas Akan tetapi bila maksudnya untuk memahami dan memilki isi bacaan maka tergolong kedalam membaca jenis a Membaca cerdas atau membaca dalam hati

b Membaca bahasa c Membaca teknis d Membaca bebas

Uraian Menurut Soedjono dalam Sue (200418-21) ada lima macam membaca yaitu membaca bahasa membaca cerdas atau

membaca dalam hati membaca teknis membaca emosional dan membaca bebas 1) Membaca bahasa Membaca bahasa adalah membaca yang mengutamakan bahasa bacaan Membaca bahasa mementingkan segi bahasa bacaan Hal-hal yang perlu diperhatikan dalam membaca bahasa adalah kesesuian pikir dengan bahasa perbendaharaan bahasa yang meliputi kosa kata struktur kalimat dan ejaan 2) Membaca cerdas atau membaca dalam hati

Beranda Soal UKG Agus Zainal M (SDN Gudang 2)

Republik_ilalangyahoocoid

Membaca cerdas adalah membaca yang mengutamakan isi bacaan sebagai ungkapan pikiran perasaan dan kehendak penulis Bila hanya ingin mengetahui isinya membaca cerdas bersifat lugas Akan tetapi bial maksudnya untuk memahami dan memilki isi bacaan maka disebut membaca belajar 3) Membaca teknis Membaca teknis adalah membaca dengan mengarahkan bacaan secara wajar Wajar maksudnya sesuai ucapan tekanan dan intonasinya Pikiran perasaan dan kemauan yang tersimpan dalam bacaan dapat diaktualisasikan dengan baik 4) Membaca emosional Membaca emosional adalah membaca sebagai sarana untuk memasuki perasaan yaitu keindahan isi dan keindahan bahasanya 5) Membaca bebas Membaca bebas adalah membaca sesuatu atas kehendak sendiri tanpa adanya unsur paksaan dari luar Unsur dari luar misalnya guru orang tua teman atau pihak-pihak lain

26 KD 131 Merumuskan hakikat (pengertiantujuanjenismanfaat) membaca membaca sesuatu atas kehendak sendiri tanpa adanya unsur paksaan dari luar Unsur dari luar misalnya guru orang tua teman atau pihak-pihak lain termasuk jenis membaca a Membaca cerdas atau membaca dalam hati b Membaca bahasa c Membaca teknis d Membaca bebas

Uraian No 25 27 KD 131 Merumuskan hakikat (pengertiantujuanjenismanfaat) membaca

1 menggunakan ucapan yang tepat 2 menggunakan frase yang tepat 3 menggunakan intonasi suara yang wajar 4 dalam posisi sikap yang baik 5 menguasai tanda-tanda baca 6 membaca dengan terang dan jelas keterampilan di atas harus dipunyai dalam jenis membaca a Membaca cerdas atau membaca dalam hati b Membaca bahasa c Membaca teknis d Membaca nyaring

Uraian JENIS-JENIS MEMBACA DAN KARAKTERISTIKNYA

Ditinjau dari segi terdengar atau tidaknya suara pembaca waktu melakukan kegiatan membaca maka proses membaca dapat dibedakan menjadi A Membaca Nyaring Membaca nyaring adalah kegiatan membaca dengan menyuarakan tulisan yang dibacanya dengan ucapan dan intonasi yang tepat agar pendengar dan pembaca dapat menangkap informasi yang disampaikan oleh penulis baik yang berupa pikiran perasaan sikap ataupun pengalaman penulis Ketrampilan yang dituntut dalam membaca nyaring adalah berbagai kemampuan diantaranya adalah 1 menggunakan ucapan yang tepat 2 menggunakan frase yang tepat 3 menggunakan intonasi suara yang wajar 4 dalam posisi sikap yang baik 5 menguasai tanda-tanda baca 6 membaca dengan terang dan jelas 7 membaca dengan penuh perasaan ekspresif 8 membaca dengan tidak terbata-bata 9 mengerti serta memahami bahan bacaan yang dibacanya 10 kecepatan bergantung pada bahan bacaan yang dibacanya 11 membaca dengan tanpa terus-menerus melihat bahan bacaan 12 membaca dengan penuh kepercayaan pada diri sendiri B Membaca Dalam Hati Membaca dalam hati adalah kegiatan membaca yang dilakukan dengan tanpa menyuarakan isi bacaan yang dibacanya Ketrampilan yang dituntut dalam membaca dalam hati antara lain sebagai berikut 1 membaca tanpa bersuara tanpa bibir bergerak tanpa ada desis apapun 2 membaca tanpa ada gerakan-gerakan kepala 3 membaca lebih cepat dibandingkan dengan membaca nyaring 4 tanpa menggunakan jari atau alat lain sebagai penunjuk 5 mengerti dan memahami bahan bacaan 6 dituntut kecepatan mata dalam membaca 7 membaca dengan pemahaman yang baik

Beranda Soal UKG Agus Zainal M (SDN Gudang 2)

Republik_ilalangyahoocoid

8 dapat menyesuaikan kecepatan dengan tingkat kesukaran yang terdapat dalam bacaan 28 KD 131 Merumuskan hakikat (pengertiantujuanjenismanfaat) membaca

Hal berikut dilakukan seseorang ketika membaca survai kecuali a memeriksa judul bacaanbuku kata pengantar daftar isi dan malihat abstrak(jika ada) b memeriksa bagian terahkir dari isi (kesimpulan) jika ada c memeriksa indeks dan apendiks(jika ada) d membaca biografi pengarang

Uraian Secara garis besar membaca dalam hati dapat dibedakan menjadi dua (I) MEMBACA EKSTENSIF amp (II) MEMBACA

INTENSIF Berikut penjelasan secara rinci kedua jenis membaca tersebut I Membaca Ekstensif membaca ekstensif adalah membaca secara luas Objeknya meliputi sebanyak mungkin teks dalam waktu yang sesingkat-singkatnya Membaca ekstensif meliputi 1 Membaca Survai (Survey Reading) Membaca survai adalah kegiatan membaca untuk mengetahui secara sekilas terhadap bahan bacaan yang akan dibaca lebih mendalam Kegiatan membaca survai merupakan pendahuluan dalam membaca ekstensif Yang dilakukan seseorang ketika membaca survai adalah sebagai berikut (a) memeriksa judul bacaanbuku kata pengantar daftar isi dan malihat abstrak(jika ada) (b) memeriksa bagian terahkir dari isi (kesimpulan) jika ada (c) memeriksa indeks dan apendiks(jika ada)

2 Membaca Sekilas Membaca sekilas atau membaca cepat adalah kegiatan membaca dengan mengandalakan kecepatan gerak mata dalam melihat dan memperhatikan bahan tertulis yang dibacanya dengan tujuan untuk mendapatkan informasi secara cepat Metode yang digunakan dalam melatihkan membaca cepat adalah (a) metode kosakata metode yang berusaha untuk menambah kosakata (b) Metode motivasi metode yang berusaha memotivasi pembaca(pemula) yang mengalami hambatan (c) Metode gerak mata metode yang mengembangkan kecepatan membaca dengan menigkatkan kecepatan gerak mata Hambatan-hambatan yang dapat mengurangi kecepatan mambaca (a) vokalisai atau berguman ketika membaca (b) membaca dengan menggerakan bibir tetapi tidak bersuara (c) kepala bergerak searah tulisan yang dibaca (d) subvokalisasi suara yang biasa ikut membaca di dalam pikiran kita (e) jari tangan selalu menunjuk tulisa yang sedang kit abaca (f) gerakan mata kembali pada kata-kata sebelumnya 3 Membaca Dangkal (Superficial Reading) membaca dangkal pada hakekatnya bertujuan untuk memperoleh pemahaman yang dangkal yang bersifat luaran yang tidak mendalam dari suatu bahan bacaan Membaca jenis ini biasanya dilakukan seseorang membaca demi kesenangan membaca bacaan ringan yang mendatangkan kesenangan kegembiraan sebagai pengisi waktu senggang

29 KD 131 Merumuskan hakikat (pengertiantujuanjenismanfaat) membaca Membaca jenis ini biasanya dilakukan seseorang membaca demi kesenangan membaca bacaan ringan yang mendatangkan kesenangan kegembiraan sebagai pengisi waktu senggang Berdasarkan karakteristik diatas kegiatan tersebut termasuk ke dalam membaca jenis a Membaca Survai (Survey Reading) b Membaca Sekilas c Membaca Dangkal (Superficial Reading)

d Membaca Nyaring Uraian No 28 30 KD 131 Merumuskan hakikat (pengertiantujuanjenismanfaat) membaca

Berikut adalah tujuan umum dalam aktifitas membaca kecuali a Membaca untuk memperoleh perincian-perincian atau fakta-fakta (reading for details or facts) Membaca tersebut

bertujuan untuk menemukan atau mengetahui penemuan-penemuan telah dilakukan oleh sang tokoh untuk memecahkan masalah-masalah yang dibuat oleh sang tokoh

b Membaca untuk mengetahui ukuran atau susunan organisasi cerita (reading for sequenceor organization) Membaca tersebut bertujuan untuk mengetahui bagian-bagian cerita dan hubungan antar bagian-bagian cerita

c Membaca untuk menyimpulkan atau membaca inferensi (reading for inference) d Membaca untuk memperoleh kekurangan suatu buku (finding mistakes)

Uraian Tujuan Membaca

Beranda Soal UKG Agus Zainal M (SDN Gudang 2)

Republik_ilalangyahoocoid

Berdasarkan maksud tujuan atau keintensifan serta cara dalam membaca di bawah ini Anderson dalam Tarigan (19799-10) mengemukakan beberapa tujuan membaca antara lain a Membaca untuk memperoleh perincian-perincian atau fakta-fakta (reading for details or facts) Membaca tersebut bertujuan untuk menemukan atau mengetahui penemuan-penemuan telah dilakukan oleh sang tokoh untuk memecahkan masalah-masalah yang dibuat oleh sang tokoh b Membaca untuk memperoleh ide-ide utama (reading for main ideas) Membaca untuk mengetahui topik atau masalah dalam bacaan Untuk menemukan ide pokok bacaan dengan membaca halamn demi halaman c Membaca untuk mengetahui ukuran atau susunan organisasi cerita (reading for sequenceor organization) Membaca tersebut bertujuan untuk mengetahui bagian-bagian cerita dan hubungan antar bagian-bagian cerita d Membaca untuk menyimpulkan atau membaca inferensi (reading for inference) Pembaca diharapkan dapat merasakan sesuatu yang dirasakan penulis e Membaca untuk mengelompokkan atau mengklasifikasikan (reading for classify) Membaca jenis ini bertujuan untuk menemukan hal-hal yang tidak wajar mengenai sesuatu hal (Anderson dalam Tarigan 197910) f Membaca untuk menilai atau mengevaluasai (reading to evaluate) Jenis membaca tersebut bertujuan menemukan suatu keberhasilan berdasarkan ukuran-ukuran tertentu Membaca jenis ini memerlukan ketelitian dengan membandingkan dan mengujinya kembali g Membaca untuk memperbandingkan atau mempertentangkan (reading to compare or contrast) Tujuan membaca tersebut adalah untuk menemukan bagaimana cara perbedaan atau persamaan dua hal atau lebih

31 KD 133 menemukan pesan pokok utama sebuah berita Simak penggalan berita berikut Hebat Siswa Indonesia Pertahankan Tradisi Emas di Olimpiade Fisika Tim Olimpiade Fisika Indonesia REPUBLIKACOID JAKARTA--Luar biasa Lima siswa Indonesia yang dikirim ke ajang Olimpiade Fisika atau International Physics Olympiad (IPhO) ke-41 di Zagreb Kroasia 17-25 Juli berhasil menyabet empat medali emas dan satu perak Pelajar yang menyumbang emas adalah Muhammad Sohibul Maromi (SMAN 1 Pamekasan Madura) Christian George Emor (SMA Lokon St Nikolaus Tomohon Sulawesi Utara) David Giovanni (SMAK Penabur Gading Serpong Banten) dan Kevin Soedyatmiko (SMAN 12 Jakarta) Sedangkan medali perak berhasil diraih oleh Ahmad Ataka Awwalur Rizqi (SMAN 1 Yogyakarta) Prestasi ini jauh lebih baik dibanding ajang Olimpiade Fisika ke-40 di Merida Yucatan Meksiko 2009 yang lalu Saat itu delegasi siswa Indonesia merebut satu medali emas dua medali perak dan satu perunggu Hasil empat medali emas dan satu perak ini hampir menyamai prestasi terbaik sebelumnya pada ajang Olimpiade Fisika ke-37 di Singapura Saat itu siswa Indonesia tidak hanya berhasil menyabet 4 medali emas namun juga meraih predikat `Absolute Winner` atas nama Mailoa Jonathan Pradana (SMAK 1 BPK Penabur Jakarta) Tapi yang terpenting lagi pelajar Indonesia berhasil mempertahankan tradisi emas di setiap ajang Olimpiade Fisika Red Endro Yuwanto Peristiwa yang diberitakan adalah a siswa Indonesia pertahankan tradisi emas di Olimpiade Matematika b siswa Indonesia pertahankan tradisi emas di Olimpiade Fisika c siswa Indonesia menyumbang emas di Olimpiade Matematika

d siswa Indonesia menyumbang perak di Olimpiade Fisika Uraian 32 KD 133 menemukan pesan pokok utama sebuah berita

Simak penggalan berita berikut Hebat Siswa Indonesia Pertahankan Tradisi Emas di Olimpiade Fisika Tim Olimpiade Fisika Indonesia REPUBLIKACOID JAKARTA--Luar biasa Lima siswa Indonesia yang dikirim ke ajang Olimpiade Fisika atau International Physics Olympiad (IPhO) ke-41 di Zagreb Kroasia 17-25 Juli berhasil menyabet empat medali emas dan satu perak Pelajar yang menyumbang emas adalah Muhammad Sohibul Maromi (SMAN 1 Pamekasan Madura) Christian George Emor (SMA Lokon St Nikolaus Tomohon Sulawesi Utara) David Giovanni (SMAK Penabur Gading Serpong Banten) dan Kevin Soedyatmiko (SMAN 12 Jakarta) Sedangkan medali perak berhasil diraih oleh Ahmad Ataka Awwalur Rizqi (SMAN 1 Yogyakarta) Prestasi ini jauh lebih baik dibanding ajang Olimpiade Fisika ke-40 di Merida Yucatan Meksiko 2009 yang lalu Saat itu delegasi siswa Indonesia merebut satu medali emas dua medali perak dan satu perunggu Hasil empat medali emas dan satu perak ini hampir menyamai prestasi terbaik sebelumnya pada ajang Olimpiade Fisika ke-37 di Singapura Saat itu siswa Indonesia tidak hanya berhasil menyabet 4 medali emas namun juga meraih predikat `Absolute Winner` atas nama Mailoa Jonathan Pradana (SMAK 1 BPK Penabur Jakarta) Tapi yang terpenting lagi pelajar Indonesia berhasil mempertahankan tradisi emas di setiap ajang Olimpiade Fisika Red Endro Yuwanto Lima siswa Indonesia berhasil mendapatkan a 4 emas 1 perak

b 1 emas 4 perak c 3 emas 2 perak d 2 perak 3 emas

Beranda Soal UKG Agus Zainal M (SDN Gudang 2)

Republik_ilalangyahoocoid

Uraian 33 KD 134 menemukan pesan pokok dalam wacana naratif seperti cerita rakyat puisi

Surat dari ibu Jika bayang telah pudar Dan elang laut pulang ke sarang Angin bertiup ke benua Tiang ndash tiang akan kering sendiri Dan nahkoda sudah tau pedoman Boleh engkau datang padaku Makna lambang dari nahkoda sudah tahu pedoman adalahhellip a Sudah mencari pedoman hidup b Sudah menemukan arah dan tujuan

c Sudah berilmu dan berpengalaman d Sudah mempunyai pasangan hidup

Uraian 34 KD 135 membandingkan berbagai jenis wacana bahasa indonesia (deskripsi narasi)

Pendekatan untuk mendapat tanggapan emosional pembaca ataupun kesan pembaca adalah contoh Pendekatan Deskripsi jenis a Pendekatan Ekspositoris b Pendekatan Impresionistik

c Pendekatan menurut sikap pengarang d Pendekatan Realistik

Uraian PENDEKATAN DESKRIPSI

Pendekatan dalam pendeskripsian dapat dibedakan menjadi beberapa kategori pendekatan yaitu

1 Pendekatan Ekspositoris

Dalam pendekatan ini kita berusaha agar deskripsi yang kita buat dapat memberi keterangan sesuai dengan keadan yang

sebenarnya sehingga pembaca dapat seolah-olah ikut melihat atau merasakan objek yang kita deskripsikan Karangan

jenis ini berisi daftar detail sesuatu secara lengkap sehingga pembaca dan penalarannya dapat memperoleh kesan

keseluruhan tentang sesuatu

2 Pendekatan Impresionistik

Tujuan deskripsi impresionistik ialah untuk mendapatkan tanggapan emosional pembaca ataupun kesan pembaca Corak

deskripsi ini diantaranya juga ditentukan oleh macam kesan apa yang diinginkan penulisnya

3 Pendekatan menurut Sikap Pengarang

Pendekatan ini sangat bergantung pada tujuan yang ingin dicapai sifat objek serta pembaca deskripsinya Dalam

menguraikan sebuah gagasan penulis mungkin mengharapkan agar pembaca merasa tidak puas terhadapa suatu

tindakan atau keadaan atau penulis menginginkan agar pembaca juga harus merasakan bahwa persoalan yang dihadapi

merupakan masalah yang gawat Penulis juga dapat membayangkan bahwa akan terjadi sesuatu yang tidak diinginkan

sehingga pembaca dari semula sudah disiapkan perasaan yang kurang enak seram takut dan sebagainya (Alkhaidah

1997) 35 kd 135 membandingkan berbagai jenis wacana bahasa indonesia (deskripsi narasi)

Prinsip-Prinsip karangan Narasi sebagai berikut kecuali a Alur b Penokohan c Amanat

d Sudut Pandang Uraian Prinsip-prinsip dasar narasi sebagai tumpuan berfikir adalah sebagai berikut

1 Alur (plot) biasa disebut dengan jalan cerita

2 Penokohan (rangkaian perbuatanaksi) 3 Latar (setting) yaitu tempat dan atau waktu terjadinya perbuatan tokoh atau peristiwa yang dialami tokoh 4 Sudut Pandang (Point of View)

a Narrator Serba Tahu (Omniscient point of view) Yaitu narrator bertindak sebagai pencipta segalanya b Narrator bertindak objektif (Objektive point of view) Pengarang tak member komentar apa pun dan pembaca bebas menafsirkan apa yang diceritakan pengarang c Narrator ikut aktif ( Narator acting) Narrator terlibat dalam cerita d Narrator sebagai peninjau

Beranda Soal UKG Agus Zainal M (SDN Gudang 2)

Republik_ilalangyahoocoid

Pelakunya adalah orang ketiga yang biasa disebut ldquodiardquo 36 KD 135 membandingkan berbagai jenis wacana bahasa indonesia (deskripsi narasi)

1 Menentukan tema atau amanat apa yang akan disampaikan 2 Menetapkan sasaran pembaca 3 Merancang peristiwa-peristiwa utama yang akan ditampilkan dalam bentuk skema perkembangandan akhir cerita 5 Memerinci peristiwa-peristiwa utama ke dalam detail-detail peristiwa sebagai pendukung cerita 6 Menyusun tokoh dan perwatakanserta latar dan sudut pandang Karakteristik diatas adalah pengembangan dari paragraf a Narasi

b Deskprisi c Ekspositoris d Sudut Pandang Persuasif

Uraian Pengembangan karangan narasi dapat dilakukan dengan langkah-langkah berikut

(1) menentukan tema atau amanat apa yang akan disampaikan (2) menetapkan sasaran pembaca (3) merancang peristiwa-peristiwa utama yang akan ditampilkan dalam bentuk skema alur (4) membagi peristiwa utama ke dalam ke dalam bagian awal perkembangan dan akhir cerita (5) memerinci peristiwa-peristiwa utama ke dalam detail-detail peristiwa sebagai pendukung cerita dan (6) menyusun tokoh dan perwatakan latar dan sudut pandang

37 KD 135 membandingkan berbagai jenis wacana bahasa indonesia (deskripsi narasi) Karangan narasi yang berusaha untuk memberikan suatu maksud tertentu menyampaikan suatu amanat terselubung kepada para pembaca atau pendengar sehingga tampak seolah-olah melihat disebut a Narasi Informatif b Narasi Ekspositoris c Narasi Sugestif d Narasi Artistik

Uraian Jenis-Jenis Narasi

Narasi Informatif Narasi informatif adalah narasi yang memiliki sasaran penyampaian informasi secara tepat tentang suatu peristiwa

dengan tujuan memperluas pengetahuan orang tentang kisah seseorang Narasi Ekspositorik

Narasi ekspositorik adalah narasi yang memiliki sasaran penyampaian informasi secara tepat tentang suatu peristiwa dengan tujuan memperluas pengetahuan orang tentang kisah seseorang Dalam narasi ekspositorik penulis menceritakan suatu peristiwa berdasarkan data yang sebenarnya Pelaku yang ditonjolkan biasanya satu orang Pelaku diceritakan mulai dari kecil sampai saat ini atau sampai terakhir dalam kehidupannya Karangan narasi ini diwarnai oleh eksposisi maka ketentuan eksposisi juga berlaku pada penulisan narasi ekspositprik Ketentuan ini berkaitan dengan penggunaan bahasa yang logis berdasarkan fakta yang ada tidak memasukan unsursugestif atau bersifat objektif

Narasi Artistik

Narasi artistik adalah narasi yang berusaha untuk memberikan suatu maksud tertentu menyampaikan suatu amanat terselubung kepada para pembaca atau pendengar sehingga tampak seolah-olah melihat Ketentuan ini berkaitan dengan penggunaan bahasa yang logis berdasarkan fakta yang ada tidak memasukan unsur sugestif atau bersifat objektif

Narasi Sugestif Narasi sugestif adalah narasi yang berusaha untuk memberikan suatu maksud tertentu menyampaikan suatu amanat

terselubung kepada para pembaca atau pendengar sehingga tampak seolah-olah melihat 38 KD 135 membandingkan berbagai jenis wacana bahasa indonesia (deskripsi narasi)

Ciri karangan Narasi yang benar kecuali a Menonjolkan unsur perbuatan atau tindakan Membuat pembaca atau pendengar merasakan sendiri atau

mengalami sendiri

b Ada konfiks menjawab pertanyaan apa yang terjadi c Dirangkai dalam urutan waktu menggambarkan dengan jelas suatu peristiwa d Berisi ajakan dirangkai dalam urutan waktu

Uraian Ciri-Ciri Karangan Narasi Menurut Gorys Keraf (2000136)

Menonjolkan unsur perbuatan atau tindakan Dirangkai dalam urutan waktu Berusaha menjawab pertanyaan apa yang terjadi Ada konfiks

39 kd 135 membandingkan berbagai jenis wacana bahasa indonesia (deskripsi narasi) Langkah menyusun paragraf deskripsi yang benar adalah (1)Tentukan objek atau tema yang akan dideskripsikan (2)Mengumpulkan data dengan mengamati objek yang akan dideskripsikan (3)Menyusun data tersebut ke dalam urutan yang baik (menyusun kerangka karangan) (4)Menguraikan kerangka karangan menjadi dekripsi yang sesuai dengan tema yang ditentukan

Beranda Soal UKG Agus Zainal M (SDN Gudang 2)

Republik_ilalangyahoocoid

(5)Tentukan tujuan a 1-5-2-3-4

b 1-5-4-3-2 c 1-5-3-2-4 d 1-3-4-2-5

Uraian Langkah menyusun deskripsi

1Tentukan objek atau tema yang akan dideskripsikan 2Tentukan tujuan 3Mengumpulkan data dengan mengamati objek yang akan dideskripsikan 4Menyusun data tersebut ke dalam urutan yang baik (menyusun kerangka karangan) 5Menguraikan kerangka karangan menjadi dekripsi yang sesuai dengan tema yang ditentukan

40 Kd 311 Merancang aktivitas pembelajaran berdasarkan prinsip dan teori pembelajaran matematika Dalam tahap ini penyajian yang dilakukan melalui tindakan anak secara langsung terlihat dalam memanipulasi (mengotak atik)objek Proses tersebut menurut Bruner dikategorikan ke dalam model a Tahap Enaktif

b Tahap Ikonik c Tahap Simbolik d Tahap Implikatif

Uraian 1 Model Tahap Enaktif

Dalam tahap ini penyajian yang dilakukan melalui tindakan anak secara langsung terlihat dalam memanipulasi (mengotak atik)objek Contoh Budi mempunyai 2 pensil kemudian ibunya memberikannya lagi 3 pinsil Berapa banyak pensil Budi sekarang

2 Model Tahap Ikonik Dalam tahap ini kegiatan penyajian dilakukan berdasarkan pada pikiran internal dimana pengetahuan disajikan melalui serangkaian gambar-gambar atau grafik yang dilakukan anak berhubungan dengan mental yang merupakan gambaran dari objek-objek yang dimanipulasinya Contoh + = hellip

3 Model Tahap Simbolis Dalam tahap ini bahasa adalah pola dasar simbolik anak memanipulasi Simbol-simbol atau lambang-lambang objek tertentu Contoh 2 pensil + 3 pensil = hellip pensil

41 Kd 311 Merancang aktivitas pembelajaran berdasarkan prinsip dan teori pembelajaran matematika Dalam tahap ini kegiatan penyajian dilakukan berdasarkan pada pikiran internal dimana pengetahuan disajikan melalui serangkaian gambar-gambar atau grafik yang dilakukan anak Proses tersebut menurut Bruner dikategorikan ke dalam model a Tahap Enaktif b Tahap Ikonik

c Tahap Simbolik d Tahap Implikatif

Uraian No 40 42 Kd 311 Merancang aktivitas pembelajaran berdasarkan prinsip dan teori pembelajaran matematika

Objek belajar matematika dibagi kedalam Objek Langsung dan Objek Tak Langsung Hal tersebut adalah teori belajar Matematika menurut a Robert M Gagne

b Jerome S Burner c Thorndike d Skinner

Uraian Teori yang diperkenalkan Robert MGagne pada tahun 1960-an pembelajaran harus dikondisikan untuk memunculkan

respons yang diharapkanMenurut Gagne (dalam Ismail 1998) belajar matematika terdiri dari objek langsung dan objek tak langsung

43 Kd 311 Merancang aktivitas pembelajaran berdasarkan prinsip dan teori pembelajaran matematika Perhatikan contoh berikut Budi mempunyai 2 pinsil kemudian ibunya memberikannya lagi 3 pinsilBerapa banyak pinsil Budi sekarang Hal tersebut dikemukakan Bruner dalam Proses Pembelajaran Matematika dalam tahap a Simbolik b Ikonik c Implikatif d Enaktif

Uraian No 40

Beranda Soal UKG Agus Zainal M (SDN Gudang 2)

Republik_ilalangyahoocoid

44 Kd 311 Merancang aktivitas pembelajaran berdasarkan prinsip dan teori pembelajaran matematika Perhatikan gambar disamping Ilustrasi di samping dikemukakan Bruner dalam Proses Pembelajaran Matematika dalam tahap

a Simbolik b Ikonik

c Implikatif d Enaktif

Uraian No 40 45 Kd 311 Merancang aktivitas pembelajaran berdasarkan prinsip dan teori pembelajaran matematika

Perhatikan contoh berikut Contoh 2 pinsil + 3 pinsil = hellippinsil Contoh tersebut dikemukakan Bruner dalam Proses Pembelajaran Matematika dalam tahap a Simbolik

b Ikonik c Implikatif d Enaktif

Uraian No 40 46 Kd 311 Merancang aktivitas pembelajaran berdasarkan prinsip dan teori pembelajaran matematika

Perhatikan contoh berikut penjumlahan bilangan positif dan negatif siswa mencoba sendiri dengan menggunakan garis bilangan Contoh tersebut dikemukakan Bruner dalam Teorema Pembelajaran Matematika a Penyusunan

b Notasi c Pengkontrasan dan Keanekaragaman d Pengaitan

Uraian Berdasarkan hasil pengamatannya Brunner merumuskan 5 teorema dalam pembelajaran matematika yaitu

1) Teorema Penyusunan Menerangkan bahwa cara yang terbaik memulai belajar suatu konsep matematika dalil defenisi dan semacamnya adalah dengan cara menyusun penyajiannya Misalnya dalam mempelajari penjumlahan bilangan positif dan negatif siswa mencoba sendiri dengan menggunakan garis bilangan 2) Teorema Notasi Menerangkan bahwa dalam pengajaran suatu konsep penggunaan notasi-notasi matematika harus diberikan secara bertahap dari yang sederhana ke yang lebih kompleks 3) Teorema Pengkontrasan dan Keanekaragaman Menerangkan bahwa pengontrasan dan keanekaragaman sangat penting dalam melakukan pengubahan konsep matematika dari yang konkrit ke yang lebih abstrak Dalam hal ini diperlukan banyak contoh Contoh yang diberikan harus sesuai dengan rumusan yang diberikan Misalnya menjelaskan persegi panjang disertai juga kemungkinan jajaran genjang dan segi empat lainnya selain persegi panjnag Dengan demikian siswa dapat membedakan apakah segi empat yang diberikan padanya termasuk persegi panjang atau tidak 4) Teorema Pengaitan Menerangkan bahwa dalam matematika terdapat hubungan yang berkaitan antara satu konsep dengan konsep yang lain Di mana materi yang satu merupakan prasyarat yang harus diketahui untuk mempelajari materi yang lain

47 Kd 311 Merancang aktivitas pembelajaran berdasarkan prinsip dan teori pembelajaran matematika Perhatikan contoh berikut Guru menjelaskan persegi panjang disertai juga kemungkinan jajaran genjang dan segi empat lainnya selain persegi panjnag Dengan demikian siswa dapat membedakan apakah segi empat yang diberikan padanya termasuk persegi panjang atau tidak Contoh tersebut dikemukakan Bruner dalam Teorema Pembelajaran Matematika a Penyusunan b Notasi c Pengkontrasan dan Keanekaragaman

d Pengaitan Uraian No 46 48 Kd 311 Merancang aktivitas pembelajaran berdasarkan prinsip dan teori pembelajaran matematika

Tahap pembelajaran Matematika menurut Van Halle adalah sebagai berikut kecuali a Tahap Pengenalan b Tahap Pengurutan c Tahap Analisis d Tahap Penyimpulan

Beranda Soal UKG Agus Zainal M (SDN Gudang 2)

Republik_ilalangyahoocoid

Uraian Van Hiele menyatakan bahwa terdapat 5 tahap belajar siswa dalam belajar geometri yaitu

a Tahap Pengenalan Pada tahap ini siswa mulai belajar mengenal suatu bangun geometri secara keseluruhan namun belum mampu mengetahui adanya sifat-sifat dari bangun geometri yang dilihatnya b Tahap Analisis Pada tahap ini siswa sudah mulai mengenal sifat-sifat yang dimiliki bangun geometri yang diamatinya c Tahap Pengurutan Pada tahap ini siswa sudah mengenal dan memahami sifat-sifat suatu bangun geometri serta sudah dapat mengurutkan bangun-bangun geometri yang satu sama yang lainnya saling berhubungan d Tahap Deduksi Pada tahap ini siswa telah mampu menarik kesimpulan secara deduktif yaitu menarik kesimpulan yang bersifat umum dan menuju ke hal yang bersifat khusus serta dapat mengambil kesimpulan e Tahap Akurasi Pada tahap ini siswa mulai menyadari pentingnya ketepatan prinsip-prinsip dasar yang melandasi suatu pembuktian Tahap berfikir ini merupakan tahap berfikir yang paling tinggi rumit dan kompleks karena di luar jangkauan usia anak-anak SD sampai tingakat SMP

49 Kd 311 Merancang aktivitas pembelajaran berdasarkan prinsip dan teori pembelajaran matematika middot Matematika untuk tujuan pembelajaran dianalisis sebagai kumpulan fakta yang berdiri sendiri dan tidak saling berkaitan middot Anak diharuskan menguasai unsur-unsur yang banyak sekali tanpa diperhatikan pengertiannya middot Anak mempelajari unsur-unsur dalam bentuk seperti yang akan digunakan nanti dalam kesempatan lain middot Anak akan mencapai tujuan ini secara efektif dan efisien dengan melalui pengulangan Teori Pembelajaran Matematika tersebut dikemukakan oleh a Skinner b Piaget c Van Brownell

d Thorndike Uraian Brownell mengemukakan tentang Teori Makna (Meaning Theory) sebagai pengganti Teori Latihan HafalUlangan (Drill

Theory) Intisari dari teori Drill adalah - Matematika untuk tujuan pembelajaran dianalisis sebagai kumpulan fakta yang berdiri sendiri dan tidak saling berkaitan - Anak diharuskan menguasai unsur-unsur yang banyak sekali tanpa diperhatikan pengertiannya - Anak mempelajari unsur-unsur dalam bentuk seperti yang akan digunakan nanti dalam kesempatan lain - Anak akan mencapai tujuan ini secara efektif dan efisien dengan melalui pengulangan

50 Kd 311 Merancang aktivitas pembelajaran berdasarkan prinsip dan teori pembelajaran matematika Tahap operasionaloperasi konkrit adalah teori Pembelajaran Matematika yang dikemukakan Peaget pada tahap usia a dibawah 2 tahun b 2-7 Tahun c 7-8 Tahun d 7-12 Tahun

Uraian Piaget membagi skema yang digunakan anak untuk memahami dunianya melalui empat periode utama yang berkorelasi

dengan dan semakin canggih seiring pertambahan usia Periode sensorimotor (usia 0ndash2 tahun) Periode praoperasional (usia 2ndash7 tahun) Periode operasional konkrit (usia 7ndash12 tahun) Periode operasional formal (usia 12 tahun sampai dewasa)

51 313 Memilih media pembelajaran yang tepat untuk pembelajaran operasi bilangan bulat Untuk operasi bilangan bulat perkalian media yang paling tepat digunakan adalah a sapu lidi

b kerikil c daun pakis d koin

Uraian 52 313 Memilih media pembelajaran yang tepat untuk pembelajaran operasi bilangan bulat

Untuk operasi bilangan bulat negatif kita bisa menggunakan media a uang kertas b kerikil c mistar d koin

Uraian 53 313 Memilih media pembelajaran yang tepat untuk pembelajaran operasi bilangan bulat

Beranda Soal UKG Agus Zainal M (SDN Gudang 2)

Republik_ilalangyahoocoid

Untuk operasi bilangan bulat penjumlahan media yang paling tepat digunakan adalah a uang kertas b kerikil c daun pakis d koin

Uraian 54 314 Memilih media pembelajaran yang tepat untuk pembelajaran operasi bilangan pecahan

Untuk operasi bilangan pecahan kita bisa menggunakan media berikut kecuali a kartu bilangan b garis bilangan c gambar bidang d blok pecahan

Uraian 55 316 Memilih media pembelajaran yang tepat untuk pembelajaran geometri dan pengukuran

Untuk menghitung luas dan keliling bangun datar media yang paling tepat digunakan adalah a Penggaris b Kertas Folio Bergaris

c Neraca d Kalkulator

Uraian

Page 4: 50 SOAL UKG & PEMBAHASAN SESUAI KISI-KISIS.pdf

Beranda Soal UKG Agus Zainal M (SDN Gudang 2)

Republik_ilalangyahoocoid

2) Pendekatan integratif menekankan keterpaduan empat aspek keterampilan berbahasa (menyimak berbicara membaca dan menulis) dalam pembelajaran

3) Pendekatan keterampilan proses memfokuskan keterampilan siswa dalam mengamati mengklasifikasi menginterpretasi dan mengkomunikasikan

4) Pendekatan tematis menekankan tema pembelajaran sebagai payungpemandu dalam pembelajaran 14 KD 113 Aspek Menulis di kelas tinggi

Teknik menulis cerita terdiri atas hal-hal sebagai berikut kecuali a menjawab pertanyaan b membuat kalimat c subtitusi d persuasi

Uraian Teknik menyusun cerita dapat dilakukan dengan menjawab pertanyaan melengkapi kalimat memperbaiki susunan kalimat

memperluas kalimat subtitusi transfomtasi dan membuat kalimat Persuasi adalah bujukan halus

15 KD 113 Aspek Menulis di kelas tinggi Model pembelajaran menulis ceritacerpen di SD meliputi hal-hal berikut kecuali a Menceritakan gambar b Melanjutkan cerita c Menceritakan pengalaman d Mendeskripsikan cerita

Uraian Model pembelajaran menulis ceritacerpen di SD meliputi menceritakan gambar melanjutkan ceria lain menceitakan

mimpi menceritakan pengalaman dan menceritakan cita-cita 16 KD 121 Memilih berbagai Metode Menulis Permulaan

1) Menulis huruf lepas 2) Merangkaikan huruf lepas menjadi suku kata 3) Merangkaikan suku kata menjadi kata 4) Menyusun kata menjadi kalimat (Djauzak 19964) Tahapan diatas adalah Teknik Menulis Permulaan dengan metode a Metode Eja

b Metode Kata Lembaga c Metode SAS d Metode Global

Uraian Metode dan pembelajaran menulis permulaan

a Metode Eja Metode eja di dasarkan pada pendekatan harfiah artinya belajar membaca dan menulis dimulai dari huruf-huruf yang dirangkaikan menjadi suku kata Oleh karena itu pengajaran dimulai dari pengenalan huruf-huruf Demikian halnya dengan pengajaran menulis di mulai dari huruf lepas dengan langka-langkah sebagai berikut 1) Menulis huruf lepas 2) Merangkaikan huruf lepas menjadi suku kata 3) Merangkaikan suku kata menjadi kata 4) Menyusun kata menjadi kalimat (Djauzak 19964) b Metode kata lembaga Metode kata lembaga di mulai mengajar dengan langkah-langkah sebagai berikut 1) Mengenalkan kata 2) Merangkaikan kata antar suku kata 3) Menguraikan suku kata atas huruf-hurufnya 4) Menggabungkan huruf menjadi kata (Djauzak 19965) c Metode Global Metode global memulai pengajaran membaca dan menulis permulaan dengan membaca kalimat secara utuh yang ada di bawah gambar Menguraikan kalimat dengan kata-kata menguraikan kata-kata menjadi suku kata (Djauzak 19966) d Metode SAS Menuryut (Supriyadi 1996 334-335) pengertian metode SAS adalah suatu pendekatan cerita di sertai dengan gambar yang didalamnya terkandung unsur analitik sintetik Metode SAS menurut (Djuzak19968) adalah suatu pembelajaran menulis permulaan yang didasarkan atas pendekatan cerita yakni cara memulai mengajar menulis dengan menampil cerita yang diambil dari dialog siswa dan guru atau siswa dengan siswa Teknik pelaksanaan pembelajaran metode SAS yakni keterampilan menulis kartu huruf kartu suku kata kartu kata dan kartu kalimat sementara sebagian siswa mencari huruf suku kata dan kata guru dan sebagian siswa menempel kata-kata yang tersusun sehingga menjadi kalimat yang berarti (Subana) Proses operasional metode SAS mempunyai langkah-lagkah dengan urutan sebagai berikut a Struktur yaitu menampilkan keseluruhan b Analitik yatu melakukan proses penguraian c Sintetik yaitu melakukan penggalan pada struktur semula

Beranda Soal UKG Agus Zainal M (SDN Gudang 2)

Republik_ilalangyahoocoid

17 KD 121 Memilih berbagai Metode Menulis Permulaan 1) Mengenalkan kata 2) Merangkaikan kata antar suku kata 3) Menguraikan suku kata atas huruf-hurufnya 4) Menggabungkan huruf menjadi kata (Djauzak 19965) Tahapan diatas adalah Teknik Menulis Permulaan dengan metode a Metode Eja b Metode Kata Lembaga

c Metode SAS d Metode Global

Uraian No 16 18 KD 121 Memilih berbagai Metode Menulis Permulaan

Metode dengan memulai pengajaran membaca dan menulis permulaan dengan membaca kalimat secara utuh yang ada di bawah gambar Menguraikan kalimat dengan kata-kata menguraikan kata-kata menjadi suku kata disebut dengan metode a Metode Eja b Metode Kata Lembaga c Metode SAS d Metode Global

Uraian No 16 19 KD 121 Memilih berbagai Metode Menulis Permulaan

Suatu pembelajaran menulis permulaan yang didasarkan atas pendekatan cerita yakni cara memulai mengajar menulis dengan menampil cerita yang diambil dari dialog siswa dan guru atau siswa dengan siswa disebut dengan metode a Metode Eja b Metode Kata Lembaga c Metode SAS

d Metode Global Uraian No 16 20 KD 121 Memilih berbagai Metode Menulis Permulaan

menulis kartu huruf kartu suku kata kartu kata dan kartu kalimat sementara sebagian siswa mencari huruf suku kata dan kata guru dan sebagian siswa menempel kata-kata yang tersusun sehingga menjadi kalimat yang berarti adalah contoh metode a Metode Eja b Metode Kata Lembaga c Metode SAS

d Metode Global Uraian No 16 21 KD 122 Merancang berbagai kegiatan menulis di kelas tinggi

Berikut adalah kegiatan menulis lanjutan di kelas tinggi kecuali a menulis tentang berbagai topik b menulis pengumuman c menulis pantun d menulis memo

Uraian 22 KD 123 Perencanaan dan Pelaksanaan evaluasi pembelajaran Bahasa dan Sastra Indonesia

Perencanaan Pengajaran meliputi hal-hal berikut kecuali a tujuan apa yang hendak dicapai b memilih bahan ajar c proses belajar mengajar d alat penilaian

Uraian Menurut Akhlan dan Rahman (199715) perencanaan pengajaran meliputi

a tujuan apa yang hendak dicapai b bahan pengajaran c proses belajar mengajardan d alat penilaian

23 KD 123 Perencanaan dan Pelaksanaan evaluasi pembelajaran Bahasa dan Sastra Indonesia karakteristik perencanaan pengajaran yang baik hendaknya mengandung prinsip sebagai berikut a Memiliki sikap objektif rasio (tepat dan masuk akal) komprehensif dan sistematis (menyeluruh dan tersusun rapi)

b Merupakan suatu wahana atau wadah untuk mengembangkan segala potensi yang ada dan dimiliki oleh anak

Beranda Soal UKG Agus Zainal M (SDN Gudang 2)

Republik_ilalangyahoocoid

didik c Mengendalikan kekuatan sendiri bukan didasarkan atas kekuatan orang lain d Melakukan studi kasus yang berkesinambungan

Uraian Menurut Akhlan dan Rahman (19977) karakteristik perencanaan pengajaran yang baik hendaknya mengandung prinsip

sebagai berikut a Mengembangkan hubungan interaksi yang baik di antara sesama manusia dalam hal ini siswa dan guru serta personal

terkait b Merupakan suatu wahana atau wadah untuk mengembangkan segala potensi yang ada dan dimiliki oleh anak didik c Memiliki sikap objektif rasio (tepat dan masuk akal) komprehensif dan sistematis (menyeluruh dan tersusun rapi) d Mengendalikan kekuatan sendiri bukan didasarkan atas kekuatan orang lain Didukung oleh fakta dan data yang

menunjang pencapaian tujuan yang telah di dirumuskan e Fleksibel dan dinamis artinya mudah disesuaikan dengan keadaan serta perkembangan ke arah yang lebih baik dan

maju 24 KD 123 Perencanaan dan Pelaksanaan evaluasi pembelajaran Bahasa dan Sastra Indonesia

tahap mengumpulkan informasi tentang keadaan objek evaluasi (siswa) dengan menggunakan teknik tes atau nontes disebut tahapan a Tahap Tindak Lanjut b Tahap Persiapan c Tahap Pelaksanaan

d Tahap Pengolahan Hasil Uraian Berikut ini penjelasan singkat tentang keempat tahap evaluasi pembelajaran tersebut

(1) Tahap Persiapan Menurut Damaianti (2007 8) tahap ini disebut juga tahap perencanaan dan perumusan kriterium Langkahnya meliputi (a) perumusan tujuan evaluasi (b) penetapan aspek-aspek yang akan dievaluasi (c) menetapkan metode dan bentuk evaluasi (tesnontes) (d) merencanakan waktu evaluasi (e) melakukan uji coba (untuk tes) agar dapat mengukur validitas dan reliabilitasnya Untuk evaluasi yang menggunakan tes hasil dari tahap ini adalah kisi-kisi soal dan seperangkat alat tes soal lembar jawaban (untuk tes tulis) kunci jawaban dan pedoman penilaian 2) Tahap Pelaksanaan Tahap pelaksanaan atau disebut juga dengan tahap pengukuran dan pengumpulan data adalah tahap untuk mengumpulkan informasi tentang keadaan objek evaluasi (siswa) dengan menggunakan teknik tes atau nontes Bila menggunakan teknik tes soal yang igunakan sebaiknya sudah teruji validitas dan reliabilitasnya Tes yang digunakan dapat berbentuk tes tulis lisan atau praktik 3) Tahap Pengolahan Hasil Tahap pengolahan hasil adalah tahap pemeriksaan hasil evaluasi dengan memberikan skor Skor yang diperoleh siswa selanjutnya diubah menjadi nilai Pada tes tulis pemeriksaan hasil dilakukan setelah tes selesai sedangkan pada tes lisan dan praktik pemberian nilai dilakukan bersamaan dengan waktu pelasanaan tes tersebut 4) Tahap Tindak Lanjut Tahap tindak lanjut atau disebut juga tahap penafsiran adalah tahap untuk mengambil keputusan berdasarkan nilai yang dihasilkan pada tahap pengolahan hasil misalnya a memperbaiki proses belajar mengajar b memperbaiki kesulitan belajar siswa c memperbaiki alat evaluasi d membuat laporan evaluasi (rapor)

25 KD 131 Merumuskan hakikat (pengertiantujuanjenismanfaat) membaca membaca yang mengutamakan isi bacaan sebagai ungkapan pikiran perasaan dan kehendak penulis Bila hanya ingin mengetahui isinya membaca cerdas bersifat lugas Akan tetapi bila maksudnya untuk memahami dan memilki isi bacaan maka tergolong kedalam membaca jenis a Membaca cerdas atau membaca dalam hati

b Membaca bahasa c Membaca teknis d Membaca bebas

Uraian Menurut Soedjono dalam Sue (200418-21) ada lima macam membaca yaitu membaca bahasa membaca cerdas atau

membaca dalam hati membaca teknis membaca emosional dan membaca bebas 1) Membaca bahasa Membaca bahasa adalah membaca yang mengutamakan bahasa bacaan Membaca bahasa mementingkan segi bahasa bacaan Hal-hal yang perlu diperhatikan dalam membaca bahasa adalah kesesuian pikir dengan bahasa perbendaharaan bahasa yang meliputi kosa kata struktur kalimat dan ejaan 2) Membaca cerdas atau membaca dalam hati

Beranda Soal UKG Agus Zainal M (SDN Gudang 2)

Republik_ilalangyahoocoid

Membaca cerdas adalah membaca yang mengutamakan isi bacaan sebagai ungkapan pikiran perasaan dan kehendak penulis Bila hanya ingin mengetahui isinya membaca cerdas bersifat lugas Akan tetapi bial maksudnya untuk memahami dan memilki isi bacaan maka disebut membaca belajar 3) Membaca teknis Membaca teknis adalah membaca dengan mengarahkan bacaan secara wajar Wajar maksudnya sesuai ucapan tekanan dan intonasinya Pikiran perasaan dan kemauan yang tersimpan dalam bacaan dapat diaktualisasikan dengan baik 4) Membaca emosional Membaca emosional adalah membaca sebagai sarana untuk memasuki perasaan yaitu keindahan isi dan keindahan bahasanya 5) Membaca bebas Membaca bebas adalah membaca sesuatu atas kehendak sendiri tanpa adanya unsur paksaan dari luar Unsur dari luar misalnya guru orang tua teman atau pihak-pihak lain

26 KD 131 Merumuskan hakikat (pengertiantujuanjenismanfaat) membaca membaca sesuatu atas kehendak sendiri tanpa adanya unsur paksaan dari luar Unsur dari luar misalnya guru orang tua teman atau pihak-pihak lain termasuk jenis membaca a Membaca cerdas atau membaca dalam hati b Membaca bahasa c Membaca teknis d Membaca bebas

Uraian No 25 27 KD 131 Merumuskan hakikat (pengertiantujuanjenismanfaat) membaca

1 menggunakan ucapan yang tepat 2 menggunakan frase yang tepat 3 menggunakan intonasi suara yang wajar 4 dalam posisi sikap yang baik 5 menguasai tanda-tanda baca 6 membaca dengan terang dan jelas keterampilan di atas harus dipunyai dalam jenis membaca a Membaca cerdas atau membaca dalam hati b Membaca bahasa c Membaca teknis d Membaca nyaring

Uraian JENIS-JENIS MEMBACA DAN KARAKTERISTIKNYA

Ditinjau dari segi terdengar atau tidaknya suara pembaca waktu melakukan kegiatan membaca maka proses membaca dapat dibedakan menjadi A Membaca Nyaring Membaca nyaring adalah kegiatan membaca dengan menyuarakan tulisan yang dibacanya dengan ucapan dan intonasi yang tepat agar pendengar dan pembaca dapat menangkap informasi yang disampaikan oleh penulis baik yang berupa pikiran perasaan sikap ataupun pengalaman penulis Ketrampilan yang dituntut dalam membaca nyaring adalah berbagai kemampuan diantaranya adalah 1 menggunakan ucapan yang tepat 2 menggunakan frase yang tepat 3 menggunakan intonasi suara yang wajar 4 dalam posisi sikap yang baik 5 menguasai tanda-tanda baca 6 membaca dengan terang dan jelas 7 membaca dengan penuh perasaan ekspresif 8 membaca dengan tidak terbata-bata 9 mengerti serta memahami bahan bacaan yang dibacanya 10 kecepatan bergantung pada bahan bacaan yang dibacanya 11 membaca dengan tanpa terus-menerus melihat bahan bacaan 12 membaca dengan penuh kepercayaan pada diri sendiri B Membaca Dalam Hati Membaca dalam hati adalah kegiatan membaca yang dilakukan dengan tanpa menyuarakan isi bacaan yang dibacanya Ketrampilan yang dituntut dalam membaca dalam hati antara lain sebagai berikut 1 membaca tanpa bersuara tanpa bibir bergerak tanpa ada desis apapun 2 membaca tanpa ada gerakan-gerakan kepala 3 membaca lebih cepat dibandingkan dengan membaca nyaring 4 tanpa menggunakan jari atau alat lain sebagai penunjuk 5 mengerti dan memahami bahan bacaan 6 dituntut kecepatan mata dalam membaca 7 membaca dengan pemahaman yang baik

Beranda Soal UKG Agus Zainal M (SDN Gudang 2)

Republik_ilalangyahoocoid

8 dapat menyesuaikan kecepatan dengan tingkat kesukaran yang terdapat dalam bacaan 28 KD 131 Merumuskan hakikat (pengertiantujuanjenismanfaat) membaca

Hal berikut dilakukan seseorang ketika membaca survai kecuali a memeriksa judul bacaanbuku kata pengantar daftar isi dan malihat abstrak(jika ada) b memeriksa bagian terahkir dari isi (kesimpulan) jika ada c memeriksa indeks dan apendiks(jika ada) d membaca biografi pengarang

Uraian Secara garis besar membaca dalam hati dapat dibedakan menjadi dua (I) MEMBACA EKSTENSIF amp (II) MEMBACA

INTENSIF Berikut penjelasan secara rinci kedua jenis membaca tersebut I Membaca Ekstensif membaca ekstensif adalah membaca secara luas Objeknya meliputi sebanyak mungkin teks dalam waktu yang sesingkat-singkatnya Membaca ekstensif meliputi 1 Membaca Survai (Survey Reading) Membaca survai adalah kegiatan membaca untuk mengetahui secara sekilas terhadap bahan bacaan yang akan dibaca lebih mendalam Kegiatan membaca survai merupakan pendahuluan dalam membaca ekstensif Yang dilakukan seseorang ketika membaca survai adalah sebagai berikut (a) memeriksa judul bacaanbuku kata pengantar daftar isi dan malihat abstrak(jika ada) (b) memeriksa bagian terahkir dari isi (kesimpulan) jika ada (c) memeriksa indeks dan apendiks(jika ada)

2 Membaca Sekilas Membaca sekilas atau membaca cepat adalah kegiatan membaca dengan mengandalakan kecepatan gerak mata dalam melihat dan memperhatikan bahan tertulis yang dibacanya dengan tujuan untuk mendapatkan informasi secara cepat Metode yang digunakan dalam melatihkan membaca cepat adalah (a) metode kosakata metode yang berusaha untuk menambah kosakata (b) Metode motivasi metode yang berusaha memotivasi pembaca(pemula) yang mengalami hambatan (c) Metode gerak mata metode yang mengembangkan kecepatan membaca dengan menigkatkan kecepatan gerak mata Hambatan-hambatan yang dapat mengurangi kecepatan mambaca (a) vokalisai atau berguman ketika membaca (b) membaca dengan menggerakan bibir tetapi tidak bersuara (c) kepala bergerak searah tulisan yang dibaca (d) subvokalisasi suara yang biasa ikut membaca di dalam pikiran kita (e) jari tangan selalu menunjuk tulisa yang sedang kit abaca (f) gerakan mata kembali pada kata-kata sebelumnya 3 Membaca Dangkal (Superficial Reading) membaca dangkal pada hakekatnya bertujuan untuk memperoleh pemahaman yang dangkal yang bersifat luaran yang tidak mendalam dari suatu bahan bacaan Membaca jenis ini biasanya dilakukan seseorang membaca demi kesenangan membaca bacaan ringan yang mendatangkan kesenangan kegembiraan sebagai pengisi waktu senggang

29 KD 131 Merumuskan hakikat (pengertiantujuanjenismanfaat) membaca Membaca jenis ini biasanya dilakukan seseorang membaca demi kesenangan membaca bacaan ringan yang mendatangkan kesenangan kegembiraan sebagai pengisi waktu senggang Berdasarkan karakteristik diatas kegiatan tersebut termasuk ke dalam membaca jenis a Membaca Survai (Survey Reading) b Membaca Sekilas c Membaca Dangkal (Superficial Reading)

d Membaca Nyaring Uraian No 28 30 KD 131 Merumuskan hakikat (pengertiantujuanjenismanfaat) membaca

Berikut adalah tujuan umum dalam aktifitas membaca kecuali a Membaca untuk memperoleh perincian-perincian atau fakta-fakta (reading for details or facts) Membaca tersebut

bertujuan untuk menemukan atau mengetahui penemuan-penemuan telah dilakukan oleh sang tokoh untuk memecahkan masalah-masalah yang dibuat oleh sang tokoh

b Membaca untuk mengetahui ukuran atau susunan organisasi cerita (reading for sequenceor organization) Membaca tersebut bertujuan untuk mengetahui bagian-bagian cerita dan hubungan antar bagian-bagian cerita

c Membaca untuk menyimpulkan atau membaca inferensi (reading for inference) d Membaca untuk memperoleh kekurangan suatu buku (finding mistakes)

Uraian Tujuan Membaca

Beranda Soal UKG Agus Zainal M (SDN Gudang 2)

Republik_ilalangyahoocoid

Berdasarkan maksud tujuan atau keintensifan serta cara dalam membaca di bawah ini Anderson dalam Tarigan (19799-10) mengemukakan beberapa tujuan membaca antara lain a Membaca untuk memperoleh perincian-perincian atau fakta-fakta (reading for details or facts) Membaca tersebut bertujuan untuk menemukan atau mengetahui penemuan-penemuan telah dilakukan oleh sang tokoh untuk memecahkan masalah-masalah yang dibuat oleh sang tokoh b Membaca untuk memperoleh ide-ide utama (reading for main ideas) Membaca untuk mengetahui topik atau masalah dalam bacaan Untuk menemukan ide pokok bacaan dengan membaca halamn demi halaman c Membaca untuk mengetahui ukuran atau susunan organisasi cerita (reading for sequenceor organization) Membaca tersebut bertujuan untuk mengetahui bagian-bagian cerita dan hubungan antar bagian-bagian cerita d Membaca untuk menyimpulkan atau membaca inferensi (reading for inference) Pembaca diharapkan dapat merasakan sesuatu yang dirasakan penulis e Membaca untuk mengelompokkan atau mengklasifikasikan (reading for classify) Membaca jenis ini bertujuan untuk menemukan hal-hal yang tidak wajar mengenai sesuatu hal (Anderson dalam Tarigan 197910) f Membaca untuk menilai atau mengevaluasai (reading to evaluate) Jenis membaca tersebut bertujuan menemukan suatu keberhasilan berdasarkan ukuran-ukuran tertentu Membaca jenis ini memerlukan ketelitian dengan membandingkan dan mengujinya kembali g Membaca untuk memperbandingkan atau mempertentangkan (reading to compare or contrast) Tujuan membaca tersebut adalah untuk menemukan bagaimana cara perbedaan atau persamaan dua hal atau lebih

31 KD 133 menemukan pesan pokok utama sebuah berita Simak penggalan berita berikut Hebat Siswa Indonesia Pertahankan Tradisi Emas di Olimpiade Fisika Tim Olimpiade Fisika Indonesia REPUBLIKACOID JAKARTA--Luar biasa Lima siswa Indonesia yang dikirim ke ajang Olimpiade Fisika atau International Physics Olympiad (IPhO) ke-41 di Zagreb Kroasia 17-25 Juli berhasil menyabet empat medali emas dan satu perak Pelajar yang menyumbang emas adalah Muhammad Sohibul Maromi (SMAN 1 Pamekasan Madura) Christian George Emor (SMA Lokon St Nikolaus Tomohon Sulawesi Utara) David Giovanni (SMAK Penabur Gading Serpong Banten) dan Kevin Soedyatmiko (SMAN 12 Jakarta) Sedangkan medali perak berhasil diraih oleh Ahmad Ataka Awwalur Rizqi (SMAN 1 Yogyakarta) Prestasi ini jauh lebih baik dibanding ajang Olimpiade Fisika ke-40 di Merida Yucatan Meksiko 2009 yang lalu Saat itu delegasi siswa Indonesia merebut satu medali emas dua medali perak dan satu perunggu Hasil empat medali emas dan satu perak ini hampir menyamai prestasi terbaik sebelumnya pada ajang Olimpiade Fisika ke-37 di Singapura Saat itu siswa Indonesia tidak hanya berhasil menyabet 4 medali emas namun juga meraih predikat `Absolute Winner` atas nama Mailoa Jonathan Pradana (SMAK 1 BPK Penabur Jakarta) Tapi yang terpenting lagi pelajar Indonesia berhasil mempertahankan tradisi emas di setiap ajang Olimpiade Fisika Red Endro Yuwanto Peristiwa yang diberitakan adalah a siswa Indonesia pertahankan tradisi emas di Olimpiade Matematika b siswa Indonesia pertahankan tradisi emas di Olimpiade Fisika c siswa Indonesia menyumbang emas di Olimpiade Matematika

d siswa Indonesia menyumbang perak di Olimpiade Fisika Uraian 32 KD 133 menemukan pesan pokok utama sebuah berita

Simak penggalan berita berikut Hebat Siswa Indonesia Pertahankan Tradisi Emas di Olimpiade Fisika Tim Olimpiade Fisika Indonesia REPUBLIKACOID JAKARTA--Luar biasa Lima siswa Indonesia yang dikirim ke ajang Olimpiade Fisika atau International Physics Olympiad (IPhO) ke-41 di Zagreb Kroasia 17-25 Juli berhasil menyabet empat medali emas dan satu perak Pelajar yang menyumbang emas adalah Muhammad Sohibul Maromi (SMAN 1 Pamekasan Madura) Christian George Emor (SMA Lokon St Nikolaus Tomohon Sulawesi Utara) David Giovanni (SMAK Penabur Gading Serpong Banten) dan Kevin Soedyatmiko (SMAN 12 Jakarta) Sedangkan medali perak berhasil diraih oleh Ahmad Ataka Awwalur Rizqi (SMAN 1 Yogyakarta) Prestasi ini jauh lebih baik dibanding ajang Olimpiade Fisika ke-40 di Merida Yucatan Meksiko 2009 yang lalu Saat itu delegasi siswa Indonesia merebut satu medali emas dua medali perak dan satu perunggu Hasil empat medali emas dan satu perak ini hampir menyamai prestasi terbaik sebelumnya pada ajang Olimpiade Fisika ke-37 di Singapura Saat itu siswa Indonesia tidak hanya berhasil menyabet 4 medali emas namun juga meraih predikat `Absolute Winner` atas nama Mailoa Jonathan Pradana (SMAK 1 BPK Penabur Jakarta) Tapi yang terpenting lagi pelajar Indonesia berhasil mempertahankan tradisi emas di setiap ajang Olimpiade Fisika Red Endro Yuwanto Lima siswa Indonesia berhasil mendapatkan a 4 emas 1 perak

b 1 emas 4 perak c 3 emas 2 perak d 2 perak 3 emas

Beranda Soal UKG Agus Zainal M (SDN Gudang 2)

Republik_ilalangyahoocoid

Uraian 33 KD 134 menemukan pesan pokok dalam wacana naratif seperti cerita rakyat puisi

Surat dari ibu Jika bayang telah pudar Dan elang laut pulang ke sarang Angin bertiup ke benua Tiang ndash tiang akan kering sendiri Dan nahkoda sudah tau pedoman Boleh engkau datang padaku Makna lambang dari nahkoda sudah tahu pedoman adalahhellip a Sudah mencari pedoman hidup b Sudah menemukan arah dan tujuan

c Sudah berilmu dan berpengalaman d Sudah mempunyai pasangan hidup

Uraian 34 KD 135 membandingkan berbagai jenis wacana bahasa indonesia (deskripsi narasi)

Pendekatan untuk mendapat tanggapan emosional pembaca ataupun kesan pembaca adalah contoh Pendekatan Deskripsi jenis a Pendekatan Ekspositoris b Pendekatan Impresionistik

c Pendekatan menurut sikap pengarang d Pendekatan Realistik

Uraian PENDEKATAN DESKRIPSI

Pendekatan dalam pendeskripsian dapat dibedakan menjadi beberapa kategori pendekatan yaitu

1 Pendekatan Ekspositoris

Dalam pendekatan ini kita berusaha agar deskripsi yang kita buat dapat memberi keterangan sesuai dengan keadan yang

sebenarnya sehingga pembaca dapat seolah-olah ikut melihat atau merasakan objek yang kita deskripsikan Karangan

jenis ini berisi daftar detail sesuatu secara lengkap sehingga pembaca dan penalarannya dapat memperoleh kesan

keseluruhan tentang sesuatu

2 Pendekatan Impresionistik

Tujuan deskripsi impresionistik ialah untuk mendapatkan tanggapan emosional pembaca ataupun kesan pembaca Corak

deskripsi ini diantaranya juga ditentukan oleh macam kesan apa yang diinginkan penulisnya

3 Pendekatan menurut Sikap Pengarang

Pendekatan ini sangat bergantung pada tujuan yang ingin dicapai sifat objek serta pembaca deskripsinya Dalam

menguraikan sebuah gagasan penulis mungkin mengharapkan agar pembaca merasa tidak puas terhadapa suatu

tindakan atau keadaan atau penulis menginginkan agar pembaca juga harus merasakan bahwa persoalan yang dihadapi

merupakan masalah yang gawat Penulis juga dapat membayangkan bahwa akan terjadi sesuatu yang tidak diinginkan

sehingga pembaca dari semula sudah disiapkan perasaan yang kurang enak seram takut dan sebagainya (Alkhaidah

1997) 35 kd 135 membandingkan berbagai jenis wacana bahasa indonesia (deskripsi narasi)

Prinsip-Prinsip karangan Narasi sebagai berikut kecuali a Alur b Penokohan c Amanat

d Sudut Pandang Uraian Prinsip-prinsip dasar narasi sebagai tumpuan berfikir adalah sebagai berikut

1 Alur (plot) biasa disebut dengan jalan cerita

2 Penokohan (rangkaian perbuatanaksi) 3 Latar (setting) yaitu tempat dan atau waktu terjadinya perbuatan tokoh atau peristiwa yang dialami tokoh 4 Sudut Pandang (Point of View)

a Narrator Serba Tahu (Omniscient point of view) Yaitu narrator bertindak sebagai pencipta segalanya b Narrator bertindak objektif (Objektive point of view) Pengarang tak member komentar apa pun dan pembaca bebas menafsirkan apa yang diceritakan pengarang c Narrator ikut aktif ( Narator acting) Narrator terlibat dalam cerita d Narrator sebagai peninjau

Beranda Soal UKG Agus Zainal M (SDN Gudang 2)

Republik_ilalangyahoocoid

Pelakunya adalah orang ketiga yang biasa disebut ldquodiardquo 36 KD 135 membandingkan berbagai jenis wacana bahasa indonesia (deskripsi narasi)

1 Menentukan tema atau amanat apa yang akan disampaikan 2 Menetapkan sasaran pembaca 3 Merancang peristiwa-peristiwa utama yang akan ditampilkan dalam bentuk skema perkembangandan akhir cerita 5 Memerinci peristiwa-peristiwa utama ke dalam detail-detail peristiwa sebagai pendukung cerita 6 Menyusun tokoh dan perwatakanserta latar dan sudut pandang Karakteristik diatas adalah pengembangan dari paragraf a Narasi

b Deskprisi c Ekspositoris d Sudut Pandang Persuasif

Uraian Pengembangan karangan narasi dapat dilakukan dengan langkah-langkah berikut

(1) menentukan tema atau amanat apa yang akan disampaikan (2) menetapkan sasaran pembaca (3) merancang peristiwa-peristiwa utama yang akan ditampilkan dalam bentuk skema alur (4) membagi peristiwa utama ke dalam ke dalam bagian awal perkembangan dan akhir cerita (5) memerinci peristiwa-peristiwa utama ke dalam detail-detail peristiwa sebagai pendukung cerita dan (6) menyusun tokoh dan perwatakan latar dan sudut pandang

37 KD 135 membandingkan berbagai jenis wacana bahasa indonesia (deskripsi narasi) Karangan narasi yang berusaha untuk memberikan suatu maksud tertentu menyampaikan suatu amanat terselubung kepada para pembaca atau pendengar sehingga tampak seolah-olah melihat disebut a Narasi Informatif b Narasi Ekspositoris c Narasi Sugestif d Narasi Artistik

Uraian Jenis-Jenis Narasi

Narasi Informatif Narasi informatif adalah narasi yang memiliki sasaran penyampaian informasi secara tepat tentang suatu peristiwa

dengan tujuan memperluas pengetahuan orang tentang kisah seseorang Narasi Ekspositorik

Narasi ekspositorik adalah narasi yang memiliki sasaran penyampaian informasi secara tepat tentang suatu peristiwa dengan tujuan memperluas pengetahuan orang tentang kisah seseorang Dalam narasi ekspositorik penulis menceritakan suatu peristiwa berdasarkan data yang sebenarnya Pelaku yang ditonjolkan biasanya satu orang Pelaku diceritakan mulai dari kecil sampai saat ini atau sampai terakhir dalam kehidupannya Karangan narasi ini diwarnai oleh eksposisi maka ketentuan eksposisi juga berlaku pada penulisan narasi ekspositprik Ketentuan ini berkaitan dengan penggunaan bahasa yang logis berdasarkan fakta yang ada tidak memasukan unsursugestif atau bersifat objektif

Narasi Artistik

Narasi artistik adalah narasi yang berusaha untuk memberikan suatu maksud tertentu menyampaikan suatu amanat terselubung kepada para pembaca atau pendengar sehingga tampak seolah-olah melihat Ketentuan ini berkaitan dengan penggunaan bahasa yang logis berdasarkan fakta yang ada tidak memasukan unsur sugestif atau bersifat objektif

Narasi Sugestif Narasi sugestif adalah narasi yang berusaha untuk memberikan suatu maksud tertentu menyampaikan suatu amanat

terselubung kepada para pembaca atau pendengar sehingga tampak seolah-olah melihat 38 KD 135 membandingkan berbagai jenis wacana bahasa indonesia (deskripsi narasi)

Ciri karangan Narasi yang benar kecuali a Menonjolkan unsur perbuatan atau tindakan Membuat pembaca atau pendengar merasakan sendiri atau

mengalami sendiri

b Ada konfiks menjawab pertanyaan apa yang terjadi c Dirangkai dalam urutan waktu menggambarkan dengan jelas suatu peristiwa d Berisi ajakan dirangkai dalam urutan waktu

Uraian Ciri-Ciri Karangan Narasi Menurut Gorys Keraf (2000136)

Menonjolkan unsur perbuatan atau tindakan Dirangkai dalam urutan waktu Berusaha menjawab pertanyaan apa yang terjadi Ada konfiks

39 kd 135 membandingkan berbagai jenis wacana bahasa indonesia (deskripsi narasi) Langkah menyusun paragraf deskripsi yang benar adalah (1)Tentukan objek atau tema yang akan dideskripsikan (2)Mengumpulkan data dengan mengamati objek yang akan dideskripsikan (3)Menyusun data tersebut ke dalam urutan yang baik (menyusun kerangka karangan) (4)Menguraikan kerangka karangan menjadi dekripsi yang sesuai dengan tema yang ditentukan

Beranda Soal UKG Agus Zainal M (SDN Gudang 2)

Republik_ilalangyahoocoid

(5)Tentukan tujuan a 1-5-2-3-4

b 1-5-4-3-2 c 1-5-3-2-4 d 1-3-4-2-5

Uraian Langkah menyusun deskripsi

1Tentukan objek atau tema yang akan dideskripsikan 2Tentukan tujuan 3Mengumpulkan data dengan mengamati objek yang akan dideskripsikan 4Menyusun data tersebut ke dalam urutan yang baik (menyusun kerangka karangan) 5Menguraikan kerangka karangan menjadi dekripsi yang sesuai dengan tema yang ditentukan

40 Kd 311 Merancang aktivitas pembelajaran berdasarkan prinsip dan teori pembelajaran matematika Dalam tahap ini penyajian yang dilakukan melalui tindakan anak secara langsung terlihat dalam memanipulasi (mengotak atik)objek Proses tersebut menurut Bruner dikategorikan ke dalam model a Tahap Enaktif

b Tahap Ikonik c Tahap Simbolik d Tahap Implikatif

Uraian 1 Model Tahap Enaktif

Dalam tahap ini penyajian yang dilakukan melalui tindakan anak secara langsung terlihat dalam memanipulasi (mengotak atik)objek Contoh Budi mempunyai 2 pensil kemudian ibunya memberikannya lagi 3 pinsil Berapa banyak pensil Budi sekarang

2 Model Tahap Ikonik Dalam tahap ini kegiatan penyajian dilakukan berdasarkan pada pikiran internal dimana pengetahuan disajikan melalui serangkaian gambar-gambar atau grafik yang dilakukan anak berhubungan dengan mental yang merupakan gambaran dari objek-objek yang dimanipulasinya Contoh + = hellip

3 Model Tahap Simbolis Dalam tahap ini bahasa adalah pola dasar simbolik anak memanipulasi Simbol-simbol atau lambang-lambang objek tertentu Contoh 2 pensil + 3 pensil = hellip pensil

41 Kd 311 Merancang aktivitas pembelajaran berdasarkan prinsip dan teori pembelajaran matematika Dalam tahap ini kegiatan penyajian dilakukan berdasarkan pada pikiran internal dimana pengetahuan disajikan melalui serangkaian gambar-gambar atau grafik yang dilakukan anak Proses tersebut menurut Bruner dikategorikan ke dalam model a Tahap Enaktif b Tahap Ikonik

c Tahap Simbolik d Tahap Implikatif

Uraian No 40 42 Kd 311 Merancang aktivitas pembelajaran berdasarkan prinsip dan teori pembelajaran matematika

Objek belajar matematika dibagi kedalam Objek Langsung dan Objek Tak Langsung Hal tersebut adalah teori belajar Matematika menurut a Robert M Gagne

b Jerome S Burner c Thorndike d Skinner

Uraian Teori yang diperkenalkan Robert MGagne pada tahun 1960-an pembelajaran harus dikondisikan untuk memunculkan

respons yang diharapkanMenurut Gagne (dalam Ismail 1998) belajar matematika terdiri dari objek langsung dan objek tak langsung

43 Kd 311 Merancang aktivitas pembelajaran berdasarkan prinsip dan teori pembelajaran matematika Perhatikan contoh berikut Budi mempunyai 2 pinsil kemudian ibunya memberikannya lagi 3 pinsilBerapa banyak pinsil Budi sekarang Hal tersebut dikemukakan Bruner dalam Proses Pembelajaran Matematika dalam tahap a Simbolik b Ikonik c Implikatif d Enaktif

Uraian No 40

Beranda Soal UKG Agus Zainal M (SDN Gudang 2)

Republik_ilalangyahoocoid

44 Kd 311 Merancang aktivitas pembelajaran berdasarkan prinsip dan teori pembelajaran matematika Perhatikan gambar disamping Ilustrasi di samping dikemukakan Bruner dalam Proses Pembelajaran Matematika dalam tahap

a Simbolik b Ikonik

c Implikatif d Enaktif

Uraian No 40 45 Kd 311 Merancang aktivitas pembelajaran berdasarkan prinsip dan teori pembelajaran matematika

Perhatikan contoh berikut Contoh 2 pinsil + 3 pinsil = hellippinsil Contoh tersebut dikemukakan Bruner dalam Proses Pembelajaran Matematika dalam tahap a Simbolik

b Ikonik c Implikatif d Enaktif

Uraian No 40 46 Kd 311 Merancang aktivitas pembelajaran berdasarkan prinsip dan teori pembelajaran matematika

Perhatikan contoh berikut penjumlahan bilangan positif dan negatif siswa mencoba sendiri dengan menggunakan garis bilangan Contoh tersebut dikemukakan Bruner dalam Teorema Pembelajaran Matematika a Penyusunan

b Notasi c Pengkontrasan dan Keanekaragaman d Pengaitan

Uraian Berdasarkan hasil pengamatannya Brunner merumuskan 5 teorema dalam pembelajaran matematika yaitu

1) Teorema Penyusunan Menerangkan bahwa cara yang terbaik memulai belajar suatu konsep matematika dalil defenisi dan semacamnya adalah dengan cara menyusun penyajiannya Misalnya dalam mempelajari penjumlahan bilangan positif dan negatif siswa mencoba sendiri dengan menggunakan garis bilangan 2) Teorema Notasi Menerangkan bahwa dalam pengajaran suatu konsep penggunaan notasi-notasi matematika harus diberikan secara bertahap dari yang sederhana ke yang lebih kompleks 3) Teorema Pengkontrasan dan Keanekaragaman Menerangkan bahwa pengontrasan dan keanekaragaman sangat penting dalam melakukan pengubahan konsep matematika dari yang konkrit ke yang lebih abstrak Dalam hal ini diperlukan banyak contoh Contoh yang diberikan harus sesuai dengan rumusan yang diberikan Misalnya menjelaskan persegi panjang disertai juga kemungkinan jajaran genjang dan segi empat lainnya selain persegi panjnag Dengan demikian siswa dapat membedakan apakah segi empat yang diberikan padanya termasuk persegi panjang atau tidak 4) Teorema Pengaitan Menerangkan bahwa dalam matematika terdapat hubungan yang berkaitan antara satu konsep dengan konsep yang lain Di mana materi yang satu merupakan prasyarat yang harus diketahui untuk mempelajari materi yang lain

47 Kd 311 Merancang aktivitas pembelajaran berdasarkan prinsip dan teori pembelajaran matematika Perhatikan contoh berikut Guru menjelaskan persegi panjang disertai juga kemungkinan jajaran genjang dan segi empat lainnya selain persegi panjnag Dengan demikian siswa dapat membedakan apakah segi empat yang diberikan padanya termasuk persegi panjang atau tidak Contoh tersebut dikemukakan Bruner dalam Teorema Pembelajaran Matematika a Penyusunan b Notasi c Pengkontrasan dan Keanekaragaman

d Pengaitan Uraian No 46 48 Kd 311 Merancang aktivitas pembelajaran berdasarkan prinsip dan teori pembelajaran matematika

Tahap pembelajaran Matematika menurut Van Halle adalah sebagai berikut kecuali a Tahap Pengenalan b Tahap Pengurutan c Tahap Analisis d Tahap Penyimpulan

Beranda Soal UKG Agus Zainal M (SDN Gudang 2)

Republik_ilalangyahoocoid

Uraian Van Hiele menyatakan bahwa terdapat 5 tahap belajar siswa dalam belajar geometri yaitu

a Tahap Pengenalan Pada tahap ini siswa mulai belajar mengenal suatu bangun geometri secara keseluruhan namun belum mampu mengetahui adanya sifat-sifat dari bangun geometri yang dilihatnya b Tahap Analisis Pada tahap ini siswa sudah mulai mengenal sifat-sifat yang dimiliki bangun geometri yang diamatinya c Tahap Pengurutan Pada tahap ini siswa sudah mengenal dan memahami sifat-sifat suatu bangun geometri serta sudah dapat mengurutkan bangun-bangun geometri yang satu sama yang lainnya saling berhubungan d Tahap Deduksi Pada tahap ini siswa telah mampu menarik kesimpulan secara deduktif yaitu menarik kesimpulan yang bersifat umum dan menuju ke hal yang bersifat khusus serta dapat mengambil kesimpulan e Tahap Akurasi Pada tahap ini siswa mulai menyadari pentingnya ketepatan prinsip-prinsip dasar yang melandasi suatu pembuktian Tahap berfikir ini merupakan tahap berfikir yang paling tinggi rumit dan kompleks karena di luar jangkauan usia anak-anak SD sampai tingakat SMP

49 Kd 311 Merancang aktivitas pembelajaran berdasarkan prinsip dan teori pembelajaran matematika middot Matematika untuk tujuan pembelajaran dianalisis sebagai kumpulan fakta yang berdiri sendiri dan tidak saling berkaitan middot Anak diharuskan menguasai unsur-unsur yang banyak sekali tanpa diperhatikan pengertiannya middot Anak mempelajari unsur-unsur dalam bentuk seperti yang akan digunakan nanti dalam kesempatan lain middot Anak akan mencapai tujuan ini secara efektif dan efisien dengan melalui pengulangan Teori Pembelajaran Matematika tersebut dikemukakan oleh a Skinner b Piaget c Van Brownell

d Thorndike Uraian Brownell mengemukakan tentang Teori Makna (Meaning Theory) sebagai pengganti Teori Latihan HafalUlangan (Drill

Theory) Intisari dari teori Drill adalah - Matematika untuk tujuan pembelajaran dianalisis sebagai kumpulan fakta yang berdiri sendiri dan tidak saling berkaitan - Anak diharuskan menguasai unsur-unsur yang banyak sekali tanpa diperhatikan pengertiannya - Anak mempelajari unsur-unsur dalam bentuk seperti yang akan digunakan nanti dalam kesempatan lain - Anak akan mencapai tujuan ini secara efektif dan efisien dengan melalui pengulangan

50 Kd 311 Merancang aktivitas pembelajaran berdasarkan prinsip dan teori pembelajaran matematika Tahap operasionaloperasi konkrit adalah teori Pembelajaran Matematika yang dikemukakan Peaget pada tahap usia a dibawah 2 tahun b 2-7 Tahun c 7-8 Tahun d 7-12 Tahun

Uraian Piaget membagi skema yang digunakan anak untuk memahami dunianya melalui empat periode utama yang berkorelasi

dengan dan semakin canggih seiring pertambahan usia Periode sensorimotor (usia 0ndash2 tahun) Periode praoperasional (usia 2ndash7 tahun) Periode operasional konkrit (usia 7ndash12 tahun) Periode operasional formal (usia 12 tahun sampai dewasa)

51 313 Memilih media pembelajaran yang tepat untuk pembelajaran operasi bilangan bulat Untuk operasi bilangan bulat perkalian media yang paling tepat digunakan adalah a sapu lidi

b kerikil c daun pakis d koin

Uraian 52 313 Memilih media pembelajaran yang tepat untuk pembelajaran operasi bilangan bulat

Untuk operasi bilangan bulat negatif kita bisa menggunakan media a uang kertas b kerikil c mistar d koin

Uraian 53 313 Memilih media pembelajaran yang tepat untuk pembelajaran operasi bilangan bulat

Beranda Soal UKG Agus Zainal M (SDN Gudang 2)

Republik_ilalangyahoocoid

Untuk operasi bilangan bulat penjumlahan media yang paling tepat digunakan adalah a uang kertas b kerikil c daun pakis d koin

Uraian 54 314 Memilih media pembelajaran yang tepat untuk pembelajaran operasi bilangan pecahan

Untuk operasi bilangan pecahan kita bisa menggunakan media berikut kecuali a kartu bilangan b garis bilangan c gambar bidang d blok pecahan

Uraian 55 316 Memilih media pembelajaran yang tepat untuk pembelajaran geometri dan pengukuran

Untuk menghitung luas dan keliling bangun datar media yang paling tepat digunakan adalah a Penggaris b Kertas Folio Bergaris

c Neraca d Kalkulator

Uraian

Page 5: 50 SOAL UKG & PEMBAHASAN SESUAI KISI-KISIS.pdf

Beranda Soal UKG Agus Zainal M (SDN Gudang 2)

Republik_ilalangyahoocoid

17 KD 121 Memilih berbagai Metode Menulis Permulaan 1) Mengenalkan kata 2) Merangkaikan kata antar suku kata 3) Menguraikan suku kata atas huruf-hurufnya 4) Menggabungkan huruf menjadi kata (Djauzak 19965) Tahapan diatas adalah Teknik Menulis Permulaan dengan metode a Metode Eja b Metode Kata Lembaga

c Metode SAS d Metode Global

Uraian No 16 18 KD 121 Memilih berbagai Metode Menulis Permulaan

Metode dengan memulai pengajaran membaca dan menulis permulaan dengan membaca kalimat secara utuh yang ada di bawah gambar Menguraikan kalimat dengan kata-kata menguraikan kata-kata menjadi suku kata disebut dengan metode a Metode Eja b Metode Kata Lembaga c Metode SAS d Metode Global

Uraian No 16 19 KD 121 Memilih berbagai Metode Menulis Permulaan

Suatu pembelajaran menulis permulaan yang didasarkan atas pendekatan cerita yakni cara memulai mengajar menulis dengan menampil cerita yang diambil dari dialog siswa dan guru atau siswa dengan siswa disebut dengan metode a Metode Eja b Metode Kata Lembaga c Metode SAS

d Metode Global Uraian No 16 20 KD 121 Memilih berbagai Metode Menulis Permulaan

menulis kartu huruf kartu suku kata kartu kata dan kartu kalimat sementara sebagian siswa mencari huruf suku kata dan kata guru dan sebagian siswa menempel kata-kata yang tersusun sehingga menjadi kalimat yang berarti adalah contoh metode a Metode Eja b Metode Kata Lembaga c Metode SAS

d Metode Global Uraian No 16 21 KD 122 Merancang berbagai kegiatan menulis di kelas tinggi

Berikut adalah kegiatan menulis lanjutan di kelas tinggi kecuali a menulis tentang berbagai topik b menulis pengumuman c menulis pantun d menulis memo

Uraian 22 KD 123 Perencanaan dan Pelaksanaan evaluasi pembelajaran Bahasa dan Sastra Indonesia

Perencanaan Pengajaran meliputi hal-hal berikut kecuali a tujuan apa yang hendak dicapai b memilih bahan ajar c proses belajar mengajar d alat penilaian

Uraian Menurut Akhlan dan Rahman (199715) perencanaan pengajaran meliputi

a tujuan apa yang hendak dicapai b bahan pengajaran c proses belajar mengajardan d alat penilaian

23 KD 123 Perencanaan dan Pelaksanaan evaluasi pembelajaran Bahasa dan Sastra Indonesia karakteristik perencanaan pengajaran yang baik hendaknya mengandung prinsip sebagai berikut a Memiliki sikap objektif rasio (tepat dan masuk akal) komprehensif dan sistematis (menyeluruh dan tersusun rapi)

b Merupakan suatu wahana atau wadah untuk mengembangkan segala potensi yang ada dan dimiliki oleh anak

Beranda Soal UKG Agus Zainal M (SDN Gudang 2)

Republik_ilalangyahoocoid

didik c Mengendalikan kekuatan sendiri bukan didasarkan atas kekuatan orang lain d Melakukan studi kasus yang berkesinambungan

Uraian Menurut Akhlan dan Rahman (19977) karakteristik perencanaan pengajaran yang baik hendaknya mengandung prinsip

sebagai berikut a Mengembangkan hubungan interaksi yang baik di antara sesama manusia dalam hal ini siswa dan guru serta personal

terkait b Merupakan suatu wahana atau wadah untuk mengembangkan segala potensi yang ada dan dimiliki oleh anak didik c Memiliki sikap objektif rasio (tepat dan masuk akal) komprehensif dan sistematis (menyeluruh dan tersusun rapi) d Mengendalikan kekuatan sendiri bukan didasarkan atas kekuatan orang lain Didukung oleh fakta dan data yang

menunjang pencapaian tujuan yang telah di dirumuskan e Fleksibel dan dinamis artinya mudah disesuaikan dengan keadaan serta perkembangan ke arah yang lebih baik dan

maju 24 KD 123 Perencanaan dan Pelaksanaan evaluasi pembelajaran Bahasa dan Sastra Indonesia

tahap mengumpulkan informasi tentang keadaan objek evaluasi (siswa) dengan menggunakan teknik tes atau nontes disebut tahapan a Tahap Tindak Lanjut b Tahap Persiapan c Tahap Pelaksanaan

d Tahap Pengolahan Hasil Uraian Berikut ini penjelasan singkat tentang keempat tahap evaluasi pembelajaran tersebut

(1) Tahap Persiapan Menurut Damaianti (2007 8) tahap ini disebut juga tahap perencanaan dan perumusan kriterium Langkahnya meliputi (a) perumusan tujuan evaluasi (b) penetapan aspek-aspek yang akan dievaluasi (c) menetapkan metode dan bentuk evaluasi (tesnontes) (d) merencanakan waktu evaluasi (e) melakukan uji coba (untuk tes) agar dapat mengukur validitas dan reliabilitasnya Untuk evaluasi yang menggunakan tes hasil dari tahap ini adalah kisi-kisi soal dan seperangkat alat tes soal lembar jawaban (untuk tes tulis) kunci jawaban dan pedoman penilaian 2) Tahap Pelaksanaan Tahap pelaksanaan atau disebut juga dengan tahap pengukuran dan pengumpulan data adalah tahap untuk mengumpulkan informasi tentang keadaan objek evaluasi (siswa) dengan menggunakan teknik tes atau nontes Bila menggunakan teknik tes soal yang igunakan sebaiknya sudah teruji validitas dan reliabilitasnya Tes yang digunakan dapat berbentuk tes tulis lisan atau praktik 3) Tahap Pengolahan Hasil Tahap pengolahan hasil adalah tahap pemeriksaan hasil evaluasi dengan memberikan skor Skor yang diperoleh siswa selanjutnya diubah menjadi nilai Pada tes tulis pemeriksaan hasil dilakukan setelah tes selesai sedangkan pada tes lisan dan praktik pemberian nilai dilakukan bersamaan dengan waktu pelasanaan tes tersebut 4) Tahap Tindak Lanjut Tahap tindak lanjut atau disebut juga tahap penafsiran adalah tahap untuk mengambil keputusan berdasarkan nilai yang dihasilkan pada tahap pengolahan hasil misalnya a memperbaiki proses belajar mengajar b memperbaiki kesulitan belajar siswa c memperbaiki alat evaluasi d membuat laporan evaluasi (rapor)

25 KD 131 Merumuskan hakikat (pengertiantujuanjenismanfaat) membaca membaca yang mengutamakan isi bacaan sebagai ungkapan pikiran perasaan dan kehendak penulis Bila hanya ingin mengetahui isinya membaca cerdas bersifat lugas Akan tetapi bila maksudnya untuk memahami dan memilki isi bacaan maka tergolong kedalam membaca jenis a Membaca cerdas atau membaca dalam hati

b Membaca bahasa c Membaca teknis d Membaca bebas

Uraian Menurut Soedjono dalam Sue (200418-21) ada lima macam membaca yaitu membaca bahasa membaca cerdas atau

membaca dalam hati membaca teknis membaca emosional dan membaca bebas 1) Membaca bahasa Membaca bahasa adalah membaca yang mengutamakan bahasa bacaan Membaca bahasa mementingkan segi bahasa bacaan Hal-hal yang perlu diperhatikan dalam membaca bahasa adalah kesesuian pikir dengan bahasa perbendaharaan bahasa yang meliputi kosa kata struktur kalimat dan ejaan 2) Membaca cerdas atau membaca dalam hati

Beranda Soal UKG Agus Zainal M (SDN Gudang 2)

Republik_ilalangyahoocoid

Membaca cerdas adalah membaca yang mengutamakan isi bacaan sebagai ungkapan pikiran perasaan dan kehendak penulis Bila hanya ingin mengetahui isinya membaca cerdas bersifat lugas Akan tetapi bial maksudnya untuk memahami dan memilki isi bacaan maka disebut membaca belajar 3) Membaca teknis Membaca teknis adalah membaca dengan mengarahkan bacaan secara wajar Wajar maksudnya sesuai ucapan tekanan dan intonasinya Pikiran perasaan dan kemauan yang tersimpan dalam bacaan dapat diaktualisasikan dengan baik 4) Membaca emosional Membaca emosional adalah membaca sebagai sarana untuk memasuki perasaan yaitu keindahan isi dan keindahan bahasanya 5) Membaca bebas Membaca bebas adalah membaca sesuatu atas kehendak sendiri tanpa adanya unsur paksaan dari luar Unsur dari luar misalnya guru orang tua teman atau pihak-pihak lain

26 KD 131 Merumuskan hakikat (pengertiantujuanjenismanfaat) membaca membaca sesuatu atas kehendak sendiri tanpa adanya unsur paksaan dari luar Unsur dari luar misalnya guru orang tua teman atau pihak-pihak lain termasuk jenis membaca a Membaca cerdas atau membaca dalam hati b Membaca bahasa c Membaca teknis d Membaca bebas

Uraian No 25 27 KD 131 Merumuskan hakikat (pengertiantujuanjenismanfaat) membaca

1 menggunakan ucapan yang tepat 2 menggunakan frase yang tepat 3 menggunakan intonasi suara yang wajar 4 dalam posisi sikap yang baik 5 menguasai tanda-tanda baca 6 membaca dengan terang dan jelas keterampilan di atas harus dipunyai dalam jenis membaca a Membaca cerdas atau membaca dalam hati b Membaca bahasa c Membaca teknis d Membaca nyaring

Uraian JENIS-JENIS MEMBACA DAN KARAKTERISTIKNYA

Ditinjau dari segi terdengar atau tidaknya suara pembaca waktu melakukan kegiatan membaca maka proses membaca dapat dibedakan menjadi A Membaca Nyaring Membaca nyaring adalah kegiatan membaca dengan menyuarakan tulisan yang dibacanya dengan ucapan dan intonasi yang tepat agar pendengar dan pembaca dapat menangkap informasi yang disampaikan oleh penulis baik yang berupa pikiran perasaan sikap ataupun pengalaman penulis Ketrampilan yang dituntut dalam membaca nyaring adalah berbagai kemampuan diantaranya adalah 1 menggunakan ucapan yang tepat 2 menggunakan frase yang tepat 3 menggunakan intonasi suara yang wajar 4 dalam posisi sikap yang baik 5 menguasai tanda-tanda baca 6 membaca dengan terang dan jelas 7 membaca dengan penuh perasaan ekspresif 8 membaca dengan tidak terbata-bata 9 mengerti serta memahami bahan bacaan yang dibacanya 10 kecepatan bergantung pada bahan bacaan yang dibacanya 11 membaca dengan tanpa terus-menerus melihat bahan bacaan 12 membaca dengan penuh kepercayaan pada diri sendiri B Membaca Dalam Hati Membaca dalam hati adalah kegiatan membaca yang dilakukan dengan tanpa menyuarakan isi bacaan yang dibacanya Ketrampilan yang dituntut dalam membaca dalam hati antara lain sebagai berikut 1 membaca tanpa bersuara tanpa bibir bergerak tanpa ada desis apapun 2 membaca tanpa ada gerakan-gerakan kepala 3 membaca lebih cepat dibandingkan dengan membaca nyaring 4 tanpa menggunakan jari atau alat lain sebagai penunjuk 5 mengerti dan memahami bahan bacaan 6 dituntut kecepatan mata dalam membaca 7 membaca dengan pemahaman yang baik

Beranda Soal UKG Agus Zainal M (SDN Gudang 2)

Republik_ilalangyahoocoid

8 dapat menyesuaikan kecepatan dengan tingkat kesukaran yang terdapat dalam bacaan 28 KD 131 Merumuskan hakikat (pengertiantujuanjenismanfaat) membaca

Hal berikut dilakukan seseorang ketika membaca survai kecuali a memeriksa judul bacaanbuku kata pengantar daftar isi dan malihat abstrak(jika ada) b memeriksa bagian terahkir dari isi (kesimpulan) jika ada c memeriksa indeks dan apendiks(jika ada) d membaca biografi pengarang

Uraian Secara garis besar membaca dalam hati dapat dibedakan menjadi dua (I) MEMBACA EKSTENSIF amp (II) MEMBACA

INTENSIF Berikut penjelasan secara rinci kedua jenis membaca tersebut I Membaca Ekstensif membaca ekstensif adalah membaca secara luas Objeknya meliputi sebanyak mungkin teks dalam waktu yang sesingkat-singkatnya Membaca ekstensif meliputi 1 Membaca Survai (Survey Reading) Membaca survai adalah kegiatan membaca untuk mengetahui secara sekilas terhadap bahan bacaan yang akan dibaca lebih mendalam Kegiatan membaca survai merupakan pendahuluan dalam membaca ekstensif Yang dilakukan seseorang ketika membaca survai adalah sebagai berikut (a) memeriksa judul bacaanbuku kata pengantar daftar isi dan malihat abstrak(jika ada) (b) memeriksa bagian terahkir dari isi (kesimpulan) jika ada (c) memeriksa indeks dan apendiks(jika ada)

2 Membaca Sekilas Membaca sekilas atau membaca cepat adalah kegiatan membaca dengan mengandalakan kecepatan gerak mata dalam melihat dan memperhatikan bahan tertulis yang dibacanya dengan tujuan untuk mendapatkan informasi secara cepat Metode yang digunakan dalam melatihkan membaca cepat adalah (a) metode kosakata metode yang berusaha untuk menambah kosakata (b) Metode motivasi metode yang berusaha memotivasi pembaca(pemula) yang mengalami hambatan (c) Metode gerak mata metode yang mengembangkan kecepatan membaca dengan menigkatkan kecepatan gerak mata Hambatan-hambatan yang dapat mengurangi kecepatan mambaca (a) vokalisai atau berguman ketika membaca (b) membaca dengan menggerakan bibir tetapi tidak bersuara (c) kepala bergerak searah tulisan yang dibaca (d) subvokalisasi suara yang biasa ikut membaca di dalam pikiran kita (e) jari tangan selalu menunjuk tulisa yang sedang kit abaca (f) gerakan mata kembali pada kata-kata sebelumnya 3 Membaca Dangkal (Superficial Reading) membaca dangkal pada hakekatnya bertujuan untuk memperoleh pemahaman yang dangkal yang bersifat luaran yang tidak mendalam dari suatu bahan bacaan Membaca jenis ini biasanya dilakukan seseorang membaca demi kesenangan membaca bacaan ringan yang mendatangkan kesenangan kegembiraan sebagai pengisi waktu senggang

29 KD 131 Merumuskan hakikat (pengertiantujuanjenismanfaat) membaca Membaca jenis ini biasanya dilakukan seseorang membaca demi kesenangan membaca bacaan ringan yang mendatangkan kesenangan kegembiraan sebagai pengisi waktu senggang Berdasarkan karakteristik diatas kegiatan tersebut termasuk ke dalam membaca jenis a Membaca Survai (Survey Reading) b Membaca Sekilas c Membaca Dangkal (Superficial Reading)

d Membaca Nyaring Uraian No 28 30 KD 131 Merumuskan hakikat (pengertiantujuanjenismanfaat) membaca

Berikut adalah tujuan umum dalam aktifitas membaca kecuali a Membaca untuk memperoleh perincian-perincian atau fakta-fakta (reading for details or facts) Membaca tersebut

bertujuan untuk menemukan atau mengetahui penemuan-penemuan telah dilakukan oleh sang tokoh untuk memecahkan masalah-masalah yang dibuat oleh sang tokoh

b Membaca untuk mengetahui ukuran atau susunan organisasi cerita (reading for sequenceor organization) Membaca tersebut bertujuan untuk mengetahui bagian-bagian cerita dan hubungan antar bagian-bagian cerita

c Membaca untuk menyimpulkan atau membaca inferensi (reading for inference) d Membaca untuk memperoleh kekurangan suatu buku (finding mistakes)

Uraian Tujuan Membaca

Beranda Soal UKG Agus Zainal M (SDN Gudang 2)

Republik_ilalangyahoocoid

Berdasarkan maksud tujuan atau keintensifan serta cara dalam membaca di bawah ini Anderson dalam Tarigan (19799-10) mengemukakan beberapa tujuan membaca antara lain a Membaca untuk memperoleh perincian-perincian atau fakta-fakta (reading for details or facts) Membaca tersebut bertujuan untuk menemukan atau mengetahui penemuan-penemuan telah dilakukan oleh sang tokoh untuk memecahkan masalah-masalah yang dibuat oleh sang tokoh b Membaca untuk memperoleh ide-ide utama (reading for main ideas) Membaca untuk mengetahui topik atau masalah dalam bacaan Untuk menemukan ide pokok bacaan dengan membaca halamn demi halaman c Membaca untuk mengetahui ukuran atau susunan organisasi cerita (reading for sequenceor organization) Membaca tersebut bertujuan untuk mengetahui bagian-bagian cerita dan hubungan antar bagian-bagian cerita d Membaca untuk menyimpulkan atau membaca inferensi (reading for inference) Pembaca diharapkan dapat merasakan sesuatu yang dirasakan penulis e Membaca untuk mengelompokkan atau mengklasifikasikan (reading for classify) Membaca jenis ini bertujuan untuk menemukan hal-hal yang tidak wajar mengenai sesuatu hal (Anderson dalam Tarigan 197910) f Membaca untuk menilai atau mengevaluasai (reading to evaluate) Jenis membaca tersebut bertujuan menemukan suatu keberhasilan berdasarkan ukuran-ukuran tertentu Membaca jenis ini memerlukan ketelitian dengan membandingkan dan mengujinya kembali g Membaca untuk memperbandingkan atau mempertentangkan (reading to compare or contrast) Tujuan membaca tersebut adalah untuk menemukan bagaimana cara perbedaan atau persamaan dua hal atau lebih

31 KD 133 menemukan pesan pokok utama sebuah berita Simak penggalan berita berikut Hebat Siswa Indonesia Pertahankan Tradisi Emas di Olimpiade Fisika Tim Olimpiade Fisika Indonesia REPUBLIKACOID JAKARTA--Luar biasa Lima siswa Indonesia yang dikirim ke ajang Olimpiade Fisika atau International Physics Olympiad (IPhO) ke-41 di Zagreb Kroasia 17-25 Juli berhasil menyabet empat medali emas dan satu perak Pelajar yang menyumbang emas adalah Muhammad Sohibul Maromi (SMAN 1 Pamekasan Madura) Christian George Emor (SMA Lokon St Nikolaus Tomohon Sulawesi Utara) David Giovanni (SMAK Penabur Gading Serpong Banten) dan Kevin Soedyatmiko (SMAN 12 Jakarta) Sedangkan medali perak berhasil diraih oleh Ahmad Ataka Awwalur Rizqi (SMAN 1 Yogyakarta) Prestasi ini jauh lebih baik dibanding ajang Olimpiade Fisika ke-40 di Merida Yucatan Meksiko 2009 yang lalu Saat itu delegasi siswa Indonesia merebut satu medali emas dua medali perak dan satu perunggu Hasil empat medali emas dan satu perak ini hampir menyamai prestasi terbaik sebelumnya pada ajang Olimpiade Fisika ke-37 di Singapura Saat itu siswa Indonesia tidak hanya berhasil menyabet 4 medali emas namun juga meraih predikat `Absolute Winner` atas nama Mailoa Jonathan Pradana (SMAK 1 BPK Penabur Jakarta) Tapi yang terpenting lagi pelajar Indonesia berhasil mempertahankan tradisi emas di setiap ajang Olimpiade Fisika Red Endro Yuwanto Peristiwa yang diberitakan adalah a siswa Indonesia pertahankan tradisi emas di Olimpiade Matematika b siswa Indonesia pertahankan tradisi emas di Olimpiade Fisika c siswa Indonesia menyumbang emas di Olimpiade Matematika

d siswa Indonesia menyumbang perak di Olimpiade Fisika Uraian 32 KD 133 menemukan pesan pokok utama sebuah berita

Simak penggalan berita berikut Hebat Siswa Indonesia Pertahankan Tradisi Emas di Olimpiade Fisika Tim Olimpiade Fisika Indonesia REPUBLIKACOID JAKARTA--Luar biasa Lima siswa Indonesia yang dikirim ke ajang Olimpiade Fisika atau International Physics Olympiad (IPhO) ke-41 di Zagreb Kroasia 17-25 Juli berhasil menyabet empat medali emas dan satu perak Pelajar yang menyumbang emas adalah Muhammad Sohibul Maromi (SMAN 1 Pamekasan Madura) Christian George Emor (SMA Lokon St Nikolaus Tomohon Sulawesi Utara) David Giovanni (SMAK Penabur Gading Serpong Banten) dan Kevin Soedyatmiko (SMAN 12 Jakarta) Sedangkan medali perak berhasil diraih oleh Ahmad Ataka Awwalur Rizqi (SMAN 1 Yogyakarta) Prestasi ini jauh lebih baik dibanding ajang Olimpiade Fisika ke-40 di Merida Yucatan Meksiko 2009 yang lalu Saat itu delegasi siswa Indonesia merebut satu medali emas dua medali perak dan satu perunggu Hasil empat medali emas dan satu perak ini hampir menyamai prestasi terbaik sebelumnya pada ajang Olimpiade Fisika ke-37 di Singapura Saat itu siswa Indonesia tidak hanya berhasil menyabet 4 medali emas namun juga meraih predikat `Absolute Winner` atas nama Mailoa Jonathan Pradana (SMAK 1 BPK Penabur Jakarta) Tapi yang terpenting lagi pelajar Indonesia berhasil mempertahankan tradisi emas di setiap ajang Olimpiade Fisika Red Endro Yuwanto Lima siswa Indonesia berhasil mendapatkan a 4 emas 1 perak

b 1 emas 4 perak c 3 emas 2 perak d 2 perak 3 emas

Beranda Soal UKG Agus Zainal M (SDN Gudang 2)

Republik_ilalangyahoocoid

Uraian 33 KD 134 menemukan pesan pokok dalam wacana naratif seperti cerita rakyat puisi

Surat dari ibu Jika bayang telah pudar Dan elang laut pulang ke sarang Angin bertiup ke benua Tiang ndash tiang akan kering sendiri Dan nahkoda sudah tau pedoman Boleh engkau datang padaku Makna lambang dari nahkoda sudah tahu pedoman adalahhellip a Sudah mencari pedoman hidup b Sudah menemukan arah dan tujuan

c Sudah berilmu dan berpengalaman d Sudah mempunyai pasangan hidup

Uraian 34 KD 135 membandingkan berbagai jenis wacana bahasa indonesia (deskripsi narasi)

Pendekatan untuk mendapat tanggapan emosional pembaca ataupun kesan pembaca adalah contoh Pendekatan Deskripsi jenis a Pendekatan Ekspositoris b Pendekatan Impresionistik

c Pendekatan menurut sikap pengarang d Pendekatan Realistik

Uraian PENDEKATAN DESKRIPSI

Pendekatan dalam pendeskripsian dapat dibedakan menjadi beberapa kategori pendekatan yaitu

1 Pendekatan Ekspositoris

Dalam pendekatan ini kita berusaha agar deskripsi yang kita buat dapat memberi keterangan sesuai dengan keadan yang

sebenarnya sehingga pembaca dapat seolah-olah ikut melihat atau merasakan objek yang kita deskripsikan Karangan

jenis ini berisi daftar detail sesuatu secara lengkap sehingga pembaca dan penalarannya dapat memperoleh kesan

keseluruhan tentang sesuatu

2 Pendekatan Impresionistik

Tujuan deskripsi impresionistik ialah untuk mendapatkan tanggapan emosional pembaca ataupun kesan pembaca Corak

deskripsi ini diantaranya juga ditentukan oleh macam kesan apa yang diinginkan penulisnya

3 Pendekatan menurut Sikap Pengarang

Pendekatan ini sangat bergantung pada tujuan yang ingin dicapai sifat objek serta pembaca deskripsinya Dalam

menguraikan sebuah gagasan penulis mungkin mengharapkan agar pembaca merasa tidak puas terhadapa suatu

tindakan atau keadaan atau penulis menginginkan agar pembaca juga harus merasakan bahwa persoalan yang dihadapi

merupakan masalah yang gawat Penulis juga dapat membayangkan bahwa akan terjadi sesuatu yang tidak diinginkan

sehingga pembaca dari semula sudah disiapkan perasaan yang kurang enak seram takut dan sebagainya (Alkhaidah

1997) 35 kd 135 membandingkan berbagai jenis wacana bahasa indonesia (deskripsi narasi)

Prinsip-Prinsip karangan Narasi sebagai berikut kecuali a Alur b Penokohan c Amanat

d Sudut Pandang Uraian Prinsip-prinsip dasar narasi sebagai tumpuan berfikir adalah sebagai berikut

1 Alur (plot) biasa disebut dengan jalan cerita

2 Penokohan (rangkaian perbuatanaksi) 3 Latar (setting) yaitu tempat dan atau waktu terjadinya perbuatan tokoh atau peristiwa yang dialami tokoh 4 Sudut Pandang (Point of View)

a Narrator Serba Tahu (Omniscient point of view) Yaitu narrator bertindak sebagai pencipta segalanya b Narrator bertindak objektif (Objektive point of view) Pengarang tak member komentar apa pun dan pembaca bebas menafsirkan apa yang diceritakan pengarang c Narrator ikut aktif ( Narator acting) Narrator terlibat dalam cerita d Narrator sebagai peninjau

Beranda Soal UKG Agus Zainal M (SDN Gudang 2)

Republik_ilalangyahoocoid

Pelakunya adalah orang ketiga yang biasa disebut ldquodiardquo 36 KD 135 membandingkan berbagai jenis wacana bahasa indonesia (deskripsi narasi)

1 Menentukan tema atau amanat apa yang akan disampaikan 2 Menetapkan sasaran pembaca 3 Merancang peristiwa-peristiwa utama yang akan ditampilkan dalam bentuk skema perkembangandan akhir cerita 5 Memerinci peristiwa-peristiwa utama ke dalam detail-detail peristiwa sebagai pendukung cerita 6 Menyusun tokoh dan perwatakanserta latar dan sudut pandang Karakteristik diatas adalah pengembangan dari paragraf a Narasi

b Deskprisi c Ekspositoris d Sudut Pandang Persuasif

Uraian Pengembangan karangan narasi dapat dilakukan dengan langkah-langkah berikut

(1) menentukan tema atau amanat apa yang akan disampaikan (2) menetapkan sasaran pembaca (3) merancang peristiwa-peristiwa utama yang akan ditampilkan dalam bentuk skema alur (4) membagi peristiwa utama ke dalam ke dalam bagian awal perkembangan dan akhir cerita (5) memerinci peristiwa-peristiwa utama ke dalam detail-detail peristiwa sebagai pendukung cerita dan (6) menyusun tokoh dan perwatakan latar dan sudut pandang

37 KD 135 membandingkan berbagai jenis wacana bahasa indonesia (deskripsi narasi) Karangan narasi yang berusaha untuk memberikan suatu maksud tertentu menyampaikan suatu amanat terselubung kepada para pembaca atau pendengar sehingga tampak seolah-olah melihat disebut a Narasi Informatif b Narasi Ekspositoris c Narasi Sugestif d Narasi Artistik

Uraian Jenis-Jenis Narasi

Narasi Informatif Narasi informatif adalah narasi yang memiliki sasaran penyampaian informasi secara tepat tentang suatu peristiwa

dengan tujuan memperluas pengetahuan orang tentang kisah seseorang Narasi Ekspositorik

Narasi ekspositorik adalah narasi yang memiliki sasaran penyampaian informasi secara tepat tentang suatu peristiwa dengan tujuan memperluas pengetahuan orang tentang kisah seseorang Dalam narasi ekspositorik penulis menceritakan suatu peristiwa berdasarkan data yang sebenarnya Pelaku yang ditonjolkan biasanya satu orang Pelaku diceritakan mulai dari kecil sampai saat ini atau sampai terakhir dalam kehidupannya Karangan narasi ini diwarnai oleh eksposisi maka ketentuan eksposisi juga berlaku pada penulisan narasi ekspositprik Ketentuan ini berkaitan dengan penggunaan bahasa yang logis berdasarkan fakta yang ada tidak memasukan unsursugestif atau bersifat objektif

Narasi Artistik

Narasi artistik adalah narasi yang berusaha untuk memberikan suatu maksud tertentu menyampaikan suatu amanat terselubung kepada para pembaca atau pendengar sehingga tampak seolah-olah melihat Ketentuan ini berkaitan dengan penggunaan bahasa yang logis berdasarkan fakta yang ada tidak memasukan unsur sugestif atau bersifat objektif

Narasi Sugestif Narasi sugestif adalah narasi yang berusaha untuk memberikan suatu maksud tertentu menyampaikan suatu amanat

terselubung kepada para pembaca atau pendengar sehingga tampak seolah-olah melihat 38 KD 135 membandingkan berbagai jenis wacana bahasa indonesia (deskripsi narasi)

Ciri karangan Narasi yang benar kecuali a Menonjolkan unsur perbuatan atau tindakan Membuat pembaca atau pendengar merasakan sendiri atau

mengalami sendiri

b Ada konfiks menjawab pertanyaan apa yang terjadi c Dirangkai dalam urutan waktu menggambarkan dengan jelas suatu peristiwa d Berisi ajakan dirangkai dalam urutan waktu

Uraian Ciri-Ciri Karangan Narasi Menurut Gorys Keraf (2000136)

Menonjolkan unsur perbuatan atau tindakan Dirangkai dalam urutan waktu Berusaha menjawab pertanyaan apa yang terjadi Ada konfiks

39 kd 135 membandingkan berbagai jenis wacana bahasa indonesia (deskripsi narasi) Langkah menyusun paragraf deskripsi yang benar adalah (1)Tentukan objek atau tema yang akan dideskripsikan (2)Mengumpulkan data dengan mengamati objek yang akan dideskripsikan (3)Menyusun data tersebut ke dalam urutan yang baik (menyusun kerangka karangan) (4)Menguraikan kerangka karangan menjadi dekripsi yang sesuai dengan tema yang ditentukan

Beranda Soal UKG Agus Zainal M (SDN Gudang 2)

Republik_ilalangyahoocoid

(5)Tentukan tujuan a 1-5-2-3-4

b 1-5-4-3-2 c 1-5-3-2-4 d 1-3-4-2-5

Uraian Langkah menyusun deskripsi

1Tentukan objek atau tema yang akan dideskripsikan 2Tentukan tujuan 3Mengumpulkan data dengan mengamati objek yang akan dideskripsikan 4Menyusun data tersebut ke dalam urutan yang baik (menyusun kerangka karangan) 5Menguraikan kerangka karangan menjadi dekripsi yang sesuai dengan tema yang ditentukan

40 Kd 311 Merancang aktivitas pembelajaran berdasarkan prinsip dan teori pembelajaran matematika Dalam tahap ini penyajian yang dilakukan melalui tindakan anak secara langsung terlihat dalam memanipulasi (mengotak atik)objek Proses tersebut menurut Bruner dikategorikan ke dalam model a Tahap Enaktif

b Tahap Ikonik c Tahap Simbolik d Tahap Implikatif

Uraian 1 Model Tahap Enaktif

Dalam tahap ini penyajian yang dilakukan melalui tindakan anak secara langsung terlihat dalam memanipulasi (mengotak atik)objek Contoh Budi mempunyai 2 pensil kemudian ibunya memberikannya lagi 3 pinsil Berapa banyak pensil Budi sekarang

2 Model Tahap Ikonik Dalam tahap ini kegiatan penyajian dilakukan berdasarkan pada pikiran internal dimana pengetahuan disajikan melalui serangkaian gambar-gambar atau grafik yang dilakukan anak berhubungan dengan mental yang merupakan gambaran dari objek-objek yang dimanipulasinya Contoh + = hellip

3 Model Tahap Simbolis Dalam tahap ini bahasa adalah pola dasar simbolik anak memanipulasi Simbol-simbol atau lambang-lambang objek tertentu Contoh 2 pensil + 3 pensil = hellip pensil

41 Kd 311 Merancang aktivitas pembelajaran berdasarkan prinsip dan teori pembelajaran matematika Dalam tahap ini kegiatan penyajian dilakukan berdasarkan pada pikiran internal dimana pengetahuan disajikan melalui serangkaian gambar-gambar atau grafik yang dilakukan anak Proses tersebut menurut Bruner dikategorikan ke dalam model a Tahap Enaktif b Tahap Ikonik

c Tahap Simbolik d Tahap Implikatif

Uraian No 40 42 Kd 311 Merancang aktivitas pembelajaran berdasarkan prinsip dan teori pembelajaran matematika

Objek belajar matematika dibagi kedalam Objek Langsung dan Objek Tak Langsung Hal tersebut adalah teori belajar Matematika menurut a Robert M Gagne

b Jerome S Burner c Thorndike d Skinner

Uraian Teori yang diperkenalkan Robert MGagne pada tahun 1960-an pembelajaran harus dikondisikan untuk memunculkan

respons yang diharapkanMenurut Gagne (dalam Ismail 1998) belajar matematika terdiri dari objek langsung dan objek tak langsung

43 Kd 311 Merancang aktivitas pembelajaran berdasarkan prinsip dan teori pembelajaran matematika Perhatikan contoh berikut Budi mempunyai 2 pinsil kemudian ibunya memberikannya lagi 3 pinsilBerapa banyak pinsil Budi sekarang Hal tersebut dikemukakan Bruner dalam Proses Pembelajaran Matematika dalam tahap a Simbolik b Ikonik c Implikatif d Enaktif

Uraian No 40

Beranda Soal UKG Agus Zainal M (SDN Gudang 2)

Republik_ilalangyahoocoid

44 Kd 311 Merancang aktivitas pembelajaran berdasarkan prinsip dan teori pembelajaran matematika Perhatikan gambar disamping Ilustrasi di samping dikemukakan Bruner dalam Proses Pembelajaran Matematika dalam tahap

a Simbolik b Ikonik

c Implikatif d Enaktif

Uraian No 40 45 Kd 311 Merancang aktivitas pembelajaran berdasarkan prinsip dan teori pembelajaran matematika

Perhatikan contoh berikut Contoh 2 pinsil + 3 pinsil = hellippinsil Contoh tersebut dikemukakan Bruner dalam Proses Pembelajaran Matematika dalam tahap a Simbolik

b Ikonik c Implikatif d Enaktif

Uraian No 40 46 Kd 311 Merancang aktivitas pembelajaran berdasarkan prinsip dan teori pembelajaran matematika

Perhatikan contoh berikut penjumlahan bilangan positif dan negatif siswa mencoba sendiri dengan menggunakan garis bilangan Contoh tersebut dikemukakan Bruner dalam Teorema Pembelajaran Matematika a Penyusunan

b Notasi c Pengkontrasan dan Keanekaragaman d Pengaitan

Uraian Berdasarkan hasil pengamatannya Brunner merumuskan 5 teorema dalam pembelajaran matematika yaitu

1) Teorema Penyusunan Menerangkan bahwa cara yang terbaik memulai belajar suatu konsep matematika dalil defenisi dan semacamnya adalah dengan cara menyusun penyajiannya Misalnya dalam mempelajari penjumlahan bilangan positif dan negatif siswa mencoba sendiri dengan menggunakan garis bilangan 2) Teorema Notasi Menerangkan bahwa dalam pengajaran suatu konsep penggunaan notasi-notasi matematika harus diberikan secara bertahap dari yang sederhana ke yang lebih kompleks 3) Teorema Pengkontrasan dan Keanekaragaman Menerangkan bahwa pengontrasan dan keanekaragaman sangat penting dalam melakukan pengubahan konsep matematika dari yang konkrit ke yang lebih abstrak Dalam hal ini diperlukan banyak contoh Contoh yang diberikan harus sesuai dengan rumusan yang diberikan Misalnya menjelaskan persegi panjang disertai juga kemungkinan jajaran genjang dan segi empat lainnya selain persegi panjnag Dengan demikian siswa dapat membedakan apakah segi empat yang diberikan padanya termasuk persegi panjang atau tidak 4) Teorema Pengaitan Menerangkan bahwa dalam matematika terdapat hubungan yang berkaitan antara satu konsep dengan konsep yang lain Di mana materi yang satu merupakan prasyarat yang harus diketahui untuk mempelajari materi yang lain

47 Kd 311 Merancang aktivitas pembelajaran berdasarkan prinsip dan teori pembelajaran matematika Perhatikan contoh berikut Guru menjelaskan persegi panjang disertai juga kemungkinan jajaran genjang dan segi empat lainnya selain persegi panjnag Dengan demikian siswa dapat membedakan apakah segi empat yang diberikan padanya termasuk persegi panjang atau tidak Contoh tersebut dikemukakan Bruner dalam Teorema Pembelajaran Matematika a Penyusunan b Notasi c Pengkontrasan dan Keanekaragaman

d Pengaitan Uraian No 46 48 Kd 311 Merancang aktivitas pembelajaran berdasarkan prinsip dan teori pembelajaran matematika

Tahap pembelajaran Matematika menurut Van Halle adalah sebagai berikut kecuali a Tahap Pengenalan b Tahap Pengurutan c Tahap Analisis d Tahap Penyimpulan

Beranda Soal UKG Agus Zainal M (SDN Gudang 2)

Republik_ilalangyahoocoid

Uraian Van Hiele menyatakan bahwa terdapat 5 tahap belajar siswa dalam belajar geometri yaitu

a Tahap Pengenalan Pada tahap ini siswa mulai belajar mengenal suatu bangun geometri secara keseluruhan namun belum mampu mengetahui adanya sifat-sifat dari bangun geometri yang dilihatnya b Tahap Analisis Pada tahap ini siswa sudah mulai mengenal sifat-sifat yang dimiliki bangun geometri yang diamatinya c Tahap Pengurutan Pada tahap ini siswa sudah mengenal dan memahami sifat-sifat suatu bangun geometri serta sudah dapat mengurutkan bangun-bangun geometri yang satu sama yang lainnya saling berhubungan d Tahap Deduksi Pada tahap ini siswa telah mampu menarik kesimpulan secara deduktif yaitu menarik kesimpulan yang bersifat umum dan menuju ke hal yang bersifat khusus serta dapat mengambil kesimpulan e Tahap Akurasi Pada tahap ini siswa mulai menyadari pentingnya ketepatan prinsip-prinsip dasar yang melandasi suatu pembuktian Tahap berfikir ini merupakan tahap berfikir yang paling tinggi rumit dan kompleks karena di luar jangkauan usia anak-anak SD sampai tingakat SMP

49 Kd 311 Merancang aktivitas pembelajaran berdasarkan prinsip dan teori pembelajaran matematika middot Matematika untuk tujuan pembelajaran dianalisis sebagai kumpulan fakta yang berdiri sendiri dan tidak saling berkaitan middot Anak diharuskan menguasai unsur-unsur yang banyak sekali tanpa diperhatikan pengertiannya middot Anak mempelajari unsur-unsur dalam bentuk seperti yang akan digunakan nanti dalam kesempatan lain middot Anak akan mencapai tujuan ini secara efektif dan efisien dengan melalui pengulangan Teori Pembelajaran Matematika tersebut dikemukakan oleh a Skinner b Piaget c Van Brownell

d Thorndike Uraian Brownell mengemukakan tentang Teori Makna (Meaning Theory) sebagai pengganti Teori Latihan HafalUlangan (Drill

Theory) Intisari dari teori Drill adalah - Matematika untuk tujuan pembelajaran dianalisis sebagai kumpulan fakta yang berdiri sendiri dan tidak saling berkaitan - Anak diharuskan menguasai unsur-unsur yang banyak sekali tanpa diperhatikan pengertiannya - Anak mempelajari unsur-unsur dalam bentuk seperti yang akan digunakan nanti dalam kesempatan lain - Anak akan mencapai tujuan ini secara efektif dan efisien dengan melalui pengulangan

50 Kd 311 Merancang aktivitas pembelajaran berdasarkan prinsip dan teori pembelajaran matematika Tahap operasionaloperasi konkrit adalah teori Pembelajaran Matematika yang dikemukakan Peaget pada tahap usia a dibawah 2 tahun b 2-7 Tahun c 7-8 Tahun d 7-12 Tahun

Uraian Piaget membagi skema yang digunakan anak untuk memahami dunianya melalui empat periode utama yang berkorelasi

dengan dan semakin canggih seiring pertambahan usia Periode sensorimotor (usia 0ndash2 tahun) Periode praoperasional (usia 2ndash7 tahun) Periode operasional konkrit (usia 7ndash12 tahun) Periode operasional formal (usia 12 tahun sampai dewasa)

51 313 Memilih media pembelajaran yang tepat untuk pembelajaran operasi bilangan bulat Untuk operasi bilangan bulat perkalian media yang paling tepat digunakan adalah a sapu lidi

b kerikil c daun pakis d koin

Uraian 52 313 Memilih media pembelajaran yang tepat untuk pembelajaran operasi bilangan bulat

Untuk operasi bilangan bulat negatif kita bisa menggunakan media a uang kertas b kerikil c mistar d koin

Uraian 53 313 Memilih media pembelajaran yang tepat untuk pembelajaran operasi bilangan bulat

Beranda Soal UKG Agus Zainal M (SDN Gudang 2)

Republik_ilalangyahoocoid

Untuk operasi bilangan bulat penjumlahan media yang paling tepat digunakan adalah a uang kertas b kerikil c daun pakis d koin

Uraian 54 314 Memilih media pembelajaran yang tepat untuk pembelajaran operasi bilangan pecahan

Untuk operasi bilangan pecahan kita bisa menggunakan media berikut kecuali a kartu bilangan b garis bilangan c gambar bidang d blok pecahan

Uraian 55 316 Memilih media pembelajaran yang tepat untuk pembelajaran geometri dan pengukuran

Untuk menghitung luas dan keliling bangun datar media yang paling tepat digunakan adalah a Penggaris b Kertas Folio Bergaris

c Neraca d Kalkulator

Uraian

Page 6: 50 SOAL UKG & PEMBAHASAN SESUAI KISI-KISIS.pdf

Beranda Soal UKG Agus Zainal M (SDN Gudang 2)

Republik_ilalangyahoocoid

didik c Mengendalikan kekuatan sendiri bukan didasarkan atas kekuatan orang lain d Melakukan studi kasus yang berkesinambungan

Uraian Menurut Akhlan dan Rahman (19977) karakteristik perencanaan pengajaran yang baik hendaknya mengandung prinsip

sebagai berikut a Mengembangkan hubungan interaksi yang baik di antara sesama manusia dalam hal ini siswa dan guru serta personal

terkait b Merupakan suatu wahana atau wadah untuk mengembangkan segala potensi yang ada dan dimiliki oleh anak didik c Memiliki sikap objektif rasio (tepat dan masuk akal) komprehensif dan sistematis (menyeluruh dan tersusun rapi) d Mengendalikan kekuatan sendiri bukan didasarkan atas kekuatan orang lain Didukung oleh fakta dan data yang

menunjang pencapaian tujuan yang telah di dirumuskan e Fleksibel dan dinamis artinya mudah disesuaikan dengan keadaan serta perkembangan ke arah yang lebih baik dan

maju 24 KD 123 Perencanaan dan Pelaksanaan evaluasi pembelajaran Bahasa dan Sastra Indonesia

tahap mengumpulkan informasi tentang keadaan objek evaluasi (siswa) dengan menggunakan teknik tes atau nontes disebut tahapan a Tahap Tindak Lanjut b Tahap Persiapan c Tahap Pelaksanaan

d Tahap Pengolahan Hasil Uraian Berikut ini penjelasan singkat tentang keempat tahap evaluasi pembelajaran tersebut

(1) Tahap Persiapan Menurut Damaianti (2007 8) tahap ini disebut juga tahap perencanaan dan perumusan kriterium Langkahnya meliputi (a) perumusan tujuan evaluasi (b) penetapan aspek-aspek yang akan dievaluasi (c) menetapkan metode dan bentuk evaluasi (tesnontes) (d) merencanakan waktu evaluasi (e) melakukan uji coba (untuk tes) agar dapat mengukur validitas dan reliabilitasnya Untuk evaluasi yang menggunakan tes hasil dari tahap ini adalah kisi-kisi soal dan seperangkat alat tes soal lembar jawaban (untuk tes tulis) kunci jawaban dan pedoman penilaian 2) Tahap Pelaksanaan Tahap pelaksanaan atau disebut juga dengan tahap pengukuran dan pengumpulan data adalah tahap untuk mengumpulkan informasi tentang keadaan objek evaluasi (siswa) dengan menggunakan teknik tes atau nontes Bila menggunakan teknik tes soal yang igunakan sebaiknya sudah teruji validitas dan reliabilitasnya Tes yang digunakan dapat berbentuk tes tulis lisan atau praktik 3) Tahap Pengolahan Hasil Tahap pengolahan hasil adalah tahap pemeriksaan hasil evaluasi dengan memberikan skor Skor yang diperoleh siswa selanjutnya diubah menjadi nilai Pada tes tulis pemeriksaan hasil dilakukan setelah tes selesai sedangkan pada tes lisan dan praktik pemberian nilai dilakukan bersamaan dengan waktu pelasanaan tes tersebut 4) Tahap Tindak Lanjut Tahap tindak lanjut atau disebut juga tahap penafsiran adalah tahap untuk mengambil keputusan berdasarkan nilai yang dihasilkan pada tahap pengolahan hasil misalnya a memperbaiki proses belajar mengajar b memperbaiki kesulitan belajar siswa c memperbaiki alat evaluasi d membuat laporan evaluasi (rapor)

25 KD 131 Merumuskan hakikat (pengertiantujuanjenismanfaat) membaca membaca yang mengutamakan isi bacaan sebagai ungkapan pikiran perasaan dan kehendak penulis Bila hanya ingin mengetahui isinya membaca cerdas bersifat lugas Akan tetapi bila maksudnya untuk memahami dan memilki isi bacaan maka tergolong kedalam membaca jenis a Membaca cerdas atau membaca dalam hati

b Membaca bahasa c Membaca teknis d Membaca bebas

Uraian Menurut Soedjono dalam Sue (200418-21) ada lima macam membaca yaitu membaca bahasa membaca cerdas atau

membaca dalam hati membaca teknis membaca emosional dan membaca bebas 1) Membaca bahasa Membaca bahasa adalah membaca yang mengutamakan bahasa bacaan Membaca bahasa mementingkan segi bahasa bacaan Hal-hal yang perlu diperhatikan dalam membaca bahasa adalah kesesuian pikir dengan bahasa perbendaharaan bahasa yang meliputi kosa kata struktur kalimat dan ejaan 2) Membaca cerdas atau membaca dalam hati

Beranda Soal UKG Agus Zainal M (SDN Gudang 2)

Republik_ilalangyahoocoid

Membaca cerdas adalah membaca yang mengutamakan isi bacaan sebagai ungkapan pikiran perasaan dan kehendak penulis Bila hanya ingin mengetahui isinya membaca cerdas bersifat lugas Akan tetapi bial maksudnya untuk memahami dan memilki isi bacaan maka disebut membaca belajar 3) Membaca teknis Membaca teknis adalah membaca dengan mengarahkan bacaan secara wajar Wajar maksudnya sesuai ucapan tekanan dan intonasinya Pikiran perasaan dan kemauan yang tersimpan dalam bacaan dapat diaktualisasikan dengan baik 4) Membaca emosional Membaca emosional adalah membaca sebagai sarana untuk memasuki perasaan yaitu keindahan isi dan keindahan bahasanya 5) Membaca bebas Membaca bebas adalah membaca sesuatu atas kehendak sendiri tanpa adanya unsur paksaan dari luar Unsur dari luar misalnya guru orang tua teman atau pihak-pihak lain

26 KD 131 Merumuskan hakikat (pengertiantujuanjenismanfaat) membaca membaca sesuatu atas kehendak sendiri tanpa adanya unsur paksaan dari luar Unsur dari luar misalnya guru orang tua teman atau pihak-pihak lain termasuk jenis membaca a Membaca cerdas atau membaca dalam hati b Membaca bahasa c Membaca teknis d Membaca bebas

Uraian No 25 27 KD 131 Merumuskan hakikat (pengertiantujuanjenismanfaat) membaca

1 menggunakan ucapan yang tepat 2 menggunakan frase yang tepat 3 menggunakan intonasi suara yang wajar 4 dalam posisi sikap yang baik 5 menguasai tanda-tanda baca 6 membaca dengan terang dan jelas keterampilan di atas harus dipunyai dalam jenis membaca a Membaca cerdas atau membaca dalam hati b Membaca bahasa c Membaca teknis d Membaca nyaring

Uraian JENIS-JENIS MEMBACA DAN KARAKTERISTIKNYA

Ditinjau dari segi terdengar atau tidaknya suara pembaca waktu melakukan kegiatan membaca maka proses membaca dapat dibedakan menjadi A Membaca Nyaring Membaca nyaring adalah kegiatan membaca dengan menyuarakan tulisan yang dibacanya dengan ucapan dan intonasi yang tepat agar pendengar dan pembaca dapat menangkap informasi yang disampaikan oleh penulis baik yang berupa pikiran perasaan sikap ataupun pengalaman penulis Ketrampilan yang dituntut dalam membaca nyaring adalah berbagai kemampuan diantaranya adalah 1 menggunakan ucapan yang tepat 2 menggunakan frase yang tepat 3 menggunakan intonasi suara yang wajar 4 dalam posisi sikap yang baik 5 menguasai tanda-tanda baca 6 membaca dengan terang dan jelas 7 membaca dengan penuh perasaan ekspresif 8 membaca dengan tidak terbata-bata 9 mengerti serta memahami bahan bacaan yang dibacanya 10 kecepatan bergantung pada bahan bacaan yang dibacanya 11 membaca dengan tanpa terus-menerus melihat bahan bacaan 12 membaca dengan penuh kepercayaan pada diri sendiri B Membaca Dalam Hati Membaca dalam hati adalah kegiatan membaca yang dilakukan dengan tanpa menyuarakan isi bacaan yang dibacanya Ketrampilan yang dituntut dalam membaca dalam hati antara lain sebagai berikut 1 membaca tanpa bersuara tanpa bibir bergerak tanpa ada desis apapun 2 membaca tanpa ada gerakan-gerakan kepala 3 membaca lebih cepat dibandingkan dengan membaca nyaring 4 tanpa menggunakan jari atau alat lain sebagai penunjuk 5 mengerti dan memahami bahan bacaan 6 dituntut kecepatan mata dalam membaca 7 membaca dengan pemahaman yang baik

Beranda Soal UKG Agus Zainal M (SDN Gudang 2)

Republik_ilalangyahoocoid

8 dapat menyesuaikan kecepatan dengan tingkat kesukaran yang terdapat dalam bacaan 28 KD 131 Merumuskan hakikat (pengertiantujuanjenismanfaat) membaca

Hal berikut dilakukan seseorang ketika membaca survai kecuali a memeriksa judul bacaanbuku kata pengantar daftar isi dan malihat abstrak(jika ada) b memeriksa bagian terahkir dari isi (kesimpulan) jika ada c memeriksa indeks dan apendiks(jika ada) d membaca biografi pengarang

Uraian Secara garis besar membaca dalam hati dapat dibedakan menjadi dua (I) MEMBACA EKSTENSIF amp (II) MEMBACA

INTENSIF Berikut penjelasan secara rinci kedua jenis membaca tersebut I Membaca Ekstensif membaca ekstensif adalah membaca secara luas Objeknya meliputi sebanyak mungkin teks dalam waktu yang sesingkat-singkatnya Membaca ekstensif meliputi 1 Membaca Survai (Survey Reading) Membaca survai adalah kegiatan membaca untuk mengetahui secara sekilas terhadap bahan bacaan yang akan dibaca lebih mendalam Kegiatan membaca survai merupakan pendahuluan dalam membaca ekstensif Yang dilakukan seseorang ketika membaca survai adalah sebagai berikut (a) memeriksa judul bacaanbuku kata pengantar daftar isi dan malihat abstrak(jika ada) (b) memeriksa bagian terahkir dari isi (kesimpulan) jika ada (c) memeriksa indeks dan apendiks(jika ada)

2 Membaca Sekilas Membaca sekilas atau membaca cepat adalah kegiatan membaca dengan mengandalakan kecepatan gerak mata dalam melihat dan memperhatikan bahan tertulis yang dibacanya dengan tujuan untuk mendapatkan informasi secara cepat Metode yang digunakan dalam melatihkan membaca cepat adalah (a) metode kosakata metode yang berusaha untuk menambah kosakata (b) Metode motivasi metode yang berusaha memotivasi pembaca(pemula) yang mengalami hambatan (c) Metode gerak mata metode yang mengembangkan kecepatan membaca dengan menigkatkan kecepatan gerak mata Hambatan-hambatan yang dapat mengurangi kecepatan mambaca (a) vokalisai atau berguman ketika membaca (b) membaca dengan menggerakan bibir tetapi tidak bersuara (c) kepala bergerak searah tulisan yang dibaca (d) subvokalisasi suara yang biasa ikut membaca di dalam pikiran kita (e) jari tangan selalu menunjuk tulisa yang sedang kit abaca (f) gerakan mata kembali pada kata-kata sebelumnya 3 Membaca Dangkal (Superficial Reading) membaca dangkal pada hakekatnya bertujuan untuk memperoleh pemahaman yang dangkal yang bersifat luaran yang tidak mendalam dari suatu bahan bacaan Membaca jenis ini biasanya dilakukan seseorang membaca demi kesenangan membaca bacaan ringan yang mendatangkan kesenangan kegembiraan sebagai pengisi waktu senggang

29 KD 131 Merumuskan hakikat (pengertiantujuanjenismanfaat) membaca Membaca jenis ini biasanya dilakukan seseorang membaca demi kesenangan membaca bacaan ringan yang mendatangkan kesenangan kegembiraan sebagai pengisi waktu senggang Berdasarkan karakteristik diatas kegiatan tersebut termasuk ke dalam membaca jenis a Membaca Survai (Survey Reading) b Membaca Sekilas c Membaca Dangkal (Superficial Reading)

d Membaca Nyaring Uraian No 28 30 KD 131 Merumuskan hakikat (pengertiantujuanjenismanfaat) membaca

Berikut adalah tujuan umum dalam aktifitas membaca kecuali a Membaca untuk memperoleh perincian-perincian atau fakta-fakta (reading for details or facts) Membaca tersebut

bertujuan untuk menemukan atau mengetahui penemuan-penemuan telah dilakukan oleh sang tokoh untuk memecahkan masalah-masalah yang dibuat oleh sang tokoh

b Membaca untuk mengetahui ukuran atau susunan organisasi cerita (reading for sequenceor organization) Membaca tersebut bertujuan untuk mengetahui bagian-bagian cerita dan hubungan antar bagian-bagian cerita

c Membaca untuk menyimpulkan atau membaca inferensi (reading for inference) d Membaca untuk memperoleh kekurangan suatu buku (finding mistakes)

Uraian Tujuan Membaca

Beranda Soal UKG Agus Zainal M (SDN Gudang 2)

Republik_ilalangyahoocoid

Berdasarkan maksud tujuan atau keintensifan serta cara dalam membaca di bawah ini Anderson dalam Tarigan (19799-10) mengemukakan beberapa tujuan membaca antara lain a Membaca untuk memperoleh perincian-perincian atau fakta-fakta (reading for details or facts) Membaca tersebut bertujuan untuk menemukan atau mengetahui penemuan-penemuan telah dilakukan oleh sang tokoh untuk memecahkan masalah-masalah yang dibuat oleh sang tokoh b Membaca untuk memperoleh ide-ide utama (reading for main ideas) Membaca untuk mengetahui topik atau masalah dalam bacaan Untuk menemukan ide pokok bacaan dengan membaca halamn demi halaman c Membaca untuk mengetahui ukuran atau susunan organisasi cerita (reading for sequenceor organization) Membaca tersebut bertujuan untuk mengetahui bagian-bagian cerita dan hubungan antar bagian-bagian cerita d Membaca untuk menyimpulkan atau membaca inferensi (reading for inference) Pembaca diharapkan dapat merasakan sesuatu yang dirasakan penulis e Membaca untuk mengelompokkan atau mengklasifikasikan (reading for classify) Membaca jenis ini bertujuan untuk menemukan hal-hal yang tidak wajar mengenai sesuatu hal (Anderson dalam Tarigan 197910) f Membaca untuk menilai atau mengevaluasai (reading to evaluate) Jenis membaca tersebut bertujuan menemukan suatu keberhasilan berdasarkan ukuran-ukuran tertentu Membaca jenis ini memerlukan ketelitian dengan membandingkan dan mengujinya kembali g Membaca untuk memperbandingkan atau mempertentangkan (reading to compare or contrast) Tujuan membaca tersebut adalah untuk menemukan bagaimana cara perbedaan atau persamaan dua hal atau lebih

31 KD 133 menemukan pesan pokok utama sebuah berita Simak penggalan berita berikut Hebat Siswa Indonesia Pertahankan Tradisi Emas di Olimpiade Fisika Tim Olimpiade Fisika Indonesia REPUBLIKACOID JAKARTA--Luar biasa Lima siswa Indonesia yang dikirim ke ajang Olimpiade Fisika atau International Physics Olympiad (IPhO) ke-41 di Zagreb Kroasia 17-25 Juli berhasil menyabet empat medali emas dan satu perak Pelajar yang menyumbang emas adalah Muhammad Sohibul Maromi (SMAN 1 Pamekasan Madura) Christian George Emor (SMA Lokon St Nikolaus Tomohon Sulawesi Utara) David Giovanni (SMAK Penabur Gading Serpong Banten) dan Kevin Soedyatmiko (SMAN 12 Jakarta) Sedangkan medali perak berhasil diraih oleh Ahmad Ataka Awwalur Rizqi (SMAN 1 Yogyakarta) Prestasi ini jauh lebih baik dibanding ajang Olimpiade Fisika ke-40 di Merida Yucatan Meksiko 2009 yang lalu Saat itu delegasi siswa Indonesia merebut satu medali emas dua medali perak dan satu perunggu Hasil empat medali emas dan satu perak ini hampir menyamai prestasi terbaik sebelumnya pada ajang Olimpiade Fisika ke-37 di Singapura Saat itu siswa Indonesia tidak hanya berhasil menyabet 4 medali emas namun juga meraih predikat `Absolute Winner` atas nama Mailoa Jonathan Pradana (SMAK 1 BPK Penabur Jakarta) Tapi yang terpenting lagi pelajar Indonesia berhasil mempertahankan tradisi emas di setiap ajang Olimpiade Fisika Red Endro Yuwanto Peristiwa yang diberitakan adalah a siswa Indonesia pertahankan tradisi emas di Olimpiade Matematika b siswa Indonesia pertahankan tradisi emas di Olimpiade Fisika c siswa Indonesia menyumbang emas di Olimpiade Matematika

d siswa Indonesia menyumbang perak di Olimpiade Fisika Uraian 32 KD 133 menemukan pesan pokok utama sebuah berita

Simak penggalan berita berikut Hebat Siswa Indonesia Pertahankan Tradisi Emas di Olimpiade Fisika Tim Olimpiade Fisika Indonesia REPUBLIKACOID JAKARTA--Luar biasa Lima siswa Indonesia yang dikirim ke ajang Olimpiade Fisika atau International Physics Olympiad (IPhO) ke-41 di Zagreb Kroasia 17-25 Juli berhasil menyabet empat medali emas dan satu perak Pelajar yang menyumbang emas adalah Muhammad Sohibul Maromi (SMAN 1 Pamekasan Madura) Christian George Emor (SMA Lokon St Nikolaus Tomohon Sulawesi Utara) David Giovanni (SMAK Penabur Gading Serpong Banten) dan Kevin Soedyatmiko (SMAN 12 Jakarta) Sedangkan medali perak berhasil diraih oleh Ahmad Ataka Awwalur Rizqi (SMAN 1 Yogyakarta) Prestasi ini jauh lebih baik dibanding ajang Olimpiade Fisika ke-40 di Merida Yucatan Meksiko 2009 yang lalu Saat itu delegasi siswa Indonesia merebut satu medali emas dua medali perak dan satu perunggu Hasil empat medali emas dan satu perak ini hampir menyamai prestasi terbaik sebelumnya pada ajang Olimpiade Fisika ke-37 di Singapura Saat itu siswa Indonesia tidak hanya berhasil menyabet 4 medali emas namun juga meraih predikat `Absolute Winner` atas nama Mailoa Jonathan Pradana (SMAK 1 BPK Penabur Jakarta) Tapi yang terpenting lagi pelajar Indonesia berhasil mempertahankan tradisi emas di setiap ajang Olimpiade Fisika Red Endro Yuwanto Lima siswa Indonesia berhasil mendapatkan a 4 emas 1 perak

b 1 emas 4 perak c 3 emas 2 perak d 2 perak 3 emas

Beranda Soal UKG Agus Zainal M (SDN Gudang 2)

Republik_ilalangyahoocoid

Uraian 33 KD 134 menemukan pesan pokok dalam wacana naratif seperti cerita rakyat puisi

Surat dari ibu Jika bayang telah pudar Dan elang laut pulang ke sarang Angin bertiup ke benua Tiang ndash tiang akan kering sendiri Dan nahkoda sudah tau pedoman Boleh engkau datang padaku Makna lambang dari nahkoda sudah tahu pedoman adalahhellip a Sudah mencari pedoman hidup b Sudah menemukan arah dan tujuan

c Sudah berilmu dan berpengalaman d Sudah mempunyai pasangan hidup

Uraian 34 KD 135 membandingkan berbagai jenis wacana bahasa indonesia (deskripsi narasi)

Pendekatan untuk mendapat tanggapan emosional pembaca ataupun kesan pembaca adalah contoh Pendekatan Deskripsi jenis a Pendekatan Ekspositoris b Pendekatan Impresionistik

c Pendekatan menurut sikap pengarang d Pendekatan Realistik

Uraian PENDEKATAN DESKRIPSI

Pendekatan dalam pendeskripsian dapat dibedakan menjadi beberapa kategori pendekatan yaitu

1 Pendekatan Ekspositoris

Dalam pendekatan ini kita berusaha agar deskripsi yang kita buat dapat memberi keterangan sesuai dengan keadan yang

sebenarnya sehingga pembaca dapat seolah-olah ikut melihat atau merasakan objek yang kita deskripsikan Karangan

jenis ini berisi daftar detail sesuatu secara lengkap sehingga pembaca dan penalarannya dapat memperoleh kesan

keseluruhan tentang sesuatu

2 Pendekatan Impresionistik

Tujuan deskripsi impresionistik ialah untuk mendapatkan tanggapan emosional pembaca ataupun kesan pembaca Corak

deskripsi ini diantaranya juga ditentukan oleh macam kesan apa yang diinginkan penulisnya

3 Pendekatan menurut Sikap Pengarang

Pendekatan ini sangat bergantung pada tujuan yang ingin dicapai sifat objek serta pembaca deskripsinya Dalam

menguraikan sebuah gagasan penulis mungkin mengharapkan agar pembaca merasa tidak puas terhadapa suatu

tindakan atau keadaan atau penulis menginginkan agar pembaca juga harus merasakan bahwa persoalan yang dihadapi

merupakan masalah yang gawat Penulis juga dapat membayangkan bahwa akan terjadi sesuatu yang tidak diinginkan

sehingga pembaca dari semula sudah disiapkan perasaan yang kurang enak seram takut dan sebagainya (Alkhaidah

1997) 35 kd 135 membandingkan berbagai jenis wacana bahasa indonesia (deskripsi narasi)

Prinsip-Prinsip karangan Narasi sebagai berikut kecuali a Alur b Penokohan c Amanat

d Sudut Pandang Uraian Prinsip-prinsip dasar narasi sebagai tumpuan berfikir adalah sebagai berikut

1 Alur (plot) biasa disebut dengan jalan cerita

2 Penokohan (rangkaian perbuatanaksi) 3 Latar (setting) yaitu tempat dan atau waktu terjadinya perbuatan tokoh atau peristiwa yang dialami tokoh 4 Sudut Pandang (Point of View)

a Narrator Serba Tahu (Omniscient point of view) Yaitu narrator bertindak sebagai pencipta segalanya b Narrator bertindak objektif (Objektive point of view) Pengarang tak member komentar apa pun dan pembaca bebas menafsirkan apa yang diceritakan pengarang c Narrator ikut aktif ( Narator acting) Narrator terlibat dalam cerita d Narrator sebagai peninjau

Beranda Soal UKG Agus Zainal M (SDN Gudang 2)

Republik_ilalangyahoocoid

Pelakunya adalah orang ketiga yang biasa disebut ldquodiardquo 36 KD 135 membandingkan berbagai jenis wacana bahasa indonesia (deskripsi narasi)

1 Menentukan tema atau amanat apa yang akan disampaikan 2 Menetapkan sasaran pembaca 3 Merancang peristiwa-peristiwa utama yang akan ditampilkan dalam bentuk skema perkembangandan akhir cerita 5 Memerinci peristiwa-peristiwa utama ke dalam detail-detail peristiwa sebagai pendukung cerita 6 Menyusun tokoh dan perwatakanserta latar dan sudut pandang Karakteristik diatas adalah pengembangan dari paragraf a Narasi

b Deskprisi c Ekspositoris d Sudut Pandang Persuasif

Uraian Pengembangan karangan narasi dapat dilakukan dengan langkah-langkah berikut

(1) menentukan tema atau amanat apa yang akan disampaikan (2) menetapkan sasaran pembaca (3) merancang peristiwa-peristiwa utama yang akan ditampilkan dalam bentuk skema alur (4) membagi peristiwa utama ke dalam ke dalam bagian awal perkembangan dan akhir cerita (5) memerinci peristiwa-peristiwa utama ke dalam detail-detail peristiwa sebagai pendukung cerita dan (6) menyusun tokoh dan perwatakan latar dan sudut pandang

37 KD 135 membandingkan berbagai jenis wacana bahasa indonesia (deskripsi narasi) Karangan narasi yang berusaha untuk memberikan suatu maksud tertentu menyampaikan suatu amanat terselubung kepada para pembaca atau pendengar sehingga tampak seolah-olah melihat disebut a Narasi Informatif b Narasi Ekspositoris c Narasi Sugestif d Narasi Artistik

Uraian Jenis-Jenis Narasi

Narasi Informatif Narasi informatif adalah narasi yang memiliki sasaran penyampaian informasi secara tepat tentang suatu peristiwa

dengan tujuan memperluas pengetahuan orang tentang kisah seseorang Narasi Ekspositorik

Narasi ekspositorik adalah narasi yang memiliki sasaran penyampaian informasi secara tepat tentang suatu peristiwa dengan tujuan memperluas pengetahuan orang tentang kisah seseorang Dalam narasi ekspositorik penulis menceritakan suatu peristiwa berdasarkan data yang sebenarnya Pelaku yang ditonjolkan biasanya satu orang Pelaku diceritakan mulai dari kecil sampai saat ini atau sampai terakhir dalam kehidupannya Karangan narasi ini diwarnai oleh eksposisi maka ketentuan eksposisi juga berlaku pada penulisan narasi ekspositprik Ketentuan ini berkaitan dengan penggunaan bahasa yang logis berdasarkan fakta yang ada tidak memasukan unsursugestif atau bersifat objektif

Narasi Artistik

Narasi artistik adalah narasi yang berusaha untuk memberikan suatu maksud tertentu menyampaikan suatu amanat terselubung kepada para pembaca atau pendengar sehingga tampak seolah-olah melihat Ketentuan ini berkaitan dengan penggunaan bahasa yang logis berdasarkan fakta yang ada tidak memasukan unsur sugestif atau bersifat objektif

Narasi Sugestif Narasi sugestif adalah narasi yang berusaha untuk memberikan suatu maksud tertentu menyampaikan suatu amanat

terselubung kepada para pembaca atau pendengar sehingga tampak seolah-olah melihat 38 KD 135 membandingkan berbagai jenis wacana bahasa indonesia (deskripsi narasi)

Ciri karangan Narasi yang benar kecuali a Menonjolkan unsur perbuatan atau tindakan Membuat pembaca atau pendengar merasakan sendiri atau

mengalami sendiri

b Ada konfiks menjawab pertanyaan apa yang terjadi c Dirangkai dalam urutan waktu menggambarkan dengan jelas suatu peristiwa d Berisi ajakan dirangkai dalam urutan waktu

Uraian Ciri-Ciri Karangan Narasi Menurut Gorys Keraf (2000136)

Menonjolkan unsur perbuatan atau tindakan Dirangkai dalam urutan waktu Berusaha menjawab pertanyaan apa yang terjadi Ada konfiks

39 kd 135 membandingkan berbagai jenis wacana bahasa indonesia (deskripsi narasi) Langkah menyusun paragraf deskripsi yang benar adalah (1)Tentukan objek atau tema yang akan dideskripsikan (2)Mengumpulkan data dengan mengamati objek yang akan dideskripsikan (3)Menyusun data tersebut ke dalam urutan yang baik (menyusun kerangka karangan) (4)Menguraikan kerangka karangan menjadi dekripsi yang sesuai dengan tema yang ditentukan

Beranda Soal UKG Agus Zainal M (SDN Gudang 2)

Republik_ilalangyahoocoid

(5)Tentukan tujuan a 1-5-2-3-4

b 1-5-4-3-2 c 1-5-3-2-4 d 1-3-4-2-5

Uraian Langkah menyusun deskripsi

1Tentukan objek atau tema yang akan dideskripsikan 2Tentukan tujuan 3Mengumpulkan data dengan mengamati objek yang akan dideskripsikan 4Menyusun data tersebut ke dalam urutan yang baik (menyusun kerangka karangan) 5Menguraikan kerangka karangan menjadi dekripsi yang sesuai dengan tema yang ditentukan

40 Kd 311 Merancang aktivitas pembelajaran berdasarkan prinsip dan teori pembelajaran matematika Dalam tahap ini penyajian yang dilakukan melalui tindakan anak secara langsung terlihat dalam memanipulasi (mengotak atik)objek Proses tersebut menurut Bruner dikategorikan ke dalam model a Tahap Enaktif

b Tahap Ikonik c Tahap Simbolik d Tahap Implikatif

Uraian 1 Model Tahap Enaktif

Dalam tahap ini penyajian yang dilakukan melalui tindakan anak secara langsung terlihat dalam memanipulasi (mengotak atik)objek Contoh Budi mempunyai 2 pensil kemudian ibunya memberikannya lagi 3 pinsil Berapa banyak pensil Budi sekarang

2 Model Tahap Ikonik Dalam tahap ini kegiatan penyajian dilakukan berdasarkan pada pikiran internal dimana pengetahuan disajikan melalui serangkaian gambar-gambar atau grafik yang dilakukan anak berhubungan dengan mental yang merupakan gambaran dari objek-objek yang dimanipulasinya Contoh + = hellip

3 Model Tahap Simbolis Dalam tahap ini bahasa adalah pola dasar simbolik anak memanipulasi Simbol-simbol atau lambang-lambang objek tertentu Contoh 2 pensil + 3 pensil = hellip pensil

41 Kd 311 Merancang aktivitas pembelajaran berdasarkan prinsip dan teori pembelajaran matematika Dalam tahap ini kegiatan penyajian dilakukan berdasarkan pada pikiran internal dimana pengetahuan disajikan melalui serangkaian gambar-gambar atau grafik yang dilakukan anak Proses tersebut menurut Bruner dikategorikan ke dalam model a Tahap Enaktif b Tahap Ikonik

c Tahap Simbolik d Tahap Implikatif

Uraian No 40 42 Kd 311 Merancang aktivitas pembelajaran berdasarkan prinsip dan teori pembelajaran matematika

Objek belajar matematika dibagi kedalam Objek Langsung dan Objek Tak Langsung Hal tersebut adalah teori belajar Matematika menurut a Robert M Gagne

b Jerome S Burner c Thorndike d Skinner

Uraian Teori yang diperkenalkan Robert MGagne pada tahun 1960-an pembelajaran harus dikondisikan untuk memunculkan

respons yang diharapkanMenurut Gagne (dalam Ismail 1998) belajar matematika terdiri dari objek langsung dan objek tak langsung

43 Kd 311 Merancang aktivitas pembelajaran berdasarkan prinsip dan teori pembelajaran matematika Perhatikan contoh berikut Budi mempunyai 2 pinsil kemudian ibunya memberikannya lagi 3 pinsilBerapa banyak pinsil Budi sekarang Hal tersebut dikemukakan Bruner dalam Proses Pembelajaran Matematika dalam tahap a Simbolik b Ikonik c Implikatif d Enaktif

Uraian No 40

Beranda Soal UKG Agus Zainal M (SDN Gudang 2)

Republik_ilalangyahoocoid

44 Kd 311 Merancang aktivitas pembelajaran berdasarkan prinsip dan teori pembelajaran matematika Perhatikan gambar disamping Ilustrasi di samping dikemukakan Bruner dalam Proses Pembelajaran Matematika dalam tahap

a Simbolik b Ikonik

c Implikatif d Enaktif

Uraian No 40 45 Kd 311 Merancang aktivitas pembelajaran berdasarkan prinsip dan teori pembelajaran matematika

Perhatikan contoh berikut Contoh 2 pinsil + 3 pinsil = hellippinsil Contoh tersebut dikemukakan Bruner dalam Proses Pembelajaran Matematika dalam tahap a Simbolik

b Ikonik c Implikatif d Enaktif

Uraian No 40 46 Kd 311 Merancang aktivitas pembelajaran berdasarkan prinsip dan teori pembelajaran matematika

Perhatikan contoh berikut penjumlahan bilangan positif dan negatif siswa mencoba sendiri dengan menggunakan garis bilangan Contoh tersebut dikemukakan Bruner dalam Teorema Pembelajaran Matematika a Penyusunan

b Notasi c Pengkontrasan dan Keanekaragaman d Pengaitan

Uraian Berdasarkan hasil pengamatannya Brunner merumuskan 5 teorema dalam pembelajaran matematika yaitu

1) Teorema Penyusunan Menerangkan bahwa cara yang terbaik memulai belajar suatu konsep matematika dalil defenisi dan semacamnya adalah dengan cara menyusun penyajiannya Misalnya dalam mempelajari penjumlahan bilangan positif dan negatif siswa mencoba sendiri dengan menggunakan garis bilangan 2) Teorema Notasi Menerangkan bahwa dalam pengajaran suatu konsep penggunaan notasi-notasi matematika harus diberikan secara bertahap dari yang sederhana ke yang lebih kompleks 3) Teorema Pengkontrasan dan Keanekaragaman Menerangkan bahwa pengontrasan dan keanekaragaman sangat penting dalam melakukan pengubahan konsep matematika dari yang konkrit ke yang lebih abstrak Dalam hal ini diperlukan banyak contoh Contoh yang diberikan harus sesuai dengan rumusan yang diberikan Misalnya menjelaskan persegi panjang disertai juga kemungkinan jajaran genjang dan segi empat lainnya selain persegi panjnag Dengan demikian siswa dapat membedakan apakah segi empat yang diberikan padanya termasuk persegi panjang atau tidak 4) Teorema Pengaitan Menerangkan bahwa dalam matematika terdapat hubungan yang berkaitan antara satu konsep dengan konsep yang lain Di mana materi yang satu merupakan prasyarat yang harus diketahui untuk mempelajari materi yang lain

47 Kd 311 Merancang aktivitas pembelajaran berdasarkan prinsip dan teori pembelajaran matematika Perhatikan contoh berikut Guru menjelaskan persegi panjang disertai juga kemungkinan jajaran genjang dan segi empat lainnya selain persegi panjnag Dengan demikian siswa dapat membedakan apakah segi empat yang diberikan padanya termasuk persegi panjang atau tidak Contoh tersebut dikemukakan Bruner dalam Teorema Pembelajaran Matematika a Penyusunan b Notasi c Pengkontrasan dan Keanekaragaman

d Pengaitan Uraian No 46 48 Kd 311 Merancang aktivitas pembelajaran berdasarkan prinsip dan teori pembelajaran matematika

Tahap pembelajaran Matematika menurut Van Halle adalah sebagai berikut kecuali a Tahap Pengenalan b Tahap Pengurutan c Tahap Analisis d Tahap Penyimpulan

Beranda Soal UKG Agus Zainal M (SDN Gudang 2)

Republik_ilalangyahoocoid

Uraian Van Hiele menyatakan bahwa terdapat 5 tahap belajar siswa dalam belajar geometri yaitu

a Tahap Pengenalan Pada tahap ini siswa mulai belajar mengenal suatu bangun geometri secara keseluruhan namun belum mampu mengetahui adanya sifat-sifat dari bangun geometri yang dilihatnya b Tahap Analisis Pada tahap ini siswa sudah mulai mengenal sifat-sifat yang dimiliki bangun geometri yang diamatinya c Tahap Pengurutan Pada tahap ini siswa sudah mengenal dan memahami sifat-sifat suatu bangun geometri serta sudah dapat mengurutkan bangun-bangun geometri yang satu sama yang lainnya saling berhubungan d Tahap Deduksi Pada tahap ini siswa telah mampu menarik kesimpulan secara deduktif yaitu menarik kesimpulan yang bersifat umum dan menuju ke hal yang bersifat khusus serta dapat mengambil kesimpulan e Tahap Akurasi Pada tahap ini siswa mulai menyadari pentingnya ketepatan prinsip-prinsip dasar yang melandasi suatu pembuktian Tahap berfikir ini merupakan tahap berfikir yang paling tinggi rumit dan kompleks karena di luar jangkauan usia anak-anak SD sampai tingakat SMP

49 Kd 311 Merancang aktivitas pembelajaran berdasarkan prinsip dan teori pembelajaran matematika middot Matematika untuk tujuan pembelajaran dianalisis sebagai kumpulan fakta yang berdiri sendiri dan tidak saling berkaitan middot Anak diharuskan menguasai unsur-unsur yang banyak sekali tanpa diperhatikan pengertiannya middot Anak mempelajari unsur-unsur dalam bentuk seperti yang akan digunakan nanti dalam kesempatan lain middot Anak akan mencapai tujuan ini secara efektif dan efisien dengan melalui pengulangan Teori Pembelajaran Matematika tersebut dikemukakan oleh a Skinner b Piaget c Van Brownell

d Thorndike Uraian Brownell mengemukakan tentang Teori Makna (Meaning Theory) sebagai pengganti Teori Latihan HafalUlangan (Drill

Theory) Intisari dari teori Drill adalah - Matematika untuk tujuan pembelajaran dianalisis sebagai kumpulan fakta yang berdiri sendiri dan tidak saling berkaitan - Anak diharuskan menguasai unsur-unsur yang banyak sekali tanpa diperhatikan pengertiannya - Anak mempelajari unsur-unsur dalam bentuk seperti yang akan digunakan nanti dalam kesempatan lain - Anak akan mencapai tujuan ini secara efektif dan efisien dengan melalui pengulangan

50 Kd 311 Merancang aktivitas pembelajaran berdasarkan prinsip dan teori pembelajaran matematika Tahap operasionaloperasi konkrit adalah teori Pembelajaran Matematika yang dikemukakan Peaget pada tahap usia a dibawah 2 tahun b 2-7 Tahun c 7-8 Tahun d 7-12 Tahun

Uraian Piaget membagi skema yang digunakan anak untuk memahami dunianya melalui empat periode utama yang berkorelasi

dengan dan semakin canggih seiring pertambahan usia Periode sensorimotor (usia 0ndash2 tahun) Periode praoperasional (usia 2ndash7 tahun) Periode operasional konkrit (usia 7ndash12 tahun) Periode operasional formal (usia 12 tahun sampai dewasa)

51 313 Memilih media pembelajaran yang tepat untuk pembelajaran operasi bilangan bulat Untuk operasi bilangan bulat perkalian media yang paling tepat digunakan adalah a sapu lidi

b kerikil c daun pakis d koin

Uraian 52 313 Memilih media pembelajaran yang tepat untuk pembelajaran operasi bilangan bulat

Untuk operasi bilangan bulat negatif kita bisa menggunakan media a uang kertas b kerikil c mistar d koin

Uraian 53 313 Memilih media pembelajaran yang tepat untuk pembelajaran operasi bilangan bulat

Beranda Soal UKG Agus Zainal M (SDN Gudang 2)

Republik_ilalangyahoocoid

Untuk operasi bilangan bulat penjumlahan media yang paling tepat digunakan adalah a uang kertas b kerikil c daun pakis d koin

Uraian 54 314 Memilih media pembelajaran yang tepat untuk pembelajaran operasi bilangan pecahan

Untuk operasi bilangan pecahan kita bisa menggunakan media berikut kecuali a kartu bilangan b garis bilangan c gambar bidang d blok pecahan

Uraian 55 316 Memilih media pembelajaran yang tepat untuk pembelajaran geometri dan pengukuran

Untuk menghitung luas dan keliling bangun datar media yang paling tepat digunakan adalah a Penggaris b Kertas Folio Bergaris

c Neraca d Kalkulator

Uraian

Page 7: 50 SOAL UKG & PEMBAHASAN SESUAI KISI-KISIS.pdf

Beranda Soal UKG Agus Zainal M (SDN Gudang 2)

Republik_ilalangyahoocoid

Membaca cerdas adalah membaca yang mengutamakan isi bacaan sebagai ungkapan pikiran perasaan dan kehendak penulis Bila hanya ingin mengetahui isinya membaca cerdas bersifat lugas Akan tetapi bial maksudnya untuk memahami dan memilki isi bacaan maka disebut membaca belajar 3) Membaca teknis Membaca teknis adalah membaca dengan mengarahkan bacaan secara wajar Wajar maksudnya sesuai ucapan tekanan dan intonasinya Pikiran perasaan dan kemauan yang tersimpan dalam bacaan dapat diaktualisasikan dengan baik 4) Membaca emosional Membaca emosional adalah membaca sebagai sarana untuk memasuki perasaan yaitu keindahan isi dan keindahan bahasanya 5) Membaca bebas Membaca bebas adalah membaca sesuatu atas kehendak sendiri tanpa adanya unsur paksaan dari luar Unsur dari luar misalnya guru orang tua teman atau pihak-pihak lain

26 KD 131 Merumuskan hakikat (pengertiantujuanjenismanfaat) membaca membaca sesuatu atas kehendak sendiri tanpa adanya unsur paksaan dari luar Unsur dari luar misalnya guru orang tua teman atau pihak-pihak lain termasuk jenis membaca a Membaca cerdas atau membaca dalam hati b Membaca bahasa c Membaca teknis d Membaca bebas

Uraian No 25 27 KD 131 Merumuskan hakikat (pengertiantujuanjenismanfaat) membaca

1 menggunakan ucapan yang tepat 2 menggunakan frase yang tepat 3 menggunakan intonasi suara yang wajar 4 dalam posisi sikap yang baik 5 menguasai tanda-tanda baca 6 membaca dengan terang dan jelas keterampilan di atas harus dipunyai dalam jenis membaca a Membaca cerdas atau membaca dalam hati b Membaca bahasa c Membaca teknis d Membaca nyaring

Uraian JENIS-JENIS MEMBACA DAN KARAKTERISTIKNYA

Ditinjau dari segi terdengar atau tidaknya suara pembaca waktu melakukan kegiatan membaca maka proses membaca dapat dibedakan menjadi A Membaca Nyaring Membaca nyaring adalah kegiatan membaca dengan menyuarakan tulisan yang dibacanya dengan ucapan dan intonasi yang tepat agar pendengar dan pembaca dapat menangkap informasi yang disampaikan oleh penulis baik yang berupa pikiran perasaan sikap ataupun pengalaman penulis Ketrampilan yang dituntut dalam membaca nyaring adalah berbagai kemampuan diantaranya adalah 1 menggunakan ucapan yang tepat 2 menggunakan frase yang tepat 3 menggunakan intonasi suara yang wajar 4 dalam posisi sikap yang baik 5 menguasai tanda-tanda baca 6 membaca dengan terang dan jelas 7 membaca dengan penuh perasaan ekspresif 8 membaca dengan tidak terbata-bata 9 mengerti serta memahami bahan bacaan yang dibacanya 10 kecepatan bergantung pada bahan bacaan yang dibacanya 11 membaca dengan tanpa terus-menerus melihat bahan bacaan 12 membaca dengan penuh kepercayaan pada diri sendiri B Membaca Dalam Hati Membaca dalam hati adalah kegiatan membaca yang dilakukan dengan tanpa menyuarakan isi bacaan yang dibacanya Ketrampilan yang dituntut dalam membaca dalam hati antara lain sebagai berikut 1 membaca tanpa bersuara tanpa bibir bergerak tanpa ada desis apapun 2 membaca tanpa ada gerakan-gerakan kepala 3 membaca lebih cepat dibandingkan dengan membaca nyaring 4 tanpa menggunakan jari atau alat lain sebagai penunjuk 5 mengerti dan memahami bahan bacaan 6 dituntut kecepatan mata dalam membaca 7 membaca dengan pemahaman yang baik

Beranda Soal UKG Agus Zainal M (SDN Gudang 2)

Republik_ilalangyahoocoid

8 dapat menyesuaikan kecepatan dengan tingkat kesukaran yang terdapat dalam bacaan 28 KD 131 Merumuskan hakikat (pengertiantujuanjenismanfaat) membaca

Hal berikut dilakukan seseorang ketika membaca survai kecuali a memeriksa judul bacaanbuku kata pengantar daftar isi dan malihat abstrak(jika ada) b memeriksa bagian terahkir dari isi (kesimpulan) jika ada c memeriksa indeks dan apendiks(jika ada) d membaca biografi pengarang

Uraian Secara garis besar membaca dalam hati dapat dibedakan menjadi dua (I) MEMBACA EKSTENSIF amp (II) MEMBACA

INTENSIF Berikut penjelasan secara rinci kedua jenis membaca tersebut I Membaca Ekstensif membaca ekstensif adalah membaca secara luas Objeknya meliputi sebanyak mungkin teks dalam waktu yang sesingkat-singkatnya Membaca ekstensif meliputi 1 Membaca Survai (Survey Reading) Membaca survai adalah kegiatan membaca untuk mengetahui secara sekilas terhadap bahan bacaan yang akan dibaca lebih mendalam Kegiatan membaca survai merupakan pendahuluan dalam membaca ekstensif Yang dilakukan seseorang ketika membaca survai adalah sebagai berikut (a) memeriksa judul bacaanbuku kata pengantar daftar isi dan malihat abstrak(jika ada) (b) memeriksa bagian terahkir dari isi (kesimpulan) jika ada (c) memeriksa indeks dan apendiks(jika ada)

2 Membaca Sekilas Membaca sekilas atau membaca cepat adalah kegiatan membaca dengan mengandalakan kecepatan gerak mata dalam melihat dan memperhatikan bahan tertulis yang dibacanya dengan tujuan untuk mendapatkan informasi secara cepat Metode yang digunakan dalam melatihkan membaca cepat adalah (a) metode kosakata metode yang berusaha untuk menambah kosakata (b) Metode motivasi metode yang berusaha memotivasi pembaca(pemula) yang mengalami hambatan (c) Metode gerak mata metode yang mengembangkan kecepatan membaca dengan menigkatkan kecepatan gerak mata Hambatan-hambatan yang dapat mengurangi kecepatan mambaca (a) vokalisai atau berguman ketika membaca (b) membaca dengan menggerakan bibir tetapi tidak bersuara (c) kepala bergerak searah tulisan yang dibaca (d) subvokalisasi suara yang biasa ikut membaca di dalam pikiran kita (e) jari tangan selalu menunjuk tulisa yang sedang kit abaca (f) gerakan mata kembali pada kata-kata sebelumnya 3 Membaca Dangkal (Superficial Reading) membaca dangkal pada hakekatnya bertujuan untuk memperoleh pemahaman yang dangkal yang bersifat luaran yang tidak mendalam dari suatu bahan bacaan Membaca jenis ini biasanya dilakukan seseorang membaca demi kesenangan membaca bacaan ringan yang mendatangkan kesenangan kegembiraan sebagai pengisi waktu senggang

29 KD 131 Merumuskan hakikat (pengertiantujuanjenismanfaat) membaca Membaca jenis ini biasanya dilakukan seseorang membaca demi kesenangan membaca bacaan ringan yang mendatangkan kesenangan kegembiraan sebagai pengisi waktu senggang Berdasarkan karakteristik diatas kegiatan tersebut termasuk ke dalam membaca jenis a Membaca Survai (Survey Reading) b Membaca Sekilas c Membaca Dangkal (Superficial Reading)

d Membaca Nyaring Uraian No 28 30 KD 131 Merumuskan hakikat (pengertiantujuanjenismanfaat) membaca

Berikut adalah tujuan umum dalam aktifitas membaca kecuali a Membaca untuk memperoleh perincian-perincian atau fakta-fakta (reading for details or facts) Membaca tersebut

bertujuan untuk menemukan atau mengetahui penemuan-penemuan telah dilakukan oleh sang tokoh untuk memecahkan masalah-masalah yang dibuat oleh sang tokoh

b Membaca untuk mengetahui ukuran atau susunan organisasi cerita (reading for sequenceor organization) Membaca tersebut bertujuan untuk mengetahui bagian-bagian cerita dan hubungan antar bagian-bagian cerita

c Membaca untuk menyimpulkan atau membaca inferensi (reading for inference) d Membaca untuk memperoleh kekurangan suatu buku (finding mistakes)

Uraian Tujuan Membaca

Beranda Soal UKG Agus Zainal M (SDN Gudang 2)

Republik_ilalangyahoocoid

Berdasarkan maksud tujuan atau keintensifan serta cara dalam membaca di bawah ini Anderson dalam Tarigan (19799-10) mengemukakan beberapa tujuan membaca antara lain a Membaca untuk memperoleh perincian-perincian atau fakta-fakta (reading for details or facts) Membaca tersebut bertujuan untuk menemukan atau mengetahui penemuan-penemuan telah dilakukan oleh sang tokoh untuk memecahkan masalah-masalah yang dibuat oleh sang tokoh b Membaca untuk memperoleh ide-ide utama (reading for main ideas) Membaca untuk mengetahui topik atau masalah dalam bacaan Untuk menemukan ide pokok bacaan dengan membaca halamn demi halaman c Membaca untuk mengetahui ukuran atau susunan organisasi cerita (reading for sequenceor organization) Membaca tersebut bertujuan untuk mengetahui bagian-bagian cerita dan hubungan antar bagian-bagian cerita d Membaca untuk menyimpulkan atau membaca inferensi (reading for inference) Pembaca diharapkan dapat merasakan sesuatu yang dirasakan penulis e Membaca untuk mengelompokkan atau mengklasifikasikan (reading for classify) Membaca jenis ini bertujuan untuk menemukan hal-hal yang tidak wajar mengenai sesuatu hal (Anderson dalam Tarigan 197910) f Membaca untuk menilai atau mengevaluasai (reading to evaluate) Jenis membaca tersebut bertujuan menemukan suatu keberhasilan berdasarkan ukuran-ukuran tertentu Membaca jenis ini memerlukan ketelitian dengan membandingkan dan mengujinya kembali g Membaca untuk memperbandingkan atau mempertentangkan (reading to compare or contrast) Tujuan membaca tersebut adalah untuk menemukan bagaimana cara perbedaan atau persamaan dua hal atau lebih

31 KD 133 menemukan pesan pokok utama sebuah berita Simak penggalan berita berikut Hebat Siswa Indonesia Pertahankan Tradisi Emas di Olimpiade Fisika Tim Olimpiade Fisika Indonesia REPUBLIKACOID JAKARTA--Luar biasa Lima siswa Indonesia yang dikirim ke ajang Olimpiade Fisika atau International Physics Olympiad (IPhO) ke-41 di Zagreb Kroasia 17-25 Juli berhasil menyabet empat medali emas dan satu perak Pelajar yang menyumbang emas adalah Muhammad Sohibul Maromi (SMAN 1 Pamekasan Madura) Christian George Emor (SMA Lokon St Nikolaus Tomohon Sulawesi Utara) David Giovanni (SMAK Penabur Gading Serpong Banten) dan Kevin Soedyatmiko (SMAN 12 Jakarta) Sedangkan medali perak berhasil diraih oleh Ahmad Ataka Awwalur Rizqi (SMAN 1 Yogyakarta) Prestasi ini jauh lebih baik dibanding ajang Olimpiade Fisika ke-40 di Merida Yucatan Meksiko 2009 yang lalu Saat itu delegasi siswa Indonesia merebut satu medali emas dua medali perak dan satu perunggu Hasil empat medali emas dan satu perak ini hampir menyamai prestasi terbaik sebelumnya pada ajang Olimpiade Fisika ke-37 di Singapura Saat itu siswa Indonesia tidak hanya berhasil menyabet 4 medali emas namun juga meraih predikat `Absolute Winner` atas nama Mailoa Jonathan Pradana (SMAK 1 BPK Penabur Jakarta) Tapi yang terpenting lagi pelajar Indonesia berhasil mempertahankan tradisi emas di setiap ajang Olimpiade Fisika Red Endro Yuwanto Peristiwa yang diberitakan adalah a siswa Indonesia pertahankan tradisi emas di Olimpiade Matematika b siswa Indonesia pertahankan tradisi emas di Olimpiade Fisika c siswa Indonesia menyumbang emas di Olimpiade Matematika

d siswa Indonesia menyumbang perak di Olimpiade Fisika Uraian 32 KD 133 menemukan pesan pokok utama sebuah berita

Simak penggalan berita berikut Hebat Siswa Indonesia Pertahankan Tradisi Emas di Olimpiade Fisika Tim Olimpiade Fisika Indonesia REPUBLIKACOID JAKARTA--Luar biasa Lima siswa Indonesia yang dikirim ke ajang Olimpiade Fisika atau International Physics Olympiad (IPhO) ke-41 di Zagreb Kroasia 17-25 Juli berhasil menyabet empat medali emas dan satu perak Pelajar yang menyumbang emas adalah Muhammad Sohibul Maromi (SMAN 1 Pamekasan Madura) Christian George Emor (SMA Lokon St Nikolaus Tomohon Sulawesi Utara) David Giovanni (SMAK Penabur Gading Serpong Banten) dan Kevin Soedyatmiko (SMAN 12 Jakarta) Sedangkan medali perak berhasil diraih oleh Ahmad Ataka Awwalur Rizqi (SMAN 1 Yogyakarta) Prestasi ini jauh lebih baik dibanding ajang Olimpiade Fisika ke-40 di Merida Yucatan Meksiko 2009 yang lalu Saat itu delegasi siswa Indonesia merebut satu medali emas dua medali perak dan satu perunggu Hasil empat medali emas dan satu perak ini hampir menyamai prestasi terbaik sebelumnya pada ajang Olimpiade Fisika ke-37 di Singapura Saat itu siswa Indonesia tidak hanya berhasil menyabet 4 medali emas namun juga meraih predikat `Absolute Winner` atas nama Mailoa Jonathan Pradana (SMAK 1 BPK Penabur Jakarta) Tapi yang terpenting lagi pelajar Indonesia berhasil mempertahankan tradisi emas di setiap ajang Olimpiade Fisika Red Endro Yuwanto Lima siswa Indonesia berhasil mendapatkan a 4 emas 1 perak

b 1 emas 4 perak c 3 emas 2 perak d 2 perak 3 emas

Beranda Soal UKG Agus Zainal M (SDN Gudang 2)

Republik_ilalangyahoocoid

Uraian 33 KD 134 menemukan pesan pokok dalam wacana naratif seperti cerita rakyat puisi

Surat dari ibu Jika bayang telah pudar Dan elang laut pulang ke sarang Angin bertiup ke benua Tiang ndash tiang akan kering sendiri Dan nahkoda sudah tau pedoman Boleh engkau datang padaku Makna lambang dari nahkoda sudah tahu pedoman adalahhellip a Sudah mencari pedoman hidup b Sudah menemukan arah dan tujuan

c Sudah berilmu dan berpengalaman d Sudah mempunyai pasangan hidup

Uraian 34 KD 135 membandingkan berbagai jenis wacana bahasa indonesia (deskripsi narasi)

Pendekatan untuk mendapat tanggapan emosional pembaca ataupun kesan pembaca adalah contoh Pendekatan Deskripsi jenis a Pendekatan Ekspositoris b Pendekatan Impresionistik

c Pendekatan menurut sikap pengarang d Pendekatan Realistik

Uraian PENDEKATAN DESKRIPSI

Pendekatan dalam pendeskripsian dapat dibedakan menjadi beberapa kategori pendekatan yaitu

1 Pendekatan Ekspositoris

Dalam pendekatan ini kita berusaha agar deskripsi yang kita buat dapat memberi keterangan sesuai dengan keadan yang

sebenarnya sehingga pembaca dapat seolah-olah ikut melihat atau merasakan objek yang kita deskripsikan Karangan

jenis ini berisi daftar detail sesuatu secara lengkap sehingga pembaca dan penalarannya dapat memperoleh kesan

keseluruhan tentang sesuatu

2 Pendekatan Impresionistik

Tujuan deskripsi impresionistik ialah untuk mendapatkan tanggapan emosional pembaca ataupun kesan pembaca Corak

deskripsi ini diantaranya juga ditentukan oleh macam kesan apa yang diinginkan penulisnya

3 Pendekatan menurut Sikap Pengarang

Pendekatan ini sangat bergantung pada tujuan yang ingin dicapai sifat objek serta pembaca deskripsinya Dalam

menguraikan sebuah gagasan penulis mungkin mengharapkan agar pembaca merasa tidak puas terhadapa suatu

tindakan atau keadaan atau penulis menginginkan agar pembaca juga harus merasakan bahwa persoalan yang dihadapi

merupakan masalah yang gawat Penulis juga dapat membayangkan bahwa akan terjadi sesuatu yang tidak diinginkan

sehingga pembaca dari semula sudah disiapkan perasaan yang kurang enak seram takut dan sebagainya (Alkhaidah

1997) 35 kd 135 membandingkan berbagai jenis wacana bahasa indonesia (deskripsi narasi)

Prinsip-Prinsip karangan Narasi sebagai berikut kecuali a Alur b Penokohan c Amanat

d Sudut Pandang Uraian Prinsip-prinsip dasar narasi sebagai tumpuan berfikir adalah sebagai berikut

1 Alur (plot) biasa disebut dengan jalan cerita

2 Penokohan (rangkaian perbuatanaksi) 3 Latar (setting) yaitu tempat dan atau waktu terjadinya perbuatan tokoh atau peristiwa yang dialami tokoh 4 Sudut Pandang (Point of View)

a Narrator Serba Tahu (Omniscient point of view) Yaitu narrator bertindak sebagai pencipta segalanya b Narrator bertindak objektif (Objektive point of view) Pengarang tak member komentar apa pun dan pembaca bebas menafsirkan apa yang diceritakan pengarang c Narrator ikut aktif ( Narator acting) Narrator terlibat dalam cerita d Narrator sebagai peninjau

Beranda Soal UKG Agus Zainal M (SDN Gudang 2)

Republik_ilalangyahoocoid

Pelakunya adalah orang ketiga yang biasa disebut ldquodiardquo 36 KD 135 membandingkan berbagai jenis wacana bahasa indonesia (deskripsi narasi)

1 Menentukan tema atau amanat apa yang akan disampaikan 2 Menetapkan sasaran pembaca 3 Merancang peristiwa-peristiwa utama yang akan ditampilkan dalam bentuk skema perkembangandan akhir cerita 5 Memerinci peristiwa-peristiwa utama ke dalam detail-detail peristiwa sebagai pendukung cerita 6 Menyusun tokoh dan perwatakanserta latar dan sudut pandang Karakteristik diatas adalah pengembangan dari paragraf a Narasi

b Deskprisi c Ekspositoris d Sudut Pandang Persuasif

Uraian Pengembangan karangan narasi dapat dilakukan dengan langkah-langkah berikut

(1) menentukan tema atau amanat apa yang akan disampaikan (2) menetapkan sasaran pembaca (3) merancang peristiwa-peristiwa utama yang akan ditampilkan dalam bentuk skema alur (4) membagi peristiwa utama ke dalam ke dalam bagian awal perkembangan dan akhir cerita (5) memerinci peristiwa-peristiwa utama ke dalam detail-detail peristiwa sebagai pendukung cerita dan (6) menyusun tokoh dan perwatakan latar dan sudut pandang

37 KD 135 membandingkan berbagai jenis wacana bahasa indonesia (deskripsi narasi) Karangan narasi yang berusaha untuk memberikan suatu maksud tertentu menyampaikan suatu amanat terselubung kepada para pembaca atau pendengar sehingga tampak seolah-olah melihat disebut a Narasi Informatif b Narasi Ekspositoris c Narasi Sugestif d Narasi Artistik

Uraian Jenis-Jenis Narasi

Narasi Informatif Narasi informatif adalah narasi yang memiliki sasaran penyampaian informasi secara tepat tentang suatu peristiwa

dengan tujuan memperluas pengetahuan orang tentang kisah seseorang Narasi Ekspositorik

Narasi ekspositorik adalah narasi yang memiliki sasaran penyampaian informasi secara tepat tentang suatu peristiwa dengan tujuan memperluas pengetahuan orang tentang kisah seseorang Dalam narasi ekspositorik penulis menceritakan suatu peristiwa berdasarkan data yang sebenarnya Pelaku yang ditonjolkan biasanya satu orang Pelaku diceritakan mulai dari kecil sampai saat ini atau sampai terakhir dalam kehidupannya Karangan narasi ini diwarnai oleh eksposisi maka ketentuan eksposisi juga berlaku pada penulisan narasi ekspositprik Ketentuan ini berkaitan dengan penggunaan bahasa yang logis berdasarkan fakta yang ada tidak memasukan unsursugestif atau bersifat objektif

Narasi Artistik

Narasi artistik adalah narasi yang berusaha untuk memberikan suatu maksud tertentu menyampaikan suatu amanat terselubung kepada para pembaca atau pendengar sehingga tampak seolah-olah melihat Ketentuan ini berkaitan dengan penggunaan bahasa yang logis berdasarkan fakta yang ada tidak memasukan unsur sugestif atau bersifat objektif

Narasi Sugestif Narasi sugestif adalah narasi yang berusaha untuk memberikan suatu maksud tertentu menyampaikan suatu amanat

terselubung kepada para pembaca atau pendengar sehingga tampak seolah-olah melihat 38 KD 135 membandingkan berbagai jenis wacana bahasa indonesia (deskripsi narasi)

Ciri karangan Narasi yang benar kecuali a Menonjolkan unsur perbuatan atau tindakan Membuat pembaca atau pendengar merasakan sendiri atau

mengalami sendiri

b Ada konfiks menjawab pertanyaan apa yang terjadi c Dirangkai dalam urutan waktu menggambarkan dengan jelas suatu peristiwa d Berisi ajakan dirangkai dalam urutan waktu

Uraian Ciri-Ciri Karangan Narasi Menurut Gorys Keraf (2000136)

Menonjolkan unsur perbuatan atau tindakan Dirangkai dalam urutan waktu Berusaha menjawab pertanyaan apa yang terjadi Ada konfiks

39 kd 135 membandingkan berbagai jenis wacana bahasa indonesia (deskripsi narasi) Langkah menyusun paragraf deskripsi yang benar adalah (1)Tentukan objek atau tema yang akan dideskripsikan (2)Mengumpulkan data dengan mengamati objek yang akan dideskripsikan (3)Menyusun data tersebut ke dalam urutan yang baik (menyusun kerangka karangan) (4)Menguraikan kerangka karangan menjadi dekripsi yang sesuai dengan tema yang ditentukan

Beranda Soal UKG Agus Zainal M (SDN Gudang 2)

Republik_ilalangyahoocoid

(5)Tentukan tujuan a 1-5-2-3-4

b 1-5-4-3-2 c 1-5-3-2-4 d 1-3-4-2-5

Uraian Langkah menyusun deskripsi

1Tentukan objek atau tema yang akan dideskripsikan 2Tentukan tujuan 3Mengumpulkan data dengan mengamati objek yang akan dideskripsikan 4Menyusun data tersebut ke dalam urutan yang baik (menyusun kerangka karangan) 5Menguraikan kerangka karangan menjadi dekripsi yang sesuai dengan tema yang ditentukan

40 Kd 311 Merancang aktivitas pembelajaran berdasarkan prinsip dan teori pembelajaran matematika Dalam tahap ini penyajian yang dilakukan melalui tindakan anak secara langsung terlihat dalam memanipulasi (mengotak atik)objek Proses tersebut menurut Bruner dikategorikan ke dalam model a Tahap Enaktif

b Tahap Ikonik c Tahap Simbolik d Tahap Implikatif

Uraian 1 Model Tahap Enaktif

Dalam tahap ini penyajian yang dilakukan melalui tindakan anak secara langsung terlihat dalam memanipulasi (mengotak atik)objek Contoh Budi mempunyai 2 pensil kemudian ibunya memberikannya lagi 3 pinsil Berapa banyak pensil Budi sekarang

2 Model Tahap Ikonik Dalam tahap ini kegiatan penyajian dilakukan berdasarkan pada pikiran internal dimana pengetahuan disajikan melalui serangkaian gambar-gambar atau grafik yang dilakukan anak berhubungan dengan mental yang merupakan gambaran dari objek-objek yang dimanipulasinya Contoh + = hellip

3 Model Tahap Simbolis Dalam tahap ini bahasa adalah pola dasar simbolik anak memanipulasi Simbol-simbol atau lambang-lambang objek tertentu Contoh 2 pensil + 3 pensil = hellip pensil

41 Kd 311 Merancang aktivitas pembelajaran berdasarkan prinsip dan teori pembelajaran matematika Dalam tahap ini kegiatan penyajian dilakukan berdasarkan pada pikiran internal dimana pengetahuan disajikan melalui serangkaian gambar-gambar atau grafik yang dilakukan anak Proses tersebut menurut Bruner dikategorikan ke dalam model a Tahap Enaktif b Tahap Ikonik

c Tahap Simbolik d Tahap Implikatif

Uraian No 40 42 Kd 311 Merancang aktivitas pembelajaran berdasarkan prinsip dan teori pembelajaran matematika

Objek belajar matematika dibagi kedalam Objek Langsung dan Objek Tak Langsung Hal tersebut adalah teori belajar Matematika menurut a Robert M Gagne

b Jerome S Burner c Thorndike d Skinner

Uraian Teori yang diperkenalkan Robert MGagne pada tahun 1960-an pembelajaran harus dikondisikan untuk memunculkan

respons yang diharapkanMenurut Gagne (dalam Ismail 1998) belajar matematika terdiri dari objek langsung dan objek tak langsung

43 Kd 311 Merancang aktivitas pembelajaran berdasarkan prinsip dan teori pembelajaran matematika Perhatikan contoh berikut Budi mempunyai 2 pinsil kemudian ibunya memberikannya lagi 3 pinsilBerapa banyak pinsil Budi sekarang Hal tersebut dikemukakan Bruner dalam Proses Pembelajaran Matematika dalam tahap a Simbolik b Ikonik c Implikatif d Enaktif

Uraian No 40

Beranda Soal UKG Agus Zainal M (SDN Gudang 2)

Republik_ilalangyahoocoid

44 Kd 311 Merancang aktivitas pembelajaran berdasarkan prinsip dan teori pembelajaran matematika Perhatikan gambar disamping Ilustrasi di samping dikemukakan Bruner dalam Proses Pembelajaran Matematika dalam tahap

a Simbolik b Ikonik

c Implikatif d Enaktif

Uraian No 40 45 Kd 311 Merancang aktivitas pembelajaran berdasarkan prinsip dan teori pembelajaran matematika

Perhatikan contoh berikut Contoh 2 pinsil + 3 pinsil = hellippinsil Contoh tersebut dikemukakan Bruner dalam Proses Pembelajaran Matematika dalam tahap a Simbolik

b Ikonik c Implikatif d Enaktif

Uraian No 40 46 Kd 311 Merancang aktivitas pembelajaran berdasarkan prinsip dan teori pembelajaran matematika

Perhatikan contoh berikut penjumlahan bilangan positif dan negatif siswa mencoba sendiri dengan menggunakan garis bilangan Contoh tersebut dikemukakan Bruner dalam Teorema Pembelajaran Matematika a Penyusunan

b Notasi c Pengkontrasan dan Keanekaragaman d Pengaitan

Uraian Berdasarkan hasil pengamatannya Brunner merumuskan 5 teorema dalam pembelajaran matematika yaitu

1) Teorema Penyusunan Menerangkan bahwa cara yang terbaik memulai belajar suatu konsep matematika dalil defenisi dan semacamnya adalah dengan cara menyusun penyajiannya Misalnya dalam mempelajari penjumlahan bilangan positif dan negatif siswa mencoba sendiri dengan menggunakan garis bilangan 2) Teorema Notasi Menerangkan bahwa dalam pengajaran suatu konsep penggunaan notasi-notasi matematika harus diberikan secara bertahap dari yang sederhana ke yang lebih kompleks 3) Teorema Pengkontrasan dan Keanekaragaman Menerangkan bahwa pengontrasan dan keanekaragaman sangat penting dalam melakukan pengubahan konsep matematika dari yang konkrit ke yang lebih abstrak Dalam hal ini diperlukan banyak contoh Contoh yang diberikan harus sesuai dengan rumusan yang diberikan Misalnya menjelaskan persegi panjang disertai juga kemungkinan jajaran genjang dan segi empat lainnya selain persegi panjnag Dengan demikian siswa dapat membedakan apakah segi empat yang diberikan padanya termasuk persegi panjang atau tidak 4) Teorema Pengaitan Menerangkan bahwa dalam matematika terdapat hubungan yang berkaitan antara satu konsep dengan konsep yang lain Di mana materi yang satu merupakan prasyarat yang harus diketahui untuk mempelajari materi yang lain

47 Kd 311 Merancang aktivitas pembelajaran berdasarkan prinsip dan teori pembelajaran matematika Perhatikan contoh berikut Guru menjelaskan persegi panjang disertai juga kemungkinan jajaran genjang dan segi empat lainnya selain persegi panjnag Dengan demikian siswa dapat membedakan apakah segi empat yang diberikan padanya termasuk persegi panjang atau tidak Contoh tersebut dikemukakan Bruner dalam Teorema Pembelajaran Matematika a Penyusunan b Notasi c Pengkontrasan dan Keanekaragaman

d Pengaitan Uraian No 46 48 Kd 311 Merancang aktivitas pembelajaran berdasarkan prinsip dan teori pembelajaran matematika

Tahap pembelajaran Matematika menurut Van Halle adalah sebagai berikut kecuali a Tahap Pengenalan b Tahap Pengurutan c Tahap Analisis d Tahap Penyimpulan

Beranda Soal UKG Agus Zainal M (SDN Gudang 2)

Republik_ilalangyahoocoid

Uraian Van Hiele menyatakan bahwa terdapat 5 tahap belajar siswa dalam belajar geometri yaitu

a Tahap Pengenalan Pada tahap ini siswa mulai belajar mengenal suatu bangun geometri secara keseluruhan namun belum mampu mengetahui adanya sifat-sifat dari bangun geometri yang dilihatnya b Tahap Analisis Pada tahap ini siswa sudah mulai mengenal sifat-sifat yang dimiliki bangun geometri yang diamatinya c Tahap Pengurutan Pada tahap ini siswa sudah mengenal dan memahami sifat-sifat suatu bangun geometri serta sudah dapat mengurutkan bangun-bangun geometri yang satu sama yang lainnya saling berhubungan d Tahap Deduksi Pada tahap ini siswa telah mampu menarik kesimpulan secara deduktif yaitu menarik kesimpulan yang bersifat umum dan menuju ke hal yang bersifat khusus serta dapat mengambil kesimpulan e Tahap Akurasi Pada tahap ini siswa mulai menyadari pentingnya ketepatan prinsip-prinsip dasar yang melandasi suatu pembuktian Tahap berfikir ini merupakan tahap berfikir yang paling tinggi rumit dan kompleks karena di luar jangkauan usia anak-anak SD sampai tingakat SMP

49 Kd 311 Merancang aktivitas pembelajaran berdasarkan prinsip dan teori pembelajaran matematika middot Matematika untuk tujuan pembelajaran dianalisis sebagai kumpulan fakta yang berdiri sendiri dan tidak saling berkaitan middot Anak diharuskan menguasai unsur-unsur yang banyak sekali tanpa diperhatikan pengertiannya middot Anak mempelajari unsur-unsur dalam bentuk seperti yang akan digunakan nanti dalam kesempatan lain middot Anak akan mencapai tujuan ini secara efektif dan efisien dengan melalui pengulangan Teori Pembelajaran Matematika tersebut dikemukakan oleh a Skinner b Piaget c Van Brownell

d Thorndike Uraian Brownell mengemukakan tentang Teori Makna (Meaning Theory) sebagai pengganti Teori Latihan HafalUlangan (Drill

Theory) Intisari dari teori Drill adalah - Matematika untuk tujuan pembelajaran dianalisis sebagai kumpulan fakta yang berdiri sendiri dan tidak saling berkaitan - Anak diharuskan menguasai unsur-unsur yang banyak sekali tanpa diperhatikan pengertiannya - Anak mempelajari unsur-unsur dalam bentuk seperti yang akan digunakan nanti dalam kesempatan lain - Anak akan mencapai tujuan ini secara efektif dan efisien dengan melalui pengulangan

50 Kd 311 Merancang aktivitas pembelajaran berdasarkan prinsip dan teori pembelajaran matematika Tahap operasionaloperasi konkrit adalah teori Pembelajaran Matematika yang dikemukakan Peaget pada tahap usia a dibawah 2 tahun b 2-7 Tahun c 7-8 Tahun d 7-12 Tahun

Uraian Piaget membagi skema yang digunakan anak untuk memahami dunianya melalui empat periode utama yang berkorelasi

dengan dan semakin canggih seiring pertambahan usia Periode sensorimotor (usia 0ndash2 tahun) Periode praoperasional (usia 2ndash7 tahun) Periode operasional konkrit (usia 7ndash12 tahun) Periode operasional formal (usia 12 tahun sampai dewasa)

51 313 Memilih media pembelajaran yang tepat untuk pembelajaran operasi bilangan bulat Untuk operasi bilangan bulat perkalian media yang paling tepat digunakan adalah a sapu lidi

b kerikil c daun pakis d koin

Uraian 52 313 Memilih media pembelajaran yang tepat untuk pembelajaran operasi bilangan bulat

Untuk operasi bilangan bulat negatif kita bisa menggunakan media a uang kertas b kerikil c mistar d koin

Uraian 53 313 Memilih media pembelajaran yang tepat untuk pembelajaran operasi bilangan bulat

Beranda Soal UKG Agus Zainal M (SDN Gudang 2)

Republik_ilalangyahoocoid

Untuk operasi bilangan bulat penjumlahan media yang paling tepat digunakan adalah a uang kertas b kerikil c daun pakis d koin

Uraian 54 314 Memilih media pembelajaran yang tepat untuk pembelajaran operasi bilangan pecahan

Untuk operasi bilangan pecahan kita bisa menggunakan media berikut kecuali a kartu bilangan b garis bilangan c gambar bidang d blok pecahan

Uraian 55 316 Memilih media pembelajaran yang tepat untuk pembelajaran geometri dan pengukuran

Untuk menghitung luas dan keliling bangun datar media yang paling tepat digunakan adalah a Penggaris b Kertas Folio Bergaris

c Neraca d Kalkulator

Uraian

Page 8: 50 SOAL UKG & PEMBAHASAN SESUAI KISI-KISIS.pdf

Beranda Soal UKG Agus Zainal M (SDN Gudang 2)

Republik_ilalangyahoocoid

8 dapat menyesuaikan kecepatan dengan tingkat kesukaran yang terdapat dalam bacaan 28 KD 131 Merumuskan hakikat (pengertiantujuanjenismanfaat) membaca

Hal berikut dilakukan seseorang ketika membaca survai kecuali a memeriksa judul bacaanbuku kata pengantar daftar isi dan malihat abstrak(jika ada) b memeriksa bagian terahkir dari isi (kesimpulan) jika ada c memeriksa indeks dan apendiks(jika ada) d membaca biografi pengarang

Uraian Secara garis besar membaca dalam hati dapat dibedakan menjadi dua (I) MEMBACA EKSTENSIF amp (II) MEMBACA

INTENSIF Berikut penjelasan secara rinci kedua jenis membaca tersebut I Membaca Ekstensif membaca ekstensif adalah membaca secara luas Objeknya meliputi sebanyak mungkin teks dalam waktu yang sesingkat-singkatnya Membaca ekstensif meliputi 1 Membaca Survai (Survey Reading) Membaca survai adalah kegiatan membaca untuk mengetahui secara sekilas terhadap bahan bacaan yang akan dibaca lebih mendalam Kegiatan membaca survai merupakan pendahuluan dalam membaca ekstensif Yang dilakukan seseorang ketika membaca survai adalah sebagai berikut (a) memeriksa judul bacaanbuku kata pengantar daftar isi dan malihat abstrak(jika ada) (b) memeriksa bagian terahkir dari isi (kesimpulan) jika ada (c) memeriksa indeks dan apendiks(jika ada)

2 Membaca Sekilas Membaca sekilas atau membaca cepat adalah kegiatan membaca dengan mengandalakan kecepatan gerak mata dalam melihat dan memperhatikan bahan tertulis yang dibacanya dengan tujuan untuk mendapatkan informasi secara cepat Metode yang digunakan dalam melatihkan membaca cepat adalah (a) metode kosakata metode yang berusaha untuk menambah kosakata (b) Metode motivasi metode yang berusaha memotivasi pembaca(pemula) yang mengalami hambatan (c) Metode gerak mata metode yang mengembangkan kecepatan membaca dengan menigkatkan kecepatan gerak mata Hambatan-hambatan yang dapat mengurangi kecepatan mambaca (a) vokalisai atau berguman ketika membaca (b) membaca dengan menggerakan bibir tetapi tidak bersuara (c) kepala bergerak searah tulisan yang dibaca (d) subvokalisasi suara yang biasa ikut membaca di dalam pikiran kita (e) jari tangan selalu menunjuk tulisa yang sedang kit abaca (f) gerakan mata kembali pada kata-kata sebelumnya 3 Membaca Dangkal (Superficial Reading) membaca dangkal pada hakekatnya bertujuan untuk memperoleh pemahaman yang dangkal yang bersifat luaran yang tidak mendalam dari suatu bahan bacaan Membaca jenis ini biasanya dilakukan seseorang membaca demi kesenangan membaca bacaan ringan yang mendatangkan kesenangan kegembiraan sebagai pengisi waktu senggang

29 KD 131 Merumuskan hakikat (pengertiantujuanjenismanfaat) membaca Membaca jenis ini biasanya dilakukan seseorang membaca demi kesenangan membaca bacaan ringan yang mendatangkan kesenangan kegembiraan sebagai pengisi waktu senggang Berdasarkan karakteristik diatas kegiatan tersebut termasuk ke dalam membaca jenis a Membaca Survai (Survey Reading) b Membaca Sekilas c Membaca Dangkal (Superficial Reading)

d Membaca Nyaring Uraian No 28 30 KD 131 Merumuskan hakikat (pengertiantujuanjenismanfaat) membaca

Berikut adalah tujuan umum dalam aktifitas membaca kecuali a Membaca untuk memperoleh perincian-perincian atau fakta-fakta (reading for details or facts) Membaca tersebut

bertujuan untuk menemukan atau mengetahui penemuan-penemuan telah dilakukan oleh sang tokoh untuk memecahkan masalah-masalah yang dibuat oleh sang tokoh

b Membaca untuk mengetahui ukuran atau susunan organisasi cerita (reading for sequenceor organization) Membaca tersebut bertujuan untuk mengetahui bagian-bagian cerita dan hubungan antar bagian-bagian cerita

c Membaca untuk menyimpulkan atau membaca inferensi (reading for inference) d Membaca untuk memperoleh kekurangan suatu buku (finding mistakes)

Uraian Tujuan Membaca

Beranda Soal UKG Agus Zainal M (SDN Gudang 2)

Republik_ilalangyahoocoid

Berdasarkan maksud tujuan atau keintensifan serta cara dalam membaca di bawah ini Anderson dalam Tarigan (19799-10) mengemukakan beberapa tujuan membaca antara lain a Membaca untuk memperoleh perincian-perincian atau fakta-fakta (reading for details or facts) Membaca tersebut bertujuan untuk menemukan atau mengetahui penemuan-penemuan telah dilakukan oleh sang tokoh untuk memecahkan masalah-masalah yang dibuat oleh sang tokoh b Membaca untuk memperoleh ide-ide utama (reading for main ideas) Membaca untuk mengetahui topik atau masalah dalam bacaan Untuk menemukan ide pokok bacaan dengan membaca halamn demi halaman c Membaca untuk mengetahui ukuran atau susunan organisasi cerita (reading for sequenceor organization) Membaca tersebut bertujuan untuk mengetahui bagian-bagian cerita dan hubungan antar bagian-bagian cerita d Membaca untuk menyimpulkan atau membaca inferensi (reading for inference) Pembaca diharapkan dapat merasakan sesuatu yang dirasakan penulis e Membaca untuk mengelompokkan atau mengklasifikasikan (reading for classify) Membaca jenis ini bertujuan untuk menemukan hal-hal yang tidak wajar mengenai sesuatu hal (Anderson dalam Tarigan 197910) f Membaca untuk menilai atau mengevaluasai (reading to evaluate) Jenis membaca tersebut bertujuan menemukan suatu keberhasilan berdasarkan ukuran-ukuran tertentu Membaca jenis ini memerlukan ketelitian dengan membandingkan dan mengujinya kembali g Membaca untuk memperbandingkan atau mempertentangkan (reading to compare or contrast) Tujuan membaca tersebut adalah untuk menemukan bagaimana cara perbedaan atau persamaan dua hal atau lebih

31 KD 133 menemukan pesan pokok utama sebuah berita Simak penggalan berita berikut Hebat Siswa Indonesia Pertahankan Tradisi Emas di Olimpiade Fisika Tim Olimpiade Fisika Indonesia REPUBLIKACOID JAKARTA--Luar biasa Lima siswa Indonesia yang dikirim ke ajang Olimpiade Fisika atau International Physics Olympiad (IPhO) ke-41 di Zagreb Kroasia 17-25 Juli berhasil menyabet empat medali emas dan satu perak Pelajar yang menyumbang emas adalah Muhammad Sohibul Maromi (SMAN 1 Pamekasan Madura) Christian George Emor (SMA Lokon St Nikolaus Tomohon Sulawesi Utara) David Giovanni (SMAK Penabur Gading Serpong Banten) dan Kevin Soedyatmiko (SMAN 12 Jakarta) Sedangkan medali perak berhasil diraih oleh Ahmad Ataka Awwalur Rizqi (SMAN 1 Yogyakarta) Prestasi ini jauh lebih baik dibanding ajang Olimpiade Fisika ke-40 di Merida Yucatan Meksiko 2009 yang lalu Saat itu delegasi siswa Indonesia merebut satu medali emas dua medali perak dan satu perunggu Hasil empat medali emas dan satu perak ini hampir menyamai prestasi terbaik sebelumnya pada ajang Olimpiade Fisika ke-37 di Singapura Saat itu siswa Indonesia tidak hanya berhasil menyabet 4 medali emas namun juga meraih predikat `Absolute Winner` atas nama Mailoa Jonathan Pradana (SMAK 1 BPK Penabur Jakarta) Tapi yang terpenting lagi pelajar Indonesia berhasil mempertahankan tradisi emas di setiap ajang Olimpiade Fisika Red Endro Yuwanto Peristiwa yang diberitakan adalah a siswa Indonesia pertahankan tradisi emas di Olimpiade Matematika b siswa Indonesia pertahankan tradisi emas di Olimpiade Fisika c siswa Indonesia menyumbang emas di Olimpiade Matematika

d siswa Indonesia menyumbang perak di Olimpiade Fisika Uraian 32 KD 133 menemukan pesan pokok utama sebuah berita

Simak penggalan berita berikut Hebat Siswa Indonesia Pertahankan Tradisi Emas di Olimpiade Fisika Tim Olimpiade Fisika Indonesia REPUBLIKACOID JAKARTA--Luar biasa Lima siswa Indonesia yang dikirim ke ajang Olimpiade Fisika atau International Physics Olympiad (IPhO) ke-41 di Zagreb Kroasia 17-25 Juli berhasil menyabet empat medali emas dan satu perak Pelajar yang menyumbang emas adalah Muhammad Sohibul Maromi (SMAN 1 Pamekasan Madura) Christian George Emor (SMA Lokon St Nikolaus Tomohon Sulawesi Utara) David Giovanni (SMAK Penabur Gading Serpong Banten) dan Kevin Soedyatmiko (SMAN 12 Jakarta) Sedangkan medali perak berhasil diraih oleh Ahmad Ataka Awwalur Rizqi (SMAN 1 Yogyakarta) Prestasi ini jauh lebih baik dibanding ajang Olimpiade Fisika ke-40 di Merida Yucatan Meksiko 2009 yang lalu Saat itu delegasi siswa Indonesia merebut satu medali emas dua medali perak dan satu perunggu Hasil empat medali emas dan satu perak ini hampir menyamai prestasi terbaik sebelumnya pada ajang Olimpiade Fisika ke-37 di Singapura Saat itu siswa Indonesia tidak hanya berhasil menyabet 4 medali emas namun juga meraih predikat `Absolute Winner` atas nama Mailoa Jonathan Pradana (SMAK 1 BPK Penabur Jakarta) Tapi yang terpenting lagi pelajar Indonesia berhasil mempertahankan tradisi emas di setiap ajang Olimpiade Fisika Red Endro Yuwanto Lima siswa Indonesia berhasil mendapatkan a 4 emas 1 perak

b 1 emas 4 perak c 3 emas 2 perak d 2 perak 3 emas

Beranda Soal UKG Agus Zainal M (SDN Gudang 2)

Republik_ilalangyahoocoid

Uraian 33 KD 134 menemukan pesan pokok dalam wacana naratif seperti cerita rakyat puisi

Surat dari ibu Jika bayang telah pudar Dan elang laut pulang ke sarang Angin bertiup ke benua Tiang ndash tiang akan kering sendiri Dan nahkoda sudah tau pedoman Boleh engkau datang padaku Makna lambang dari nahkoda sudah tahu pedoman adalahhellip a Sudah mencari pedoman hidup b Sudah menemukan arah dan tujuan

c Sudah berilmu dan berpengalaman d Sudah mempunyai pasangan hidup

Uraian 34 KD 135 membandingkan berbagai jenis wacana bahasa indonesia (deskripsi narasi)

Pendekatan untuk mendapat tanggapan emosional pembaca ataupun kesan pembaca adalah contoh Pendekatan Deskripsi jenis a Pendekatan Ekspositoris b Pendekatan Impresionistik

c Pendekatan menurut sikap pengarang d Pendekatan Realistik

Uraian PENDEKATAN DESKRIPSI

Pendekatan dalam pendeskripsian dapat dibedakan menjadi beberapa kategori pendekatan yaitu

1 Pendekatan Ekspositoris

Dalam pendekatan ini kita berusaha agar deskripsi yang kita buat dapat memberi keterangan sesuai dengan keadan yang

sebenarnya sehingga pembaca dapat seolah-olah ikut melihat atau merasakan objek yang kita deskripsikan Karangan

jenis ini berisi daftar detail sesuatu secara lengkap sehingga pembaca dan penalarannya dapat memperoleh kesan

keseluruhan tentang sesuatu

2 Pendekatan Impresionistik

Tujuan deskripsi impresionistik ialah untuk mendapatkan tanggapan emosional pembaca ataupun kesan pembaca Corak

deskripsi ini diantaranya juga ditentukan oleh macam kesan apa yang diinginkan penulisnya

3 Pendekatan menurut Sikap Pengarang

Pendekatan ini sangat bergantung pada tujuan yang ingin dicapai sifat objek serta pembaca deskripsinya Dalam

menguraikan sebuah gagasan penulis mungkin mengharapkan agar pembaca merasa tidak puas terhadapa suatu

tindakan atau keadaan atau penulis menginginkan agar pembaca juga harus merasakan bahwa persoalan yang dihadapi

merupakan masalah yang gawat Penulis juga dapat membayangkan bahwa akan terjadi sesuatu yang tidak diinginkan

sehingga pembaca dari semula sudah disiapkan perasaan yang kurang enak seram takut dan sebagainya (Alkhaidah

1997) 35 kd 135 membandingkan berbagai jenis wacana bahasa indonesia (deskripsi narasi)

Prinsip-Prinsip karangan Narasi sebagai berikut kecuali a Alur b Penokohan c Amanat

d Sudut Pandang Uraian Prinsip-prinsip dasar narasi sebagai tumpuan berfikir adalah sebagai berikut

1 Alur (plot) biasa disebut dengan jalan cerita

2 Penokohan (rangkaian perbuatanaksi) 3 Latar (setting) yaitu tempat dan atau waktu terjadinya perbuatan tokoh atau peristiwa yang dialami tokoh 4 Sudut Pandang (Point of View)

a Narrator Serba Tahu (Omniscient point of view) Yaitu narrator bertindak sebagai pencipta segalanya b Narrator bertindak objektif (Objektive point of view) Pengarang tak member komentar apa pun dan pembaca bebas menafsirkan apa yang diceritakan pengarang c Narrator ikut aktif ( Narator acting) Narrator terlibat dalam cerita d Narrator sebagai peninjau

Beranda Soal UKG Agus Zainal M (SDN Gudang 2)

Republik_ilalangyahoocoid

Pelakunya adalah orang ketiga yang biasa disebut ldquodiardquo 36 KD 135 membandingkan berbagai jenis wacana bahasa indonesia (deskripsi narasi)

1 Menentukan tema atau amanat apa yang akan disampaikan 2 Menetapkan sasaran pembaca 3 Merancang peristiwa-peristiwa utama yang akan ditampilkan dalam bentuk skema perkembangandan akhir cerita 5 Memerinci peristiwa-peristiwa utama ke dalam detail-detail peristiwa sebagai pendukung cerita 6 Menyusun tokoh dan perwatakanserta latar dan sudut pandang Karakteristik diatas adalah pengembangan dari paragraf a Narasi

b Deskprisi c Ekspositoris d Sudut Pandang Persuasif

Uraian Pengembangan karangan narasi dapat dilakukan dengan langkah-langkah berikut

(1) menentukan tema atau amanat apa yang akan disampaikan (2) menetapkan sasaran pembaca (3) merancang peristiwa-peristiwa utama yang akan ditampilkan dalam bentuk skema alur (4) membagi peristiwa utama ke dalam ke dalam bagian awal perkembangan dan akhir cerita (5) memerinci peristiwa-peristiwa utama ke dalam detail-detail peristiwa sebagai pendukung cerita dan (6) menyusun tokoh dan perwatakan latar dan sudut pandang

37 KD 135 membandingkan berbagai jenis wacana bahasa indonesia (deskripsi narasi) Karangan narasi yang berusaha untuk memberikan suatu maksud tertentu menyampaikan suatu amanat terselubung kepada para pembaca atau pendengar sehingga tampak seolah-olah melihat disebut a Narasi Informatif b Narasi Ekspositoris c Narasi Sugestif d Narasi Artistik

Uraian Jenis-Jenis Narasi

Narasi Informatif Narasi informatif adalah narasi yang memiliki sasaran penyampaian informasi secara tepat tentang suatu peristiwa

dengan tujuan memperluas pengetahuan orang tentang kisah seseorang Narasi Ekspositorik

Narasi ekspositorik adalah narasi yang memiliki sasaran penyampaian informasi secara tepat tentang suatu peristiwa dengan tujuan memperluas pengetahuan orang tentang kisah seseorang Dalam narasi ekspositorik penulis menceritakan suatu peristiwa berdasarkan data yang sebenarnya Pelaku yang ditonjolkan biasanya satu orang Pelaku diceritakan mulai dari kecil sampai saat ini atau sampai terakhir dalam kehidupannya Karangan narasi ini diwarnai oleh eksposisi maka ketentuan eksposisi juga berlaku pada penulisan narasi ekspositprik Ketentuan ini berkaitan dengan penggunaan bahasa yang logis berdasarkan fakta yang ada tidak memasukan unsursugestif atau bersifat objektif

Narasi Artistik

Narasi artistik adalah narasi yang berusaha untuk memberikan suatu maksud tertentu menyampaikan suatu amanat terselubung kepada para pembaca atau pendengar sehingga tampak seolah-olah melihat Ketentuan ini berkaitan dengan penggunaan bahasa yang logis berdasarkan fakta yang ada tidak memasukan unsur sugestif atau bersifat objektif

Narasi Sugestif Narasi sugestif adalah narasi yang berusaha untuk memberikan suatu maksud tertentu menyampaikan suatu amanat

terselubung kepada para pembaca atau pendengar sehingga tampak seolah-olah melihat 38 KD 135 membandingkan berbagai jenis wacana bahasa indonesia (deskripsi narasi)

Ciri karangan Narasi yang benar kecuali a Menonjolkan unsur perbuatan atau tindakan Membuat pembaca atau pendengar merasakan sendiri atau

mengalami sendiri

b Ada konfiks menjawab pertanyaan apa yang terjadi c Dirangkai dalam urutan waktu menggambarkan dengan jelas suatu peristiwa d Berisi ajakan dirangkai dalam urutan waktu

Uraian Ciri-Ciri Karangan Narasi Menurut Gorys Keraf (2000136)

Menonjolkan unsur perbuatan atau tindakan Dirangkai dalam urutan waktu Berusaha menjawab pertanyaan apa yang terjadi Ada konfiks

39 kd 135 membandingkan berbagai jenis wacana bahasa indonesia (deskripsi narasi) Langkah menyusun paragraf deskripsi yang benar adalah (1)Tentukan objek atau tema yang akan dideskripsikan (2)Mengumpulkan data dengan mengamati objek yang akan dideskripsikan (3)Menyusun data tersebut ke dalam urutan yang baik (menyusun kerangka karangan) (4)Menguraikan kerangka karangan menjadi dekripsi yang sesuai dengan tema yang ditentukan

Beranda Soal UKG Agus Zainal M (SDN Gudang 2)

Republik_ilalangyahoocoid

(5)Tentukan tujuan a 1-5-2-3-4

b 1-5-4-3-2 c 1-5-3-2-4 d 1-3-4-2-5

Uraian Langkah menyusun deskripsi

1Tentukan objek atau tema yang akan dideskripsikan 2Tentukan tujuan 3Mengumpulkan data dengan mengamati objek yang akan dideskripsikan 4Menyusun data tersebut ke dalam urutan yang baik (menyusun kerangka karangan) 5Menguraikan kerangka karangan menjadi dekripsi yang sesuai dengan tema yang ditentukan

40 Kd 311 Merancang aktivitas pembelajaran berdasarkan prinsip dan teori pembelajaran matematika Dalam tahap ini penyajian yang dilakukan melalui tindakan anak secara langsung terlihat dalam memanipulasi (mengotak atik)objek Proses tersebut menurut Bruner dikategorikan ke dalam model a Tahap Enaktif

b Tahap Ikonik c Tahap Simbolik d Tahap Implikatif

Uraian 1 Model Tahap Enaktif

Dalam tahap ini penyajian yang dilakukan melalui tindakan anak secara langsung terlihat dalam memanipulasi (mengotak atik)objek Contoh Budi mempunyai 2 pensil kemudian ibunya memberikannya lagi 3 pinsil Berapa banyak pensil Budi sekarang

2 Model Tahap Ikonik Dalam tahap ini kegiatan penyajian dilakukan berdasarkan pada pikiran internal dimana pengetahuan disajikan melalui serangkaian gambar-gambar atau grafik yang dilakukan anak berhubungan dengan mental yang merupakan gambaran dari objek-objek yang dimanipulasinya Contoh + = hellip

3 Model Tahap Simbolis Dalam tahap ini bahasa adalah pola dasar simbolik anak memanipulasi Simbol-simbol atau lambang-lambang objek tertentu Contoh 2 pensil + 3 pensil = hellip pensil

41 Kd 311 Merancang aktivitas pembelajaran berdasarkan prinsip dan teori pembelajaran matematika Dalam tahap ini kegiatan penyajian dilakukan berdasarkan pada pikiran internal dimana pengetahuan disajikan melalui serangkaian gambar-gambar atau grafik yang dilakukan anak Proses tersebut menurut Bruner dikategorikan ke dalam model a Tahap Enaktif b Tahap Ikonik

c Tahap Simbolik d Tahap Implikatif

Uraian No 40 42 Kd 311 Merancang aktivitas pembelajaran berdasarkan prinsip dan teori pembelajaran matematika

Objek belajar matematika dibagi kedalam Objek Langsung dan Objek Tak Langsung Hal tersebut adalah teori belajar Matematika menurut a Robert M Gagne

b Jerome S Burner c Thorndike d Skinner

Uraian Teori yang diperkenalkan Robert MGagne pada tahun 1960-an pembelajaran harus dikondisikan untuk memunculkan

respons yang diharapkanMenurut Gagne (dalam Ismail 1998) belajar matematika terdiri dari objek langsung dan objek tak langsung

43 Kd 311 Merancang aktivitas pembelajaran berdasarkan prinsip dan teori pembelajaran matematika Perhatikan contoh berikut Budi mempunyai 2 pinsil kemudian ibunya memberikannya lagi 3 pinsilBerapa banyak pinsil Budi sekarang Hal tersebut dikemukakan Bruner dalam Proses Pembelajaran Matematika dalam tahap a Simbolik b Ikonik c Implikatif d Enaktif

Uraian No 40

Beranda Soal UKG Agus Zainal M (SDN Gudang 2)

Republik_ilalangyahoocoid

44 Kd 311 Merancang aktivitas pembelajaran berdasarkan prinsip dan teori pembelajaran matematika Perhatikan gambar disamping Ilustrasi di samping dikemukakan Bruner dalam Proses Pembelajaran Matematika dalam tahap

a Simbolik b Ikonik

c Implikatif d Enaktif

Uraian No 40 45 Kd 311 Merancang aktivitas pembelajaran berdasarkan prinsip dan teori pembelajaran matematika

Perhatikan contoh berikut Contoh 2 pinsil + 3 pinsil = hellippinsil Contoh tersebut dikemukakan Bruner dalam Proses Pembelajaran Matematika dalam tahap a Simbolik

b Ikonik c Implikatif d Enaktif

Uraian No 40 46 Kd 311 Merancang aktivitas pembelajaran berdasarkan prinsip dan teori pembelajaran matematika

Perhatikan contoh berikut penjumlahan bilangan positif dan negatif siswa mencoba sendiri dengan menggunakan garis bilangan Contoh tersebut dikemukakan Bruner dalam Teorema Pembelajaran Matematika a Penyusunan

b Notasi c Pengkontrasan dan Keanekaragaman d Pengaitan

Uraian Berdasarkan hasil pengamatannya Brunner merumuskan 5 teorema dalam pembelajaran matematika yaitu

1) Teorema Penyusunan Menerangkan bahwa cara yang terbaik memulai belajar suatu konsep matematika dalil defenisi dan semacamnya adalah dengan cara menyusun penyajiannya Misalnya dalam mempelajari penjumlahan bilangan positif dan negatif siswa mencoba sendiri dengan menggunakan garis bilangan 2) Teorema Notasi Menerangkan bahwa dalam pengajaran suatu konsep penggunaan notasi-notasi matematika harus diberikan secara bertahap dari yang sederhana ke yang lebih kompleks 3) Teorema Pengkontrasan dan Keanekaragaman Menerangkan bahwa pengontrasan dan keanekaragaman sangat penting dalam melakukan pengubahan konsep matematika dari yang konkrit ke yang lebih abstrak Dalam hal ini diperlukan banyak contoh Contoh yang diberikan harus sesuai dengan rumusan yang diberikan Misalnya menjelaskan persegi panjang disertai juga kemungkinan jajaran genjang dan segi empat lainnya selain persegi panjnag Dengan demikian siswa dapat membedakan apakah segi empat yang diberikan padanya termasuk persegi panjang atau tidak 4) Teorema Pengaitan Menerangkan bahwa dalam matematika terdapat hubungan yang berkaitan antara satu konsep dengan konsep yang lain Di mana materi yang satu merupakan prasyarat yang harus diketahui untuk mempelajari materi yang lain

47 Kd 311 Merancang aktivitas pembelajaran berdasarkan prinsip dan teori pembelajaran matematika Perhatikan contoh berikut Guru menjelaskan persegi panjang disertai juga kemungkinan jajaran genjang dan segi empat lainnya selain persegi panjnag Dengan demikian siswa dapat membedakan apakah segi empat yang diberikan padanya termasuk persegi panjang atau tidak Contoh tersebut dikemukakan Bruner dalam Teorema Pembelajaran Matematika a Penyusunan b Notasi c Pengkontrasan dan Keanekaragaman

d Pengaitan Uraian No 46 48 Kd 311 Merancang aktivitas pembelajaran berdasarkan prinsip dan teori pembelajaran matematika

Tahap pembelajaran Matematika menurut Van Halle adalah sebagai berikut kecuali a Tahap Pengenalan b Tahap Pengurutan c Tahap Analisis d Tahap Penyimpulan

Beranda Soal UKG Agus Zainal M (SDN Gudang 2)

Republik_ilalangyahoocoid

Uraian Van Hiele menyatakan bahwa terdapat 5 tahap belajar siswa dalam belajar geometri yaitu

a Tahap Pengenalan Pada tahap ini siswa mulai belajar mengenal suatu bangun geometri secara keseluruhan namun belum mampu mengetahui adanya sifat-sifat dari bangun geometri yang dilihatnya b Tahap Analisis Pada tahap ini siswa sudah mulai mengenal sifat-sifat yang dimiliki bangun geometri yang diamatinya c Tahap Pengurutan Pada tahap ini siswa sudah mengenal dan memahami sifat-sifat suatu bangun geometri serta sudah dapat mengurutkan bangun-bangun geometri yang satu sama yang lainnya saling berhubungan d Tahap Deduksi Pada tahap ini siswa telah mampu menarik kesimpulan secara deduktif yaitu menarik kesimpulan yang bersifat umum dan menuju ke hal yang bersifat khusus serta dapat mengambil kesimpulan e Tahap Akurasi Pada tahap ini siswa mulai menyadari pentingnya ketepatan prinsip-prinsip dasar yang melandasi suatu pembuktian Tahap berfikir ini merupakan tahap berfikir yang paling tinggi rumit dan kompleks karena di luar jangkauan usia anak-anak SD sampai tingakat SMP

49 Kd 311 Merancang aktivitas pembelajaran berdasarkan prinsip dan teori pembelajaran matematika middot Matematika untuk tujuan pembelajaran dianalisis sebagai kumpulan fakta yang berdiri sendiri dan tidak saling berkaitan middot Anak diharuskan menguasai unsur-unsur yang banyak sekali tanpa diperhatikan pengertiannya middot Anak mempelajari unsur-unsur dalam bentuk seperti yang akan digunakan nanti dalam kesempatan lain middot Anak akan mencapai tujuan ini secara efektif dan efisien dengan melalui pengulangan Teori Pembelajaran Matematika tersebut dikemukakan oleh a Skinner b Piaget c Van Brownell

d Thorndike Uraian Brownell mengemukakan tentang Teori Makna (Meaning Theory) sebagai pengganti Teori Latihan HafalUlangan (Drill

Theory) Intisari dari teori Drill adalah - Matematika untuk tujuan pembelajaran dianalisis sebagai kumpulan fakta yang berdiri sendiri dan tidak saling berkaitan - Anak diharuskan menguasai unsur-unsur yang banyak sekali tanpa diperhatikan pengertiannya - Anak mempelajari unsur-unsur dalam bentuk seperti yang akan digunakan nanti dalam kesempatan lain - Anak akan mencapai tujuan ini secara efektif dan efisien dengan melalui pengulangan

50 Kd 311 Merancang aktivitas pembelajaran berdasarkan prinsip dan teori pembelajaran matematika Tahap operasionaloperasi konkrit adalah teori Pembelajaran Matematika yang dikemukakan Peaget pada tahap usia a dibawah 2 tahun b 2-7 Tahun c 7-8 Tahun d 7-12 Tahun

Uraian Piaget membagi skema yang digunakan anak untuk memahami dunianya melalui empat periode utama yang berkorelasi

dengan dan semakin canggih seiring pertambahan usia Periode sensorimotor (usia 0ndash2 tahun) Periode praoperasional (usia 2ndash7 tahun) Periode operasional konkrit (usia 7ndash12 tahun) Periode operasional formal (usia 12 tahun sampai dewasa)

51 313 Memilih media pembelajaran yang tepat untuk pembelajaran operasi bilangan bulat Untuk operasi bilangan bulat perkalian media yang paling tepat digunakan adalah a sapu lidi

b kerikil c daun pakis d koin

Uraian 52 313 Memilih media pembelajaran yang tepat untuk pembelajaran operasi bilangan bulat

Untuk operasi bilangan bulat negatif kita bisa menggunakan media a uang kertas b kerikil c mistar d koin

Uraian 53 313 Memilih media pembelajaran yang tepat untuk pembelajaran operasi bilangan bulat

Beranda Soal UKG Agus Zainal M (SDN Gudang 2)

Republik_ilalangyahoocoid

Untuk operasi bilangan bulat penjumlahan media yang paling tepat digunakan adalah a uang kertas b kerikil c daun pakis d koin

Uraian 54 314 Memilih media pembelajaran yang tepat untuk pembelajaran operasi bilangan pecahan

Untuk operasi bilangan pecahan kita bisa menggunakan media berikut kecuali a kartu bilangan b garis bilangan c gambar bidang d blok pecahan

Uraian 55 316 Memilih media pembelajaran yang tepat untuk pembelajaran geometri dan pengukuran

Untuk menghitung luas dan keliling bangun datar media yang paling tepat digunakan adalah a Penggaris b Kertas Folio Bergaris

c Neraca d Kalkulator

Uraian

Page 9: 50 SOAL UKG & PEMBAHASAN SESUAI KISI-KISIS.pdf

Beranda Soal UKG Agus Zainal M (SDN Gudang 2)

Republik_ilalangyahoocoid

Berdasarkan maksud tujuan atau keintensifan serta cara dalam membaca di bawah ini Anderson dalam Tarigan (19799-10) mengemukakan beberapa tujuan membaca antara lain a Membaca untuk memperoleh perincian-perincian atau fakta-fakta (reading for details or facts) Membaca tersebut bertujuan untuk menemukan atau mengetahui penemuan-penemuan telah dilakukan oleh sang tokoh untuk memecahkan masalah-masalah yang dibuat oleh sang tokoh b Membaca untuk memperoleh ide-ide utama (reading for main ideas) Membaca untuk mengetahui topik atau masalah dalam bacaan Untuk menemukan ide pokok bacaan dengan membaca halamn demi halaman c Membaca untuk mengetahui ukuran atau susunan organisasi cerita (reading for sequenceor organization) Membaca tersebut bertujuan untuk mengetahui bagian-bagian cerita dan hubungan antar bagian-bagian cerita d Membaca untuk menyimpulkan atau membaca inferensi (reading for inference) Pembaca diharapkan dapat merasakan sesuatu yang dirasakan penulis e Membaca untuk mengelompokkan atau mengklasifikasikan (reading for classify) Membaca jenis ini bertujuan untuk menemukan hal-hal yang tidak wajar mengenai sesuatu hal (Anderson dalam Tarigan 197910) f Membaca untuk menilai atau mengevaluasai (reading to evaluate) Jenis membaca tersebut bertujuan menemukan suatu keberhasilan berdasarkan ukuran-ukuran tertentu Membaca jenis ini memerlukan ketelitian dengan membandingkan dan mengujinya kembali g Membaca untuk memperbandingkan atau mempertentangkan (reading to compare or contrast) Tujuan membaca tersebut adalah untuk menemukan bagaimana cara perbedaan atau persamaan dua hal atau lebih

31 KD 133 menemukan pesan pokok utama sebuah berita Simak penggalan berita berikut Hebat Siswa Indonesia Pertahankan Tradisi Emas di Olimpiade Fisika Tim Olimpiade Fisika Indonesia REPUBLIKACOID JAKARTA--Luar biasa Lima siswa Indonesia yang dikirim ke ajang Olimpiade Fisika atau International Physics Olympiad (IPhO) ke-41 di Zagreb Kroasia 17-25 Juli berhasil menyabet empat medali emas dan satu perak Pelajar yang menyumbang emas adalah Muhammad Sohibul Maromi (SMAN 1 Pamekasan Madura) Christian George Emor (SMA Lokon St Nikolaus Tomohon Sulawesi Utara) David Giovanni (SMAK Penabur Gading Serpong Banten) dan Kevin Soedyatmiko (SMAN 12 Jakarta) Sedangkan medali perak berhasil diraih oleh Ahmad Ataka Awwalur Rizqi (SMAN 1 Yogyakarta) Prestasi ini jauh lebih baik dibanding ajang Olimpiade Fisika ke-40 di Merida Yucatan Meksiko 2009 yang lalu Saat itu delegasi siswa Indonesia merebut satu medali emas dua medali perak dan satu perunggu Hasil empat medali emas dan satu perak ini hampir menyamai prestasi terbaik sebelumnya pada ajang Olimpiade Fisika ke-37 di Singapura Saat itu siswa Indonesia tidak hanya berhasil menyabet 4 medali emas namun juga meraih predikat `Absolute Winner` atas nama Mailoa Jonathan Pradana (SMAK 1 BPK Penabur Jakarta) Tapi yang terpenting lagi pelajar Indonesia berhasil mempertahankan tradisi emas di setiap ajang Olimpiade Fisika Red Endro Yuwanto Peristiwa yang diberitakan adalah a siswa Indonesia pertahankan tradisi emas di Olimpiade Matematika b siswa Indonesia pertahankan tradisi emas di Olimpiade Fisika c siswa Indonesia menyumbang emas di Olimpiade Matematika

d siswa Indonesia menyumbang perak di Olimpiade Fisika Uraian 32 KD 133 menemukan pesan pokok utama sebuah berita

Simak penggalan berita berikut Hebat Siswa Indonesia Pertahankan Tradisi Emas di Olimpiade Fisika Tim Olimpiade Fisika Indonesia REPUBLIKACOID JAKARTA--Luar biasa Lima siswa Indonesia yang dikirim ke ajang Olimpiade Fisika atau International Physics Olympiad (IPhO) ke-41 di Zagreb Kroasia 17-25 Juli berhasil menyabet empat medali emas dan satu perak Pelajar yang menyumbang emas adalah Muhammad Sohibul Maromi (SMAN 1 Pamekasan Madura) Christian George Emor (SMA Lokon St Nikolaus Tomohon Sulawesi Utara) David Giovanni (SMAK Penabur Gading Serpong Banten) dan Kevin Soedyatmiko (SMAN 12 Jakarta) Sedangkan medali perak berhasil diraih oleh Ahmad Ataka Awwalur Rizqi (SMAN 1 Yogyakarta) Prestasi ini jauh lebih baik dibanding ajang Olimpiade Fisika ke-40 di Merida Yucatan Meksiko 2009 yang lalu Saat itu delegasi siswa Indonesia merebut satu medali emas dua medali perak dan satu perunggu Hasil empat medali emas dan satu perak ini hampir menyamai prestasi terbaik sebelumnya pada ajang Olimpiade Fisika ke-37 di Singapura Saat itu siswa Indonesia tidak hanya berhasil menyabet 4 medali emas namun juga meraih predikat `Absolute Winner` atas nama Mailoa Jonathan Pradana (SMAK 1 BPK Penabur Jakarta) Tapi yang terpenting lagi pelajar Indonesia berhasil mempertahankan tradisi emas di setiap ajang Olimpiade Fisika Red Endro Yuwanto Lima siswa Indonesia berhasil mendapatkan a 4 emas 1 perak

b 1 emas 4 perak c 3 emas 2 perak d 2 perak 3 emas

Beranda Soal UKG Agus Zainal M (SDN Gudang 2)

Republik_ilalangyahoocoid

Uraian 33 KD 134 menemukan pesan pokok dalam wacana naratif seperti cerita rakyat puisi

Surat dari ibu Jika bayang telah pudar Dan elang laut pulang ke sarang Angin bertiup ke benua Tiang ndash tiang akan kering sendiri Dan nahkoda sudah tau pedoman Boleh engkau datang padaku Makna lambang dari nahkoda sudah tahu pedoman adalahhellip a Sudah mencari pedoman hidup b Sudah menemukan arah dan tujuan

c Sudah berilmu dan berpengalaman d Sudah mempunyai pasangan hidup

Uraian 34 KD 135 membandingkan berbagai jenis wacana bahasa indonesia (deskripsi narasi)

Pendekatan untuk mendapat tanggapan emosional pembaca ataupun kesan pembaca adalah contoh Pendekatan Deskripsi jenis a Pendekatan Ekspositoris b Pendekatan Impresionistik

c Pendekatan menurut sikap pengarang d Pendekatan Realistik

Uraian PENDEKATAN DESKRIPSI

Pendekatan dalam pendeskripsian dapat dibedakan menjadi beberapa kategori pendekatan yaitu

1 Pendekatan Ekspositoris

Dalam pendekatan ini kita berusaha agar deskripsi yang kita buat dapat memberi keterangan sesuai dengan keadan yang

sebenarnya sehingga pembaca dapat seolah-olah ikut melihat atau merasakan objek yang kita deskripsikan Karangan

jenis ini berisi daftar detail sesuatu secara lengkap sehingga pembaca dan penalarannya dapat memperoleh kesan

keseluruhan tentang sesuatu

2 Pendekatan Impresionistik

Tujuan deskripsi impresionistik ialah untuk mendapatkan tanggapan emosional pembaca ataupun kesan pembaca Corak

deskripsi ini diantaranya juga ditentukan oleh macam kesan apa yang diinginkan penulisnya

3 Pendekatan menurut Sikap Pengarang

Pendekatan ini sangat bergantung pada tujuan yang ingin dicapai sifat objek serta pembaca deskripsinya Dalam

menguraikan sebuah gagasan penulis mungkin mengharapkan agar pembaca merasa tidak puas terhadapa suatu

tindakan atau keadaan atau penulis menginginkan agar pembaca juga harus merasakan bahwa persoalan yang dihadapi

merupakan masalah yang gawat Penulis juga dapat membayangkan bahwa akan terjadi sesuatu yang tidak diinginkan

sehingga pembaca dari semula sudah disiapkan perasaan yang kurang enak seram takut dan sebagainya (Alkhaidah

1997) 35 kd 135 membandingkan berbagai jenis wacana bahasa indonesia (deskripsi narasi)

Prinsip-Prinsip karangan Narasi sebagai berikut kecuali a Alur b Penokohan c Amanat

d Sudut Pandang Uraian Prinsip-prinsip dasar narasi sebagai tumpuan berfikir adalah sebagai berikut

1 Alur (plot) biasa disebut dengan jalan cerita

2 Penokohan (rangkaian perbuatanaksi) 3 Latar (setting) yaitu tempat dan atau waktu terjadinya perbuatan tokoh atau peristiwa yang dialami tokoh 4 Sudut Pandang (Point of View)

a Narrator Serba Tahu (Omniscient point of view) Yaitu narrator bertindak sebagai pencipta segalanya b Narrator bertindak objektif (Objektive point of view) Pengarang tak member komentar apa pun dan pembaca bebas menafsirkan apa yang diceritakan pengarang c Narrator ikut aktif ( Narator acting) Narrator terlibat dalam cerita d Narrator sebagai peninjau

Beranda Soal UKG Agus Zainal M (SDN Gudang 2)

Republik_ilalangyahoocoid

Pelakunya adalah orang ketiga yang biasa disebut ldquodiardquo 36 KD 135 membandingkan berbagai jenis wacana bahasa indonesia (deskripsi narasi)

1 Menentukan tema atau amanat apa yang akan disampaikan 2 Menetapkan sasaran pembaca 3 Merancang peristiwa-peristiwa utama yang akan ditampilkan dalam bentuk skema perkembangandan akhir cerita 5 Memerinci peristiwa-peristiwa utama ke dalam detail-detail peristiwa sebagai pendukung cerita 6 Menyusun tokoh dan perwatakanserta latar dan sudut pandang Karakteristik diatas adalah pengembangan dari paragraf a Narasi

b Deskprisi c Ekspositoris d Sudut Pandang Persuasif

Uraian Pengembangan karangan narasi dapat dilakukan dengan langkah-langkah berikut

(1) menentukan tema atau amanat apa yang akan disampaikan (2) menetapkan sasaran pembaca (3) merancang peristiwa-peristiwa utama yang akan ditampilkan dalam bentuk skema alur (4) membagi peristiwa utama ke dalam ke dalam bagian awal perkembangan dan akhir cerita (5) memerinci peristiwa-peristiwa utama ke dalam detail-detail peristiwa sebagai pendukung cerita dan (6) menyusun tokoh dan perwatakan latar dan sudut pandang

37 KD 135 membandingkan berbagai jenis wacana bahasa indonesia (deskripsi narasi) Karangan narasi yang berusaha untuk memberikan suatu maksud tertentu menyampaikan suatu amanat terselubung kepada para pembaca atau pendengar sehingga tampak seolah-olah melihat disebut a Narasi Informatif b Narasi Ekspositoris c Narasi Sugestif d Narasi Artistik

Uraian Jenis-Jenis Narasi

Narasi Informatif Narasi informatif adalah narasi yang memiliki sasaran penyampaian informasi secara tepat tentang suatu peristiwa

dengan tujuan memperluas pengetahuan orang tentang kisah seseorang Narasi Ekspositorik

Narasi ekspositorik adalah narasi yang memiliki sasaran penyampaian informasi secara tepat tentang suatu peristiwa dengan tujuan memperluas pengetahuan orang tentang kisah seseorang Dalam narasi ekspositorik penulis menceritakan suatu peristiwa berdasarkan data yang sebenarnya Pelaku yang ditonjolkan biasanya satu orang Pelaku diceritakan mulai dari kecil sampai saat ini atau sampai terakhir dalam kehidupannya Karangan narasi ini diwarnai oleh eksposisi maka ketentuan eksposisi juga berlaku pada penulisan narasi ekspositprik Ketentuan ini berkaitan dengan penggunaan bahasa yang logis berdasarkan fakta yang ada tidak memasukan unsursugestif atau bersifat objektif

Narasi Artistik

Narasi artistik adalah narasi yang berusaha untuk memberikan suatu maksud tertentu menyampaikan suatu amanat terselubung kepada para pembaca atau pendengar sehingga tampak seolah-olah melihat Ketentuan ini berkaitan dengan penggunaan bahasa yang logis berdasarkan fakta yang ada tidak memasukan unsur sugestif atau bersifat objektif

Narasi Sugestif Narasi sugestif adalah narasi yang berusaha untuk memberikan suatu maksud tertentu menyampaikan suatu amanat

terselubung kepada para pembaca atau pendengar sehingga tampak seolah-olah melihat 38 KD 135 membandingkan berbagai jenis wacana bahasa indonesia (deskripsi narasi)

Ciri karangan Narasi yang benar kecuali a Menonjolkan unsur perbuatan atau tindakan Membuat pembaca atau pendengar merasakan sendiri atau

mengalami sendiri

b Ada konfiks menjawab pertanyaan apa yang terjadi c Dirangkai dalam urutan waktu menggambarkan dengan jelas suatu peristiwa d Berisi ajakan dirangkai dalam urutan waktu

Uraian Ciri-Ciri Karangan Narasi Menurut Gorys Keraf (2000136)

Menonjolkan unsur perbuatan atau tindakan Dirangkai dalam urutan waktu Berusaha menjawab pertanyaan apa yang terjadi Ada konfiks

39 kd 135 membandingkan berbagai jenis wacana bahasa indonesia (deskripsi narasi) Langkah menyusun paragraf deskripsi yang benar adalah (1)Tentukan objek atau tema yang akan dideskripsikan (2)Mengumpulkan data dengan mengamati objek yang akan dideskripsikan (3)Menyusun data tersebut ke dalam urutan yang baik (menyusun kerangka karangan) (4)Menguraikan kerangka karangan menjadi dekripsi yang sesuai dengan tema yang ditentukan

Beranda Soal UKG Agus Zainal M (SDN Gudang 2)

Republik_ilalangyahoocoid

(5)Tentukan tujuan a 1-5-2-3-4

b 1-5-4-3-2 c 1-5-3-2-4 d 1-3-4-2-5

Uraian Langkah menyusun deskripsi

1Tentukan objek atau tema yang akan dideskripsikan 2Tentukan tujuan 3Mengumpulkan data dengan mengamati objek yang akan dideskripsikan 4Menyusun data tersebut ke dalam urutan yang baik (menyusun kerangka karangan) 5Menguraikan kerangka karangan menjadi dekripsi yang sesuai dengan tema yang ditentukan

40 Kd 311 Merancang aktivitas pembelajaran berdasarkan prinsip dan teori pembelajaran matematika Dalam tahap ini penyajian yang dilakukan melalui tindakan anak secara langsung terlihat dalam memanipulasi (mengotak atik)objek Proses tersebut menurut Bruner dikategorikan ke dalam model a Tahap Enaktif

b Tahap Ikonik c Tahap Simbolik d Tahap Implikatif

Uraian 1 Model Tahap Enaktif

Dalam tahap ini penyajian yang dilakukan melalui tindakan anak secara langsung terlihat dalam memanipulasi (mengotak atik)objek Contoh Budi mempunyai 2 pensil kemudian ibunya memberikannya lagi 3 pinsil Berapa banyak pensil Budi sekarang

2 Model Tahap Ikonik Dalam tahap ini kegiatan penyajian dilakukan berdasarkan pada pikiran internal dimana pengetahuan disajikan melalui serangkaian gambar-gambar atau grafik yang dilakukan anak berhubungan dengan mental yang merupakan gambaran dari objek-objek yang dimanipulasinya Contoh + = hellip

3 Model Tahap Simbolis Dalam tahap ini bahasa adalah pola dasar simbolik anak memanipulasi Simbol-simbol atau lambang-lambang objek tertentu Contoh 2 pensil + 3 pensil = hellip pensil

41 Kd 311 Merancang aktivitas pembelajaran berdasarkan prinsip dan teori pembelajaran matematika Dalam tahap ini kegiatan penyajian dilakukan berdasarkan pada pikiran internal dimana pengetahuan disajikan melalui serangkaian gambar-gambar atau grafik yang dilakukan anak Proses tersebut menurut Bruner dikategorikan ke dalam model a Tahap Enaktif b Tahap Ikonik

c Tahap Simbolik d Tahap Implikatif

Uraian No 40 42 Kd 311 Merancang aktivitas pembelajaran berdasarkan prinsip dan teori pembelajaran matematika

Objek belajar matematika dibagi kedalam Objek Langsung dan Objek Tak Langsung Hal tersebut adalah teori belajar Matematika menurut a Robert M Gagne

b Jerome S Burner c Thorndike d Skinner

Uraian Teori yang diperkenalkan Robert MGagne pada tahun 1960-an pembelajaran harus dikondisikan untuk memunculkan

respons yang diharapkanMenurut Gagne (dalam Ismail 1998) belajar matematika terdiri dari objek langsung dan objek tak langsung

43 Kd 311 Merancang aktivitas pembelajaran berdasarkan prinsip dan teori pembelajaran matematika Perhatikan contoh berikut Budi mempunyai 2 pinsil kemudian ibunya memberikannya lagi 3 pinsilBerapa banyak pinsil Budi sekarang Hal tersebut dikemukakan Bruner dalam Proses Pembelajaran Matematika dalam tahap a Simbolik b Ikonik c Implikatif d Enaktif

Uraian No 40

Beranda Soal UKG Agus Zainal M (SDN Gudang 2)

Republik_ilalangyahoocoid

44 Kd 311 Merancang aktivitas pembelajaran berdasarkan prinsip dan teori pembelajaran matematika Perhatikan gambar disamping Ilustrasi di samping dikemukakan Bruner dalam Proses Pembelajaran Matematika dalam tahap

a Simbolik b Ikonik

c Implikatif d Enaktif

Uraian No 40 45 Kd 311 Merancang aktivitas pembelajaran berdasarkan prinsip dan teori pembelajaran matematika

Perhatikan contoh berikut Contoh 2 pinsil + 3 pinsil = hellippinsil Contoh tersebut dikemukakan Bruner dalam Proses Pembelajaran Matematika dalam tahap a Simbolik

b Ikonik c Implikatif d Enaktif

Uraian No 40 46 Kd 311 Merancang aktivitas pembelajaran berdasarkan prinsip dan teori pembelajaran matematika

Perhatikan contoh berikut penjumlahan bilangan positif dan negatif siswa mencoba sendiri dengan menggunakan garis bilangan Contoh tersebut dikemukakan Bruner dalam Teorema Pembelajaran Matematika a Penyusunan

b Notasi c Pengkontrasan dan Keanekaragaman d Pengaitan

Uraian Berdasarkan hasil pengamatannya Brunner merumuskan 5 teorema dalam pembelajaran matematika yaitu

1) Teorema Penyusunan Menerangkan bahwa cara yang terbaik memulai belajar suatu konsep matematika dalil defenisi dan semacamnya adalah dengan cara menyusun penyajiannya Misalnya dalam mempelajari penjumlahan bilangan positif dan negatif siswa mencoba sendiri dengan menggunakan garis bilangan 2) Teorema Notasi Menerangkan bahwa dalam pengajaran suatu konsep penggunaan notasi-notasi matematika harus diberikan secara bertahap dari yang sederhana ke yang lebih kompleks 3) Teorema Pengkontrasan dan Keanekaragaman Menerangkan bahwa pengontrasan dan keanekaragaman sangat penting dalam melakukan pengubahan konsep matematika dari yang konkrit ke yang lebih abstrak Dalam hal ini diperlukan banyak contoh Contoh yang diberikan harus sesuai dengan rumusan yang diberikan Misalnya menjelaskan persegi panjang disertai juga kemungkinan jajaran genjang dan segi empat lainnya selain persegi panjnag Dengan demikian siswa dapat membedakan apakah segi empat yang diberikan padanya termasuk persegi panjang atau tidak 4) Teorema Pengaitan Menerangkan bahwa dalam matematika terdapat hubungan yang berkaitan antara satu konsep dengan konsep yang lain Di mana materi yang satu merupakan prasyarat yang harus diketahui untuk mempelajari materi yang lain

47 Kd 311 Merancang aktivitas pembelajaran berdasarkan prinsip dan teori pembelajaran matematika Perhatikan contoh berikut Guru menjelaskan persegi panjang disertai juga kemungkinan jajaran genjang dan segi empat lainnya selain persegi panjnag Dengan demikian siswa dapat membedakan apakah segi empat yang diberikan padanya termasuk persegi panjang atau tidak Contoh tersebut dikemukakan Bruner dalam Teorema Pembelajaran Matematika a Penyusunan b Notasi c Pengkontrasan dan Keanekaragaman

d Pengaitan Uraian No 46 48 Kd 311 Merancang aktivitas pembelajaran berdasarkan prinsip dan teori pembelajaran matematika

Tahap pembelajaran Matematika menurut Van Halle adalah sebagai berikut kecuali a Tahap Pengenalan b Tahap Pengurutan c Tahap Analisis d Tahap Penyimpulan

Beranda Soal UKG Agus Zainal M (SDN Gudang 2)

Republik_ilalangyahoocoid

Uraian Van Hiele menyatakan bahwa terdapat 5 tahap belajar siswa dalam belajar geometri yaitu

a Tahap Pengenalan Pada tahap ini siswa mulai belajar mengenal suatu bangun geometri secara keseluruhan namun belum mampu mengetahui adanya sifat-sifat dari bangun geometri yang dilihatnya b Tahap Analisis Pada tahap ini siswa sudah mulai mengenal sifat-sifat yang dimiliki bangun geometri yang diamatinya c Tahap Pengurutan Pada tahap ini siswa sudah mengenal dan memahami sifat-sifat suatu bangun geometri serta sudah dapat mengurutkan bangun-bangun geometri yang satu sama yang lainnya saling berhubungan d Tahap Deduksi Pada tahap ini siswa telah mampu menarik kesimpulan secara deduktif yaitu menarik kesimpulan yang bersifat umum dan menuju ke hal yang bersifat khusus serta dapat mengambil kesimpulan e Tahap Akurasi Pada tahap ini siswa mulai menyadari pentingnya ketepatan prinsip-prinsip dasar yang melandasi suatu pembuktian Tahap berfikir ini merupakan tahap berfikir yang paling tinggi rumit dan kompleks karena di luar jangkauan usia anak-anak SD sampai tingakat SMP

49 Kd 311 Merancang aktivitas pembelajaran berdasarkan prinsip dan teori pembelajaran matematika middot Matematika untuk tujuan pembelajaran dianalisis sebagai kumpulan fakta yang berdiri sendiri dan tidak saling berkaitan middot Anak diharuskan menguasai unsur-unsur yang banyak sekali tanpa diperhatikan pengertiannya middot Anak mempelajari unsur-unsur dalam bentuk seperti yang akan digunakan nanti dalam kesempatan lain middot Anak akan mencapai tujuan ini secara efektif dan efisien dengan melalui pengulangan Teori Pembelajaran Matematika tersebut dikemukakan oleh a Skinner b Piaget c Van Brownell

d Thorndike Uraian Brownell mengemukakan tentang Teori Makna (Meaning Theory) sebagai pengganti Teori Latihan HafalUlangan (Drill

Theory) Intisari dari teori Drill adalah - Matematika untuk tujuan pembelajaran dianalisis sebagai kumpulan fakta yang berdiri sendiri dan tidak saling berkaitan - Anak diharuskan menguasai unsur-unsur yang banyak sekali tanpa diperhatikan pengertiannya - Anak mempelajari unsur-unsur dalam bentuk seperti yang akan digunakan nanti dalam kesempatan lain - Anak akan mencapai tujuan ini secara efektif dan efisien dengan melalui pengulangan

50 Kd 311 Merancang aktivitas pembelajaran berdasarkan prinsip dan teori pembelajaran matematika Tahap operasionaloperasi konkrit adalah teori Pembelajaran Matematika yang dikemukakan Peaget pada tahap usia a dibawah 2 tahun b 2-7 Tahun c 7-8 Tahun d 7-12 Tahun

Uraian Piaget membagi skema yang digunakan anak untuk memahami dunianya melalui empat periode utama yang berkorelasi

dengan dan semakin canggih seiring pertambahan usia Periode sensorimotor (usia 0ndash2 tahun) Periode praoperasional (usia 2ndash7 tahun) Periode operasional konkrit (usia 7ndash12 tahun) Periode operasional formal (usia 12 tahun sampai dewasa)

51 313 Memilih media pembelajaran yang tepat untuk pembelajaran operasi bilangan bulat Untuk operasi bilangan bulat perkalian media yang paling tepat digunakan adalah a sapu lidi

b kerikil c daun pakis d koin

Uraian 52 313 Memilih media pembelajaran yang tepat untuk pembelajaran operasi bilangan bulat

Untuk operasi bilangan bulat negatif kita bisa menggunakan media a uang kertas b kerikil c mistar d koin

Uraian 53 313 Memilih media pembelajaran yang tepat untuk pembelajaran operasi bilangan bulat

Beranda Soal UKG Agus Zainal M (SDN Gudang 2)

Republik_ilalangyahoocoid

Untuk operasi bilangan bulat penjumlahan media yang paling tepat digunakan adalah a uang kertas b kerikil c daun pakis d koin

Uraian 54 314 Memilih media pembelajaran yang tepat untuk pembelajaran operasi bilangan pecahan

Untuk operasi bilangan pecahan kita bisa menggunakan media berikut kecuali a kartu bilangan b garis bilangan c gambar bidang d blok pecahan

Uraian 55 316 Memilih media pembelajaran yang tepat untuk pembelajaran geometri dan pengukuran

Untuk menghitung luas dan keliling bangun datar media yang paling tepat digunakan adalah a Penggaris b Kertas Folio Bergaris

c Neraca d Kalkulator

Uraian

Page 10: 50 SOAL UKG & PEMBAHASAN SESUAI KISI-KISIS.pdf

Beranda Soal UKG Agus Zainal M (SDN Gudang 2)

Republik_ilalangyahoocoid

Uraian 33 KD 134 menemukan pesan pokok dalam wacana naratif seperti cerita rakyat puisi

Surat dari ibu Jika bayang telah pudar Dan elang laut pulang ke sarang Angin bertiup ke benua Tiang ndash tiang akan kering sendiri Dan nahkoda sudah tau pedoman Boleh engkau datang padaku Makna lambang dari nahkoda sudah tahu pedoman adalahhellip a Sudah mencari pedoman hidup b Sudah menemukan arah dan tujuan

c Sudah berilmu dan berpengalaman d Sudah mempunyai pasangan hidup

Uraian 34 KD 135 membandingkan berbagai jenis wacana bahasa indonesia (deskripsi narasi)

Pendekatan untuk mendapat tanggapan emosional pembaca ataupun kesan pembaca adalah contoh Pendekatan Deskripsi jenis a Pendekatan Ekspositoris b Pendekatan Impresionistik

c Pendekatan menurut sikap pengarang d Pendekatan Realistik

Uraian PENDEKATAN DESKRIPSI

Pendekatan dalam pendeskripsian dapat dibedakan menjadi beberapa kategori pendekatan yaitu

1 Pendekatan Ekspositoris

Dalam pendekatan ini kita berusaha agar deskripsi yang kita buat dapat memberi keterangan sesuai dengan keadan yang

sebenarnya sehingga pembaca dapat seolah-olah ikut melihat atau merasakan objek yang kita deskripsikan Karangan

jenis ini berisi daftar detail sesuatu secara lengkap sehingga pembaca dan penalarannya dapat memperoleh kesan

keseluruhan tentang sesuatu

2 Pendekatan Impresionistik

Tujuan deskripsi impresionistik ialah untuk mendapatkan tanggapan emosional pembaca ataupun kesan pembaca Corak

deskripsi ini diantaranya juga ditentukan oleh macam kesan apa yang diinginkan penulisnya

3 Pendekatan menurut Sikap Pengarang

Pendekatan ini sangat bergantung pada tujuan yang ingin dicapai sifat objek serta pembaca deskripsinya Dalam

menguraikan sebuah gagasan penulis mungkin mengharapkan agar pembaca merasa tidak puas terhadapa suatu

tindakan atau keadaan atau penulis menginginkan agar pembaca juga harus merasakan bahwa persoalan yang dihadapi

merupakan masalah yang gawat Penulis juga dapat membayangkan bahwa akan terjadi sesuatu yang tidak diinginkan

sehingga pembaca dari semula sudah disiapkan perasaan yang kurang enak seram takut dan sebagainya (Alkhaidah

1997) 35 kd 135 membandingkan berbagai jenis wacana bahasa indonesia (deskripsi narasi)

Prinsip-Prinsip karangan Narasi sebagai berikut kecuali a Alur b Penokohan c Amanat

d Sudut Pandang Uraian Prinsip-prinsip dasar narasi sebagai tumpuan berfikir adalah sebagai berikut

1 Alur (plot) biasa disebut dengan jalan cerita

2 Penokohan (rangkaian perbuatanaksi) 3 Latar (setting) yaitu tempat dan atau waktu terjadinya perbuatan tokoh atau peristiwa yang dialami tokoh 4 Sudut Pandang (Point of View)

a Narrator Serba Tahu (Omniscient point of view) Yaitu narrator bertindak sebagai pencipta segalanya b Narrator bertindak objektif (Objektive point of view) Pengarang tak member komentar apa pun dan pembaca bebas menafsirkan apa yang diceritakan pengarang c Narrator ikut aktif ( Narator acting) Narrator terlibat dalam cerita d Narrator sebagai peninjau

Beranda Soal UKG Agus Zainal M (SDN Gudang 2)

Republik_ilalangyahoocoid

Pelakunya adalah orang ketiga yang biasa disebut ldquodiardquo 36 KD 135 membandingkan berbagai jenis wacana bahasa indonesia (deskripsi narasi)

1 Menentukan tema atau amanat apa yang akan disampaikan 2 Menetapkan sasaran pembaca 3 Merancang peristiwa-peristiwa utama yang akan ditampilkan dalam bentuk skema perkembangandan akhir cerita 5 Memerinci peristiwa-peristiwa utama ke dalam detail-detail peristiwa sebagai pendukung cerita 6 Menyusun tokoh dan perwatakanserta latar dan sudut pandang Karakteristik diatas adalah pengembangan dari paragraf a Narasi

b Deskprisi c Ekspositoris d Sudut Pandang Persuasif

Uraian Pengembangan karangan narasi dapat dilakukan dengan langkah-langkah berikut

(1) menentukan tema atau amanat apa yang akan disampaikan (2) menetapkan sasaran pembaca (3) merancang peristiwa-peristiwa utama yang akan ditampilkan dalam bentuk skema alur (4) membagi peristiwa utama ke dalam ke dalam bagian awal perkembangan dan akhir cerita (5) memerinci peristiwa-peristiwa utama ke dalam detail-detail peristiwa sebagai pendukung cerita dan (6) menyusun tokoh dan perwatakan latar dan sudut pandang

37 KD 135 membandingkan berbagai jenis wacana bahasa indonesia (deskripsi narasi) Karangan narasi yang berusaha untuk memberikan suatu maksud tertentu menyampaikan suatu amanat terselubung kepada para pembaca atau pendengar sehingga tampak seolah-olah melihat disebut a Narasi Informatif b Narasi Ekspositoris c Narasi Sugestif d Narasi Artistik

Uraian Jenis-Jenis Narasi

Narasi Informatif Narasi informatif adalah narasi yang memiliki sasaran penyampaian informasi secara tepat tentang suatu peristiwa

dengan tujuan memperluas pengetahuan orang tentang kisah seseorang Narasi Ekspositorik

Narasi ekspositorik adalah narasi yang memiliki sasaran penyampaian informasi secara tepat tentang suatu peristiwa dengan tujuan memperluas pengetahuan orang tentang kisah seseorang Dalam narasi ekspositorik penulis menceritakan suatu peristiwa berdasarkan data yang sebenarnya Pelaku yang ditonjolkan biasanya satu orang Pelaku diceritakan mulai dari kecil sampai saat ini atau sampai terakhir dalam kehidupannya Karangan narasi ini diwarnai oleh eksposisi maka ketentuan eksposisi juga berlaku pada penulisan narasi ekspositprik Ketentuan ini berkaitan dengan penggunaan bahasa yang logis berdasarkan fakta yang ada tidak memasukan unsursugestif atau bersifat objektif

Narasi Artistik

Narasi artistik adalah narasi yang berusaha untuk memberikan suatu maksud tertentu menyampaikan suatu amanat terselubung kepada para pembaca atau pendengar sehingga tampak seolah-olah melihat Ketentuan ini berkaitan dengan penggunaan bahasa yang logis berdasarkan fakta yang ada tidak memasukan unsur sugestif atau bersifat objektif

Narasi Sugestif Narasi sugestif adalah narasi yang berusaha untuk memberikan suatu maksud tertentu menyampaikan suatu amanat

terselubung kepada para pembaca atau pendengar sehingga tampak seolah-olah melihat 38 KD 135 membandingkan berbagai jenis wacana bahasa indonesia (deskripsi narasi)

Ciri karangan Narasi yang benar kecuali a Menonjolkan unsur perbuatan atau tindakan Membuat pembaca atau pendengar merasakan sendiri atau

mengalami sendiri

b Ada konfiks menjawab pertanyaan apa yang terjadi c Dirangkai dalam urutan waktu menggambarkan dengan jelas suatu peristiwa d Berisi ajakan dirangkai dalam urutan waktu

Uraian Ciri-Ciri Karangan Narasi Menurut Gorys Keraf (2000136)

Menonjolkan unsur perbuatan atau tindakan Dirangkai dalam urutan waktu Berusaha menjawab pertanyaan apa yang terjadi Ada konfiks

39 kd 135 membandingkan berbagai jenis wacana bahasa indonesia (deskripsi narasi) Langkah menyusun paragraf deskripsi yang benar adalah (1)Tentukan objek atau tema yang akan dideskripsikan (2)Mengumpulkan data dengan mengamati objek yang akan dideskripsikan (3)Menyusun data tersebut ke dalam urutan yang baik (menyusun kerangka karangan) (4)Menguraikan kerangka karangan menjadi dekripsi yang sesuai dengan tema yang ditentukan

Beranda Soal UKG Agus Zainal M (SDN Gudang 2)

Republik_ilalangyahoocoid

(5)Tentukan tujuan a 1-5-2-3-4

b 1-5-4-3-2 c 1-5-3-2-4 d 1-3-4-2-5

Uraian Langkah menyusun deskripsi

1Tentukan objek atau tema yang akan dideskripsikan 2Tentukan tujuan 3Mengumpulkan data dengan mengamati objek yang akan dideskripsikan 4Menyusun data tersebut ke dalam urutan yang baik (menyusun kerangka karangan) 5Menguraikan kerangka karangan menjadi dekripsi yang sesuai dengan tema yang ditentukan

40 Kd 311 Merancang aktivitas pembelajaran berdasarkan prinsip dan teori pembelajaran matematika Dalam tahap ini penyajian yang dilakukan melalui tindakan anak secara langsung terlihat dalam memanipulasi (mengotak atik)objek Proses tersebut menurut Bruner dikategorikan ke dalam model a Tahap Enaktif

b Tahap Ikonik c Tahap Simbolik d Tahap Implikatif

Uraian 1 Model Tahap Enaktif

Dalam tahap ini penyajian yang dilakukan melalui tindakan anak secara langsung terlihat dalam memanipulasi (mengotak atik)objek Contoh Budi mempunyai 2 pensil kemudian ibunya memberikannya lagi 3 pinsil Berapa banyak pensil Budi sekarang

2 Model Tahap Ikonik Dalam tahap ini kegiatan penyajian dilakukan berdasarkan pada pikiran internal dimana pengetahuan disajikan melalui serangkaian gambar-gambar atau grafik yang dilakukan anak berhubungan dengan mental yang merupakan gambaran dari objek-objek yang dimanipulasinya Contoh + = hellip

3 Model Tahap Simbolis Dalam tahap ini bahasa adalah pola dasar simbolik anak memanipulasi Simbol-simbol atau lambang-lambang objek tertentu Contoh 2 pensil + 3 pensil = hellip pensil

41 Kd 311 Merancang aktivitas pembelajaran berdasarkan prinsip dan teori pembelajaran matematika Dalam tahap ini kegiatan penyajian dilakukan berdasarkan pada pikiran internal dimana pengetahuan disajikan melalui serangkaian gambar-gambar atau grafik yang dilakukan anak Proses tersebut menurut Bruner dikategorikan ke dalam model a Tahap Enaktif b Tahap Ikonik

c Tahap Simbolik d Tahap Implikatif

Uraian No 40 42 Kd 311 Merancang aktivitas pembelajaran berdasarkan prinsip dan teori pembelajaran matematika

Objek belajar matematika dibagi kedalam Objek Langsung dan Objek Tak Langsung Hal tersebut adalah teori belajar Matematika menurut a Robert M Gagne

b Jerome S Burner c Thorndike d Skinner

Uraian Teori yang diperkenalkan Robert MGagne pada tahun 1960-an pembelajaran harus dikondisikan untuk memunculkan

respons yang diharapkanMenurut Gagne (dalam Ismail 1998) belajar matematika terdiri dari objek langsung dan objek tak langsung

43 Kd 311 Merancang aktivitas pembelajaran berdasarkan prinsip dan teori pembelajaran matematika Perhatikan contoh berikut Budi mempunyai 2 pinsil kemudian ibunya memberikannya lagi 3 pinsilBerapa banyak pinsil Budi sekarang Hal tersebut dikemukakan Bruner dalam Proses Pembelajaran Matematika dalam tahap a Simbolik b Ikonik c Implikatif d Enaktif

Uraian No 40

Beranda Soal UKG Agus Zainal M (SDN Gudang 2)

Republik_ilalangyahoocoid

44 Kd 311 Merancang aktivitas pembelajaran berdasarkan prinsip dan teori pembelajaran matematika Perhatikan gambar disamping Ilustrasi di samping dikemukakan Bruner dalam Proses Pembelajaran Matematika dalam tahap

a Simbolik b Ikonik

c Implikatif d Enaktif

Uraian No 40 45 Kd 311 Merancang aktivitas pembelajaran berdasarkan prinsip dan teori pembelajaran matematika

Perhatikan contoh berikut Contoh 2 pinsil + 3 pinsil = hellippinsil Contoh tersebut dikemukakan Bruner dalam Proses Pembelajaran Matematika dalam tahap a Simbolik

b Ikonik c Implikatif d Enaktif

Uraian No 40 46 Kd 311 Merancang aktivitas pembelajaran berdasarkan prinsip dan teori pembelajaran matematika

Perhatikan contoh berikut penjumlahan bilangan positif dan negatif siswa mencoba sendiri dengan menggunakan garis bilangan Contoh tersebut dikemukakan Bruner dalam Teorema Pembelajaran Matematika a Penyusunan

b Notasi c Pengkontrasan dan Keanekaragaman d Pengaitan

Uraian Berdasarkan hasil pengamatannya Brunner merumuskan 5 teorema dalam pembelajaran matematika yaitu

1) Teorema Penyusunan Menerangkan bahwa cara yang terbaik memulai belajar suatu konsep matematika dalil defenisi dan semacamnya adalah dengan cara menyusun penyajiannya Misalnya dalam mempelajari penjumlahan bilangan positif dan negatif siswa mencoba sendiri dengan menggunakan garis bilangan 2) Teorema Notasi Menerangkan bahwa dalam pengajaran suatu konsep penggunaan notasi-notasi matematika harus diberikan secara bertahap dari yang sederhana ke yang lebih kompleks 3) Teorema Pengkontrasan dan Keanekaragaman Menerangkan bahwa pengontrasan dan keanekaragaman sangat penting dalam melakukan pengubahan konsep matematika dari yang konkrit ke yang lebih abstrak Dalam hal ini diperlukan banyak contoh Contoh yang diberikan harus sesuai dengan rumusan yang diberikan Misalnya menjelaskan persegi panjang disertai juga kemungkinan jajaran genjang dan segi empat lainnya selain persegi panjnag Dengan demikian siswa dapat membedakan apakah segi empat yang diberikan padanya termasuk persegi panjang atau tidak 4) Teorema Pengaitan Menerangkan bahwa dalam matematika terdapat hubungan yang berkaitan antara satu konsep dengan konsep yang lain Di mana materi yang satu merupakan prasyarat yang harus diketahui untuk mempelajari materi yang lain

47 Kd 311 Merancang aktivitas pembelajaran berdasarkan prinsip dan teori pembelajaran matematika Perhatikan contoh berikut Guru menjelaskan persegi panjang disertai juga kemungkinan jajaran genjang dan segi empat lainnya selain persegi panjnag Dengan demikian siswa dapat membedakan apakah segi empat yang diberikan padanya termasuk persegi panjang atau tidak Contoh tersebut dikemukakan Bruner dalam Teorema Pembelajaran Matematika a Penyusunan b Notasi c Pengkontrasan dan Keanekaragaman

d Pengaitan Uraian No 46 48 Kd 311 Merancang aktivitas pembelajaran berdasarkan prinsip dan teori pembelajaran matematika

Tahap pembelajaran Matematika menurut Van Halle adalah sebagai berikut kecuali a Tahap Pengenalan b Tahap Pengurutan c Tahap Analisis d Tahap Penyimpulan

Beranda Soal UKG Agus Zainal M (SDN Gudang 2)

Republik_ilalangyahoocoid

Uraian Van Hiele menyatakan bahwa terdapat 5 tahap belajar siswa dalam belajar geometri yaitu

a Tahap Pengenalan Pada tahap ini siswa mulai belajar mengenal suatu bangun geometri secara keseluruhan namun belum mampu mengetahui adanya sifat-sifat dari bangun geometri yang dilihatnya b Tahap Analisis Pada tahap ini siswa sudah mulai mengenal sifat-sifat yang dimiliki bangun geometri yang diamatinya c Tahap Pengurutan Pada tahap ini siswa sudah mengenal dan memahami sifat-sifat suatu bangun geometri serta sudah dapat mengurutkan bangun-bangun geometri yang satu sama yang lainnya saling berhubungan d Tahap Deduksi Pada tahap ini siswa telah mampu menarik kesimpulan secara deduktif yaitu menarik kesimpulan yang bersifat umum dan menuju ke hal yang bersifat khusus serta dapat mengambil kesimpulan e Tahap Akurasi Pada tahap ini siswa mulai menyadari pentingnya ketepatan prinsip-prinsip dasar yang melandasi suatu pembuktian Tahap berfikir ini merupakan tahap berfikir yang paling tinggi rumit dan kompleks karena di luar jangkauan usia anak-anak SD sampai tingakat SMP

49 Kd 311 Merancang aktivitas pembelajaran berdasarkan prinsip dan teori pembelajaran matematika middot Matematika untuk tujuan pembelajaran dianalisis sebagai kumpulan fakta yang berdiri sendiri dan tidak saling berkaitan middot Anak diharuskan menguasai unsur-unsur yang banyak sekali tanpa diperhatikan pengertiannya middot Anak mempelajari unsur-unsur dalam bentuk seperti yang akan digunakan nanti dalam kesempatan lain middot Anak akan mencapai tujuan ini secara efektif dan efisien dengan melalui pengulangan Teori Pembelajaran Matematika tersebut dikemukakan oleh a Skinner b Piaget c Van Brownell

d Thorndike Uraian Brownell mengemukakan tentang Teori Makna (Meaning Theory) sebagai pengganti Teori Latihan HafalUlangan (Drill

Theory) Intisari dari teori Drill adalah - Matematika untuk tujuan pembelajaran dianalisis sebagai kumpulan fakta yang berdiri sendiri dan tidak saling berkaitan - Anak diharuskan menguasai unsur-unsur yang banyak sekali tanpa diperhatikan pengertiannya - Anak mempelajari unsur-unsur dalam bentuk seperti yang akan digunakan nanti dalam kesempatan lain - Anak akan mencapai tujuan ini secara efektif dan efisien dengan melalui pengulangan

50 Kd 311 Merancang aktivitas pembelajaran berdasarkan prinsip dan teori pembelajaran matematika Tahap operasionaloperasi konkrit adalah teori Pembelajaran Matematika yang dikemukakan Peaget pada tahap usia a dibawah 2 tahun b 2-7 Tahun c 7-8 Tahun d 7-12 Tahun

Uraian Piaget membagi skema yang digunakan anak untuk memahami dunianya melalui empat periode utama yang berkorelasi

dengan dan semakin canggih seiring pertambahan usia Periode sensorimotor (usia 0ndash2 tahun) Periode praoperasional (usia 2ndash7 tahun) Periode operasional konkrit (usia 7ndash12 tahun) Periode operasional formal (usia 12 tahun sampai dewasa)

51 313 Memilih media pembelajaran yang tepat untuk pembelajaran operasi bilangan bulat Untuk operasi bilangan bulat perkalian media yang paling tepat digunakan adalah a sapu lidi

b kerikil c daun pakis d koin

Uraian 52 313 Memilih media pembelajaran yang tepat untuk pembelajaran operasi bilangan bulat

Untuk operasi bilangan bulat negatif kita bisa menggunakan media a uang kertas b kerikil c mistar d koin

Uraian 53 313 Memilih media pembelajaran yang tepat untuk pembelajaran operasi bilangan bulat

Beranda Soal UKG Agus Zainal M (SDN Gudang 2)

Republik_ilalangyahoocoid

Untuk operasi bilangan bulat penjumlahan media yang paling tepat digunakan adalah a uang kertas b kerikil c daun pakis d koin

Uraian 54 314 Memilih media pembelajaran yang tepat untuk pembelajaran operasi bilangan pecahan

Untuk operasi bilangan pecahan kita bisa menggunakan media berikut kecuali a kartu bilangan b garis bilangan c gambar bidang d blok pecahan

Uraian 55 316 Memilih media pembelajaran yang tepat untuk pembelajaran geometri dan pengukuran

Untuk menghitung luas dan keliling bangun datar media yang paling tepat digunakan adalah a Penggaris b Kertas Folio Bergaris

c Neraca d Kalkulator

Uraian

Page 11: 50 SOAL UKG & PEMBAHASAN SESUAI KISI-KISIS.pdf

Beranda Soal UKG Agus Zainal M (SDN Gudang 2)

Republik_ilalangyahoocoid

Pelakunya adalah orang ketiga yang biasa disebut ldquodiardquo 36 KD 135 membandingkan berbagai jenis wacana bahasa indonesia (deskripsi narasi)

1 Menentukan tema atau amanat apa yang akan disampaikan 2 Menetapkan sasaran pembaca 3 Merancang peristiwa-peristiwa utama yang akan ditampilkan dalam bentuk skema perkembangandan akhir cerita 5 Memerinci peristiwa-peristiwa utama ke dalam detail-detail peristiwa sebagai pendukung cerita 6 Menyusun tokoh dan perwatakanserta latar dan sudut pandang Karakteristik diatas adalah pengembangan dari paragraf a Narasi

b Deskprisi c Ekspositoris d Sudut Pandang Persuasif

Uraian Pengembangan karangan narasi dapat dilakukan dengan langkah-langkah berikut

(1) menentukan tema atau amanat apa yang akan disampaikan (2) menetapkan sasaran pembaca (3) merancang peristiwa-peristiwa utama yang akan ditampilkan dalam bentuk skema alur (4) membagi peristiwa utama ke dalam ke dalam bagian awal perkembangan dan akhir cerita (5) memerinci peristiwa-peristiwa utama ke dalam detail-detail peristiwa sebagai pendukung cerita dan (6) menyusun tokoh dan perwatakan latar dan sudut pandang

37 KD 135 membandingkan berbagai jenis wacana bahasa indonesia (deskripsi narasi) Karangan narasi yang berusaha untuk memberikan suatu maksud tertentu menyampaikan suatu amanat terselubung kepada para pembaca atau pendengar sehingga tampak seolah-olah melihat disebut a Narasi Informatif b Narasi Ekspositoris c Narasi Sugestif d Narasi Artistik

Uraian Jenis-Jenis Narasi

Narasi Informatif Narasi informatif adalah narasi yang memiliki sasaran penyampaian informasi secara tepat tentang suatu peristiwa

dengan tujuan memperluas pengetahuan orang tentang kisah seseorang Narasi Ekspositorik

Narasi ekspositorik adalah narasi yang memiliki sasaran penyampaian informasi secara tepat tentang suatu peristiwa dengan tujuan memperluas pengetahuan orang tentang kisah seseorang Dalam narasi ekspositorik penulis menceritakan suatu peristiwa berdasarkan data yang sebenarnya Pelaku yang ditonjolkan biasanya satu orang Pelaku diceritakan mulai dari kecil sampai saat ini atau sampai terakhir dalam kehidupannya Karangan narasi ini diwarnai oleh eksposisi maka ketentuan eksposisi juga berlaku pada penulisan narasi ekspositprik Ketentuan ini berkaitan dengan penggunaan bahasa yang logis berdasarkan fakta yang ada tidak memasukan unsursugestif atau bersifat objektif

Narasi Artistik

Narasi artistik adalah narasi yang berusaha untuk memberikan suatu maksud tertentu menyampaikan suatu amanat terselubung kepada para pembaca atau pendengar sehingga tampak seolah-olah melihat Ketentuan ini berkaitan dengan penggunaan bahasa yang logis berdasarkan fakta yang ada tidak memasukan unsur sugestif atau bersifat objektif

Narasi Sugestif Narasi sugestif adalah narasi yang berusaha untuk memberikan suatu maksud tertentu menyampaikan suatu amanat

terselubung kepada para pembaca atau pendengar sehingga tampak seolah-olah melihat 38 KD 135 membandingkan berbagai jenis wacana bahasa indonesia (deskripsi narasi)

Ciri karangan Narasi yang benar kecuali a Menonjolkan unsur perbuatan atau tindakan Membuat pembaca atau pendengar merasakan sendiri atau

mengalami sendiri

b Ada konfiks menjawab pertanyaan apa yang terjadi c Dirangkai dalam urutan waktu menggambarkan dengan jelas suatu peristiwa d Berisi ajakan dirangkai dalam urutan waktu

Uraian Ciri-Ciri Karangan Narasi Menurut Gorys Keraf (2000136)

Menonjolkan unsur perbuatan atau tindakan Dirangkai dalam urutan waktu Berusaha menjawab pertanyaan apa yang terjadi Ada konfiks

39 kd 135 membandingkan berbagai jenis wacana bahasa indonesia (deskripsi narasi) Langkah menyusun paragraf deskripsi yang benar adalah (1)Tentukan objek atau tema yang akan dideskripsikan (2)Mengumpulkan data dengan mengamati objek yang akan dideskripsikan (3)Menyusun data tersebut ke dalam urutan yang baik (menyusun kerangka karangan) (4)Menguraikan kerangka karangan menjadi dekripsi yang sesuai dengan tema yang ditentukan

Beranda Soal UKG Agus Zainal M (SDN Gudang 2)

Republik_ilalangyahoocoid

(5)Tentukan tujuan a 1-5-2-3-4

b 1-5-4-3-2 c 1-5-3-2-4 d 1-3-4-2-5

Uraian Langkah menyusun deskripsi

1Tentukan objek atau tema yang akan dideskripsikan 2Tentukan tujuan 3Mengumpulkan data dengan mengamati objek yang akan dideskripsikan 4Menyusun data tersebut ke dalam urutan yang baik (menyusun kerangka karangan) 5Menguraikan kerangka karangan menjadi dekripsi yang sesuai dengan tema yang ditentukan

40 Kd 311 Merancang aktivitas pembelajaran berdasarkan prinsip dan teori pembelajaran matematika Dalam tahap ini penyajian yang dilakukan melalui tindakan anak secara langsung terlihat dalam memanipulasi (mengotak atik)objek Proses tersebut menurut Bruner dikategorikan ke dalam model a Tahap Enaktif

b Tahap Ikonik c Tahap Simbolik d Tahap Implikatif

Uraian 1 Model Tahap Enaktif

Dalam tahap ini penyajian yang dilakukan melalui tindakan anak secara langsung terlihat dalam memanipulasi (mengotak atik)objek Contoh Budi mempunyai 2 pensil kemudian ibunya memberikannya lagi 3 pinsil Berapa banyak pensil Budi sekarang

2 Model Tahap Ikonik Dalam tahap ini kegiatan penyajian dilakukan berdasarkan pada pikiran internal dimana pengetahuan disajikan melalui serangkaian gambar-gambar atau grafik yang dilakukan anak berhubungan dengan mental yang merupakan gambaran dari objek-objek yang dimanipulasinya Contoh + = hellip

3 Model Tahap Simbolis Dalam tahap ini bahasa adalah pola dasar simbolik anak memanipulasi Simbol-simbol atau lambang-lambang objek tertentu Contoh 2 pensil + 3 pensil = hellip pensil

41 Kd 311 Merancang aktivitas pembelajaran berdasarkan prinsip dan teori pembelajaran matematika Dalam tahap ini kegiatan penyajian dilakukan berdasarkan pada pikiran internal dimana pengetahuan disajikan melalui serangkaian gambar-gambar atau grafik yang dilakukan anak Proses tersebut menurut Bruner dikategorikan ke dalam model a Tahap Enaktif b Tahap Ikonik

c Tahap Simbolik d Tahap Implikatif

Uraian No 40 42 Kd 311 Merancang aktivitas pembelajaran berdasarkan prinsip dan teori pembelajaran matematika

Objek belajar matematika dibagi kedalam Objek Langsung dan Objek Tak Langsung Hal tersebut adalah teori belajar Matematika menurut a Robert M Gagne

b Jerome S Burner c Thorndike d Skinner

Uraian Teori yang diperkenalkan Robert MGagne pada tahun 1960-an pembelajaran harus dikondisikan untuk memunculkan

respons yang diharapkanMenurut Gagne (dalam Ismail 1998) belajar matematika terdiri dari objek langsung dan objek tak langsung

43 Kd 311 Merancang aktivitas pembelajaran berdasarkan prinsip dan teori pembelajaran matematika Perhatikan contoh berikut Budi mempunyai 2 pinsil kemudian ibunya memberikannya lagi 3 pinsilBerapa banyak pinsil Budi sekarang Hal tersebut dikemukakan Bruner dalam Proses Pembelajaran Matematika dalam tahap a Simbolik b Ikonik c Implikatif d Enaktif

Uraian No 40

Beranda Soal UKG Agus Zainal M (SDN Gudang 2)

Republik_ilalangyahoocoid

44 Kd 311 Merancang aktivitas pembelajaran berdasarkan prinsip dan teori pembelajaran matematika Perhatikan gambar disamping Ilustrasi di samping dikemukakan Bruner dalam Proses Pembelajaran Matematika dalam tahap

a Simbolik b Ikonik

c Implikatif d Enaktif

Uraian No 40 45 Kd 311 Merancang aktivitas pembelajaran berdasarkan prinsip dan teori pembelajaran matematika

Perhatikan contoh berikut Contoh 2 pinsil + 3 pinsil = hellippinsil Contoh tersebut dikemukakan Bruner dalam Proses Pembelajaran Matematika dalam tahap a Simbolik

b Ikonik c Implikatif d Enaktif

Uraian No 40 46 Kd 311 Merancang aktivitas pembelajaran berdasarkan prinsip dan teori pembelajaran matematika

Perhatikan contoh berikut penjumlahan bilangan positif dan negatif siswa mencoba sendiri dengan menggunakan garis bilangan Contoh tersebut dikemukakan Bruner dalam Teorema Pembelajaran Matematika a Penyusunan

b Notasi c Pengkontrasan dan Keanekaragaman d Pengaitan

Uraian Berdasarkan hasil pengamatannya Brunner merumuskan 5 teorema dalam pembelajaran matematika yaitu

1) Teorema Penyusunan Menerangkan bahwa cara yang terbaik memulai belajar suatu konsep matematika dalil defenisi dan semacamnya adalah dengan cara menyusun penyajiannya Misalnya dalam mempelajari penjumlahan bilangan positif dan negatif siswa mencoba sendiri dengan menggunakan garis bilangan 2) Teorema Notasi Menerangkan bahwa dalam pengajaran suatu konsep penggunaan notasi-notasi matematika harus diberikan secara bertahap dari yang sederhana ke yang lebih kompleks 3) Teorema Pengkontrasan dan Keanekaragaman Menerangkan bahwa pengontrasan dan keanekaragaman sangat penting dalam melakukan pengubahan konsep matematika dari yang konkrit ke yang lebih abstrak Dalam hal ini diperlukan banyak contoh Contoh yang diberikan harus sesuai dengan rumusan yang diberikan Misalnya menjelaskan persegi panjang disertai juga kemungkinan jajaran genjang dan segi empat lainnya selain persegi panjnag Dengan demikian siswa dapat membedakan apakah segi empat yang diberikan padanya termasuk persegi panjang atau tidak 4) Teorema Pengaitan Menerangkan bahwa dalam matematika terdapat hubungan yang berkaitan antara satu konsep dengan konsep yang lain Di mana materi yang satu merupakan prasyarat yang harus diketahui untuk mempelajari materi yang lain

47 Kd 311 Merancang aktivitas pembelajaran berdasarkan prinsip dan teori pembelajaran matematika Perhatikan contoh berikut Guru menjelaskan persegi panjang disertai juga kemungkinan jajaran genjang dan segi empat lainnya selain persegi panjnag Dengan demikian siswa dapat membedakan apakah segi empat yang diberikan padanya termasuk persegi panjang atau tidak Contoh tersebut dikemukakan Bruner dalam Teorema Pembelajaran Matematika a Penyusunan b Notasi c Pengkontrasan dan Keanekaragaman

d Pengaitan Uraian No 46 48 Kd 311 Merancang aktivitas pembelajaran berdasarkan prinsip dan teori pembelajaran matematika

Tahap pembelajaran Matematika menurut Van Halle adalah sebagai berikut kecuali a Tahap Pengenalan b Tahap Pengurutan c Tahap Analisis d Tahap Penyimpulan

Beranda Soal UKG Agus Zainal M (SDN Gudang 2)

Republik_ilalangyahoocoid

Uraian Van Hiele menyatakan bahwa terdapat 5 tahap belajar siswa dalam belajar geometri yaitu

a Tahap Pengenalan Pada tahap ini siswa mulai belajar mengenal suatu bangun geometri secara keseluruhan namun belum mampu mengetahui adanya sifat-sifat dari bangun geometri yang dilihatnya b Tahap Analisis Pada tahap ini siswa sudah mulai mengenal sifat-sifat yang dimiliki bangun geometri yang diamatinya c Tahap Pengurutan Pada tahap ini siswa sudah mengenal dan memahami sifat-sifat suatu bangun geometri serta sudah dapat mengurutkan bangun-bangun geometri yang satu sama yang lainnya saling berhubungan d Tahap Deduksi Pada tahap ini siswa telah mampu menarik kesimpulan secara deduktif yaitu menarik kesimpulan yang bersifat umum dan menuju ke hal yang bersifat khusus serta dapat mengambil kesimpulan e Tahap Akurasi Pada tahap ini siswa mulai menyadari pentingnya ketepatan prinsip-prinsip dasar yang melandasi suatu pembuktian Tahap berfikir ini merupakan tahap berfikir yang paling tinggi rumit dan kompleks karena di luar jangkauan usia anak-anak SD sampai tingakat SMP

49 Kd 311 Merancang aktivitas pembelajaran berdasarkan prinsip dan teori pembelajaran matematika middot Matematika untuk tujuan pembelajaran dianalisis sebagai kumpulan fakta yang berdiri sendiri dan tidak saling berkaitan middot Anak diharuskan menguasai unsur-unsur yang banyak sekali tanpa diperhatikan pengertiannya middot Anak mempelajari unsur-unsur dalam bentuk seperti yang akan digunakan nanti dalam kesempatan lain middot Anak akan mencapai tujuan ini secara efektif dan efisien dengan melalui pengulangan Teori Pembelajaran Matematika tersebut dikemukakan oleh a Skinner b Piaget c Van Brownell

d Thorndike Uraian Brownell mengemukakan tentang Teori Makna (Meaning Theory) sebagai pengganti Teori Latihan HafalUlangan (Drill

Theory) Intisari dari teori Drill adalah - Matematika untuk tujuan pembelajaran dianalisis sebagai kumpulan fakta yang berdiri sendiri dan tidak saling berkaitan - Anak diharuskan menguasai unsur-unsur yang banyak sekali tanpa diperhatikan pengertiannya - Anak mempelajari unsur-unsur dalam bentuk seperti yang akan digunakan nanti dalam kesempatan lain - Anak akan mencapai tujuan ini secara efektif dan efisien dengan melalui pengulangan

50 Kd 311 Merancang aktivitas pembelajaran berdasarkan prinsip dan teori pembelajaran matematika Tahap operasionaloperasi konkrit adalah teori Pembelajaran Matematika yang dikemukakan Peaget pada tahap usia a dibawah 2 tahun b 2-7 Tahun c 7-8 Tahun d 7-12 Tahun

Uraian Piaget membagi skema yang digunakan anak untuk memahami dunianya melalui empat periode utama yang berkorelasi

dengan dan semakin canggih seiring pertambahan usia Periode sensorimotor (usia 0ndash2 tahun) Periode praoperasional (usia 2ndash7 tahun) Periode operasional konkrit (usia 7ndash12 tahun) Periode operasional formal (usia 12 tahun sampai dewasa)

51 313 Memilih media pembelajaran yang tepat untuk pembelajaran operasi bilangan bulat Untuk operasi bilangan bulat perkalian media yang paling tepat digunakan adalah a sapu lidi

b kerikil c daun pakis d koin

Uraian 52 313 Memilih media pembelajaran yang tepat untuk pembelajaran operasi bilangan bulat

Untuk operasi bilangan bulat negatif kita bisa menggunakan media a uang kertas b kerikil c mistar d koin

Uraian 53 313 Memilih media pembelajaran yang tepat untuk pembelajaran operasi bilangan bulat

Beranda Soal UKG Agus Zainal M (SDN Gudang 2)

Republik_ilalangyahoocoid

Untuk operasi bilangan bulat penjumlahan media yang paling tepat digunakan adalah a uang kertas b kerikil c daun pakis d koin

Uraian 54 314 Memilih media pembelajaran yang tepat untuk pembelajaran operasi bilangan pecahan

Untuk operasi bilangan pecahan kita bisa menggunakan media berikut kecuali a kartu bilangan b garis bilangan c gambar bidang d blok pecahan

Uraian 55 316 Memilih media pembelajaran yang tepat untuk pembelajaran geometri dan pengukuran

Untuk menghitung luas dan keliling bangun datar media yang paling tepat digunakan adalah a Penggaris b Kertas Folio Bergaris

c Neraca d Kalkulator

Uraian

Page 12: 50 SOAL UKG & PEMBAHASAN SESUAI KISI-KISIS.pdf

Beranda Soal UKG Agus Zainal M (SDN Gudang 2)

Republik_ilalangyahoocoid

(5)Tentukan tujuan a 1-5-2-3-4

b 1-5-4-3-2 c 1-5-3-2-4 d 1-3-4-2-5

Uraian Langkah menyusun deskripsi

1Tentukan objek atau tema yang akan dideskripsikan 2Tentukan tujuan 3Mengumpulkan data dengan mengamati objek yang akan dideskripsikan 4Menyusun data tersebut ke dalam urutan yang baik (menyusun kerangka karangan) 5Menguraikan kerangka karangan menjadi dekripsi yang sesuai dengan tema yang ditentukan

40 Kd 311 Merancang aktivitas pembelajaran berdasarkan prinsip dan teori pembelajaran matematika Dalam tahap ini penyajian yang dilakukan melalui tindakan anak secara langsung terlihat dalam memanipulasi (mengotak atik)objek Proses tersebut menurut Bruner dikategorikan ke dalam model a Tahap Enaktif

b Tahap Ikonik c Tahap Simbolik d Tahap Implikatif

Uraian 1 Model Tahap Enaktif

Dalam tahap ini penyajian yang dilakukan melalui tindakan anak secara langsung terlihat dalam memanipulasi (mengotak atik)objek Contoh Budi mempunyai 2 pensil kemudian ibunya memberikannya lagi 3 pinsil Berapa banyak pensil Budi sekarang

2 Model Tahap Ikonik Dalam tahap ini kegiatan penyajian dilakukan berdasarkan pada pikiran internal dimana pengetahuan disajikan melalui serangkaian gambar-gambar atau grafik yang dilakukan anak berhubungan dengan mental yang merupakan gambaran dari objek-objek yang dimanipulasinya Contoh + = hellip

3 Model Tahap Simbolis Dalam tahap ini bahasa adalah pola dasar simbolik anak memanipulasi Simbol-simbol atau lambang-lambang objek tertentu Contoh 2 pensil + 3 pensil = hellip pensil

41 Kd 311 Merancang aktivitas pembelajaran berdasarkan prinsip dan teori pembelajaran matematika Dalam tahap ini kegiatan penyajian dilakukan berdasarkan pada pikiran internal dimana pengetahuan disajikan melalui serangkaian gambar-gambar atau grafik yang dilakukan anak Proses tersebut menurut Bruner dikategorikan ke dalam model a Tahap Enaktif b Tahap Ikonik

c Tahap Simbolik d Tahap Implikatif

Uraian No 40 42 Kd 311 Merancang aktivitas pembelajaran berdasarkan prinsip dan teori pembelajaran matematika

Objek belajar matematika dibagi kedalam Objek Langsung dan Objek Tak Langsung Hal tersebut adalah teori belajar Matematika menurut a Robert M Gagne

b Jerome S Burner c Thorndike d Skinner

Uraian Teori yang diperkenalkan Robert MGagne pada tahun 1960-an pembelajaran harus dikondisikan untuk memunculkan

respons yang diharapkanMenurut Gagne (dalam Ismail 1998) belajar matematika terdiri dari objek langsung dan objek tak langsung

43 Kd 311 Merancang aktivitas pembelajaran berdasarkan prinsip dan teori pembelajaran matematika Perhatikan contoh berikut Budi mempunyai 2 pinsil kemudian ibunya memberikannya lagi 3 pinsilBerapa banyak pinsil Budi sekarang Hal tersebut dikemukakan Bruner dalam Proses Pembelajaran Matematika dalam tahap a Simbolik b Ikonik c Implikatif d Enaktif

Uraian No 40

Beranda Soal UKG Agus Zainal M (SDN Gudang 2)

Republik_ilalangyahoocoid

44 Kd 311 Merancang aktivitas pembelajaran berdasarkan prinsip dan teori pembelajaran matematika Perhatikan gambar disamping Ilustrasi di samping dikemukakan Bruner dalam Proses Pembelajaran Matematika dalam tahap

a Simbolik b Ikonik

c Implikatif d Enaktif

Uraian No 40 45 Kd 311 Merancang aktivitas pembelajaran berdasarkan prinsip dan teori pembelajaran matematika

Perhatikan contoh berikut Contoh 2 pinsil + 3 pinsil = hellippinsil Contoh tersebut dikemukakan Bruner dalam Proses Pembelajaran Matematika dalam tahap a Simbolik

b Ikonik c Implikatif d Enaktif

Uraian No 40 46 Kd 311 Merancang aktivitas pembelajaran berdasarkan prinsip dan teori pembelajaran matematika

Perhatikan contoh berikut penjumlahan bilangan positif dan negatif siswa mencoba sendiri dengan menggunakan garis bilangan Contoh tersebut dikemukakan Bruner dalam Teorema Pembelajaran Matematika a Penyusunan

b Notasi c Pengkontrasan dan Keanekaragaman d Pengaitan

Uraian Berdasarkan hasil pengamatannya Brunner merumuskan 5 teorema dalam pembelajaran matematika yaitu

1) Teorema Penyusunan Menerangkan bahwa cara yang terbaik memulai belajar suatu konsep matematika dalil defenisi dan semacamnya adalah dengan cara menyusun penyajiannya Misalnya dalam mempelajari penjumlahan bilangan positif dan negatif siswa mencoba sendiri dengan menggunakan garis bilangan 2) Teorema Notasi Menerangkan bahwa dalam pengajaran suatu konsep penggunaan notasi-notasi matematika harus diberikan secara bertahap dari yang sederhana ke yang lebih kompleks 3) Teorema Pengkontrasan dan Keanekaragaman Menerangkan bahwa pengontrasan dan keanekaragaman sangat penting dalam melakukan pengubahan konsep matematika dari yang konkrit ke yang lebih abstrak Dalam hal ini diperlukan banyak contoh Contoh yang diberikan harus sesuai dengan rumusan yang diberikan Misalnya menjelaskan persegi panjang disertai juga kemungkinan jajaran genjang dan segi empat lainnya selain persegi panjnag Dengan demikian siswa dapat membedakan apakah segi empat yang diberikan padanya termasuk persegi panjang atau tidak 4) Teorema Pengaitan Menerangkan bahwa dalam matematika terdapat hubungan yang berkaitan antara satu konsep dengan konsep yang lain Di mana materi yang satu merupakan prasyarat yang harus diketahui untuk mempelajari materi yang lain

47 Kd 311 Merancang aktivitas pembelajaran berdasarkan prinsip dan teori pembelajaran matematika Perhatikan contoh berikut Guru menjelaskan persegi panjang disertai juga kemungkinan jajaran genjang dan segi empat lainnya selain persegi panjnag Dengan demikian siswa dapat membedakan apakah segi empat yang diberikan padanya termasuk persegi panjang atau tidak Contoh tersebut dikemukakan Bruner dalam Teorema Pembelajaran Matematika a Penyusunan b Notasi c Pengkontrasan dan Keanekaragaman

d Pengaitan Uraian No 46 48 Kd 311 Merancang aktivitas pembelajaran berdasarkan prinsip dan teori pembelajaran matematika

Tahap pembelajaran Matematika menurut Van Halle adalah sebagai berikut kecuali a Tahap Pengenalan b Tahap Pengurutan c Tahap Analisis d Tahap Penyimpulan

Beranda Soal UKG Agus Zainal M (SDN Gudang 2)

Republik_ilalangyahoocoid

Uraian Van Hiele menyatakan bahwa terdapat 5 tahap belajar siswa dalam belajar geometri yaitu

a Tahap Pengenalan Pada tahap ini siswa mulai belajar mengenal suatu bangun geometri secara keseluruhan namun belum mampu mengetahui adanya sifat-sifat dari bangun geometri yang dilihatnya b Tahap Analisis Pada tahap ini siswa sudah mulai mengenal sifat-sifat yang dimiliki bangun geometri yang diamatinya c Tahap Pengurutan Pada tahap ini siswa sudah mengenal dan memahami sifat-sifat suatu bangun geometri serta sudah dapat mengurutkan bangun-bangun geometri yang satu sama yang lainnya saling berhubungan d Tahap Deduksi Pada tahap ini siswa telah mampu menarik kesimpulan secara deduktif yaitu menarik kesimpulan yang bersifat umum dan menuju ke hal yang bersifat khusus serta dapat mengambil kesimpulan e Tahap Akurasi Pada tahap ini siswa mulai menyadari pentingnya ketepatan prinsip-prinsip dasar yang melandasi suatu pembuktian Tahap berfikir ini merupakan tahap berfikir yang paling tinggi rumit dan kompleks karena di luar jangkauan usia anak-anak SD sampai tingakat SMP

49 Kd 311 Merancang aktivitas pembelajaran berdasarkan prinsip dan teori pembelajaran matematika middot Matematika untuk tujuan pembelajaran dianalisis sebagai kumpulan fakta yang berdiri sendiri dan tidak saling berkaitan middot Anak diharuskan menguasai unsur-unsur yang banyak sekali tanpa diperhatikan pengertiannya middot Anak mempelajari unsur-unsur dalam bentuk seperti yang akan digunakan nanti dalam kesempatan lain middot Anak akan mencapai tujuan ini secara efektif dan efisien dengan melalui pengulangan Teori Pembelajaran Matematika tersebut dikemukakan oleh a Skinner b Piaget c Van Brownell

d Thorndike Uraian Brownell mengemukakan tentang Teori Makna (Meaning Theory) sebagai pengganti Teori Latihan HafalUlangan (Drill

Theory) Intisari dari teori Drill adalah - Matematika untuk tujuan pembelajaran dianalisis sebagai kumpulan fakta yang berdiri sendiri dan tidak saling berkaitan - Anak diharuskan menguasai unsur-unsur yang banyak sekali tanpa diperhatikan pengertiannya - Anak mempelajari unsur-unsur dalam bentuk seperti yang akan digunakan nanti dalam kesempatan lain - Anak akan mencapai tujuan ini secara efektif dan efisien dengan melalui pengulangan

50 Kd 311 Merancang aktivitas pembelajaran berdasarkan prinsip dan teori pembelajaran matematika Tahap operasionaloperasi konkrit adalah teori Pembelajaran Matematika yang dikemukakan Peaget pada tahap usia a dibawah 2 tahun b 2-7 Tahun c 7-8 Tahun d 7-12 Tahun

Uraian Piaget membagi skema yang digunakan anak untuk memahami dunianya melalui empat periode utama yang berkorelasi

dengan dan semakin canggih seiring pertambahan usia Periode sensorimotor (usia 0ndash2 tahun) Periode praoperasional (usia 2ndash7 tahun) Periode operasional konkrit (usia 7ndash12 tahun) Periode operasional formal (usia 12 tahun sampai dewasa)

51 313 Memilih media pembelajaran yang tepat untuk pembelajaran operasi bilangan bulat Untuk operasi bilangan bulat perkalian media yang paling tepat digunakan adalah a sapu lidi

b kerikil c daun pakis d koin

Uraian 52 313 Memilih media pembelajaran yang tepat untuk pembelajaran operasi bilangan bulat

Untuk operasi bilangan bulat negatif kita bisa menggunakan media a uang kertas b kerikil c mistar d koin

Uraian 53 313 Memilih media pembelajaran yang tepat untuk pembelajaran operasi bilangan bulat

Beranda Soal UKG Agus Zainal M (SDN Gudang 2)

Republik_ilalangyahoocoid

Untuk operasi bilangan bulat penjumlahan media yang paling tepat digunakan adalah a uang kertas b kerikil c daun pakis d koin

Uraian 54 314 Memilih media pembelajaran yang tepat untuk pembelajaran operasi bilangan pecahan

Untuk operasi bilangan pecahan kita bisa menggunakan media berikut kecuali a kartu bilangan b garis bilangan c gambar bidang d blok pecahan

Uraian 55 316 Memilih media pembelajaran yang tepat untuk pembelajaran geometri dan pengukuran

Untuk menghitung luas dan keliling bangun datar media yang paling tepat digunakan adalah a Penggaris b Kertas Folio Bergaris

c Neraca d Kalkulator

Uraian

Page 13: 50 SOAL UKG & PEMBAHASAN SESUAI KISI-KISIS.pdf

Beranda Soal UKG Agus Zainal M (SDN Gudang 2)

Republik_ilalangyahoocoid

44 Kd 311 Merancang aktivitas pembelajaran berdasarkan prinsip dan teori pembelajaran matematika Perhatikan gambar disamping Ilustrasi di samping dikemukakan Bruner dalam Proses Pembelajaran Matematika dalam tahap

a Simbolik b Ikonik

c Implikatif d Enaktif

Uraian No 40 45 Kd 311 Merancang aktivitas pembelajaran berdasarkan prinsip dan teori pembelajaran matematika

Perhatikan contoh berikut Contoh 2 pinsil + 3 pinsil = hellippinsil Contoh tersebut dikemukakan Bruner dalam Proses Pembelajaran Matematika dalam tahap a Simbolik

b Ikonik c Implikatif d Enaktif

Uraian No 40 46 Kd 311 Merancang aktivitas pembelajaran berdasarkan prinsip dan teori pembelajaran matematika

Perhatikan contoh berikut penjumlahan bilangan positif dan negatif siswa mencoba sendiri dengan menggunakan garis bilangan Contoh tersebut dikemukakan Bruner dalam Teorema Pembelajaran Matematika a Penyusunan

b Notasi c Pengkontrasan dan Keanekaragaman d Pengaitan

Uraian Berdasarkan hasil pengamatannya Brunner merumuskan 5 teorema dalam pembelajaran matematika yaitu

1) Teorema Penyusunan Menerangkan bahwa cara yang terbaik memulai belajar suatu konsep matematika dalil defenisi dan semacamnya adalah dengan cara menyusun penyajiannya Misalnya dalam mempelajari penjumlahan bilangan positif dan negatif siswa mencoba sendiri dengan menggunakan garis bilangan 2) Teorema Notasi Menerangkan bahwa dalam pengajaran suatu konsep penggunaan notasi-notasi matematika harus diberikan secara bertahap dari yang sederhana ke yang lebih kompleks 3) Teorema Pengkontrasan dan Keanekaragaman Menerangkan bahwa pengontrasan dan keanekaragaman sangat penting dalam melakukan pengubahan konsep matematika dari yang konkrit ke yang lebih abstrak Dalam hal ini diperlukan banyak contoh Contoh yang diberikan harus sesuai dengan rumusan yang diberikan Misalnya menjelaskan persegi panjang disertai juga kemungkinan jajaran genjang dan segi empat lainnya selain persegi panjnag Dengan demikian siswa dapat membedakan apakah segi empat yang diberikan padanya termasuk persegi panjang atau tidak 4) Teorema Pengaitan Menerangkan bahwa dalam matematika terdapat hubungan yang berkaitan antara satu konsep dengan konsep yang lain Di mana materi yang satu merupakan prasyarat yang harus diketahui untuk mempelajari materi yang lain

47 Kd 311 Merancang aktivitas pembelajaran berdasarkan prinsip dan teori pembelajaran matematika Perhatikan contoh berikut Guru menjelaskan persegi panjang disertai juga kemungkinan jajaran genjang dan segi empat lainnya selain persegi panjnag Dengan demikian siswa dapat membedakan apakah segi empat yang diberikan padanya termasuk persegi panjang atau tidak Contoh tersebut dikemukakan Bruner dalam Teorema Pembelajaran Matematika a Penyusunan b Notasi c Pengkontrasan dan Keanekaragaman

d Pengaitan Uraian No 46 48 Kd 311 Merancang aktivitas pembelajaran berdasarkan prinsip dan teori pembelajaran matematika

Tahap pembelajaran Matematika menurut Van Halle adalah sebagai berikut kecuali a Tahap Pengenalan b Tahap Pengurutan c Tahap Analisis d Tahap Penyimpulan

Beranda Soal UKG Agus Zainal M (SDN Gudang 2)

Republik_ilalangyahoocoid

Uraian Van Hiele menyatakan bahwa terdapat 5 tahap belajar siswa dalam belajar geometri yaitu

a Tahap Pengenalan Pada tahap ini siswa mulai belajar mengenal suatu bangun geometri secara keseluruhan namun belum mampu mengetahui adanya sifat-sifat dari bangun geometri yang dilihatnya b Tahap Analisis Pada tahap ini siswa sudah mulai mengenal sifat-sifat yang dimiliki bangun geometri yang diamatinya c Tahap Pengurutan Pada tahap ini siswa sudah mengenal dan memahami sifat-sifat suatu bangun geometri serta sudah dapat mengurutkan bangun-bangun geometri yang satu sama yang lainnya saling berhubungan d Tahap Deduksi Pada tahap ini siswa telah mampu menarik kesimpulan secara deduktif yaitu menarik kesimpulan yang bersifat umum dan menuju ke hal yang bersifat khusus serta dapat mengambil kesimpulan e Tahap Akurasi Pada tahap ini siswa mulai menyadari pentingnya ketepatan prinsip-prinsip dasar yang melandasi suatu pembuktian Tahap berfikir ini merupakan tahap berfikir yang paling tinggi rumit dan kompleks karena di luar jangkauan usia anak-anak SD sampai tingakat SMP

49 Kd 311 Merancang aktivitas pembelajaran berdasarkan prinsip dan teori pembelajaran matematika middot Matematika untuk tujuan pembelajaran dianalisis sebagai kumpulan fakta yang berdiri sendiri dan tidak saling berkaitan middot Anak diharuskan menguasai unsur-unsur yang banyak sekali tanpa diperhatikan pengertiannya middot Anak mempelajari unsur-unsur dalam bentuk seperti yang akan digunakan nanti dalam kesempatan lain middot Anak akan mencapai tujuan ini secara efektif dan efisien dengan melalui pengulangan Teori Pembelajaran Matematika tersebut dikemukakan oleh a Skinner b Piaget c Van Brownell

d Thorndike Uraian Brownell mengemukakan tentang Teori Makna (Meaning Theory) sebagai pengganti Teori Latihan HafalUlangan (Drill

Theory) Intisari dari teori Drill adalah - Matematika untuk tujuan pembelajaran dianalisis sebagai kumpulan fakta yang berdiri sendiri dan tidak saling berkaitan - Anak diharuskan menguasai unsur-unsur yang banyak sekali tanpa diperhatikan pengertiannya - Anak mempelajari unsur-unsur dalam bentuk seperti yang akan digunakan nanti dalam kesempatan lain - Anak akan mencapai tujuan ini secara efektif dan efisien dengan melalui pengulangan

50 Kd 311 Merancang aktivitas pembelajaran berdasarkan prinsip dan teori pembelajaran matematika Tahap operasionaloperasi konkrit adalah teori Pembelajaran Matematika yang dikemukakan Peaget pada tahap usia a dibawah 2 tahun b 2-7 Tahun c 7-8 Tahun d 7-12 Tahun

Uraian Piaget membagi skema yang digunakan anak untuk memahami dunianya melalui empat periode utama yang berkorelasi

dengan dan semakin canggih seiring pertambahan usia Periode sensorimotor (usia 0ndash2 tahun) Periode praoperasional (usia 2ndash7 tahun) Periode operasional konkrit (usia 7ndash12 tahun) Periode operasional formal (usia 12 tahun sampai dewasa)

51 313 Memilih media pembelajaran yang tepat untuk pembelajaran operasi bilangan bulat Untuk operasi bilangan bulat perkalian media yang paling tepat digunakan adalah a sapu lidi

b kerikil c daun pakis d koin

Uraian 52 313 Memilih media pembelajaran yang tepat untuk pembelajaran operasi bilangan bulat

Untuk operasi bilangan bulat negatif kita bisa menggunakan media a uang kertas b kerikil c mistar d koin

Uraian 53 313 Memilih media pembelajaran yang tepat untuk pembelajaran operasi bilangan bulat

Beranda Soal UKG Agus Zainal M (SDN Gudang 2)

Republik_ilalangyahoocoid

Untuk operasi bilangan bulat penjumlahan media yang paling tepat digunakan adalah a uang kertas b kerikil c daun pakis d koin

Uraian 54 314 Memilih media pembelajaran yang tepat untuk pembelajaran operasi bilangan pecahan

Untuk operasi bilangan pecahan kita bisa menggunakan media berikut kecuali a kartu bilangan b garis bilangan c gambar bidang d blok pecahan

Uraian 55 316 Memilih media pembelajaran yang tepat untuk pembelajaran geometri dan pengukuran

Untuk menghitung luas dan keliling bangun datar media yang paling tepat digunakan adalah a Penggaris b Kertas Folio Bergaris

c Neraca d Kalkulator

Uraian

Page 14: 50 SOAL UKG & PEMBAHASAN SESUAI KISI-KISIS.pdf

Beranda Soal UKG Agus Zainal M (SDN Gudang 2)

Republik_ilalangyahoocoid

Uraian Van Hiele menyatakan bahwa terdapat 5 tahap belajar siswa dalam belajar geometri yaitu

a Tahap Pengenalan Pada tahap ini siswa mulai belajar mengenal suatu bangun geometri secara keseluruhan namun belum mampu mengetahui adanya sifat-sifat dari bangun geometri yang dilihatnya b Tahap Analisis Pada tahap ini siswa sudah mulai mengenal sifat-sifat yang dimiliki bangun geometri yang diamatinya c Tahap Pengurutan Pada tahap ini siswa sudah mengenal dan memahami sifat-sifat suatu bangun geometri serta sudah dapat mengurutkan bangun-bangun geometri yang satu sama yang lainnya saling berhubungan d Tahap Deduksi Pada tahap ini siswa telah mampu menarik kesimpulan secara deduktif yaitu menarik kesimpulan yang bersifat umum dan menuju ke hal yang bersifat khusus serta dapat mengambil kesimpulan e Tahap Akurasi Pada tahap ini siswa mulai menyadari pentingnya ketepatan prinsip-prinsip dasar yang melandasi suatu pembuktian Tahap berfikir ini merupakan tahap berfikir yang paling tinggi rumit dan kompleks karena di luar jangkauan usia anak-anak SD sampai tingakat SMP

49 Kd 311 Merancang aktivitas pembelajaran berdasarkan prinsip dan teori pembelajaran matematika middot Matematika untuk tujuan pembelajaran dianalisis sebagai kumpulan fakta yang berdiri sendiri dan tidak saling berkaitan middot Anak diharuskan menguasai unsur-unsur yang banyak sekali tanpa diperhatikan pengertiannya middot Anak mempelajari unsur-unsur dalam bentuk seperti yang akan digunakan nanti dalam kesempatan lain middot Anak akan mencapai tujuan ini secara efektif dan efisien dengan melalui pengulangan Teori Pembelajaran Matematika tersebut dikemukakan oleh a Skinner b Piaget c Van Brownell

d Thorndike Uraian Brownell mengemukakan tentang Teori Makna (Meaning Theory) sebagai pengganti Teori Latihan HafalUlangan (Drill

Theory) Intisari dari teori Drill adalah - Matematika untuk tujuan pembelajaran dianalisis sebagai kumpulan fakta yang berdiri sendiri dan tidak saling berkaitan - Anak diharuskan menguasai unsur-unsur yang banyak sekali tanpa diperhatikan pengertiannya - Anak mempelajari unsur-unsur dalam bentuk seperti yang akan digunakan nanti dalam kesempatan lain - Anak akan mencapai tujuan ini secara efektif dan efisien dengan melalui pengulangan

50 Kd 311 Merancang aktivitas pembelajaran berdasarkan prinsip dan teori pembelajaran matematika Tahap operasionaloperasi konkrit adalah teori Pembelajaran Matematika yang dikemukakan Peaget pada tahap usia a dibawah 2 tahun b 2-7 Tahun c 7-8 Tahun d 7-12 Tahun

Uraian Piaget membagi skema yang digunakan anak untuk memahami dunianya melalui empat periode utama yang berkorelasi

dengan dan semakin canggih seiring pertambahan usia Periode sensorimotor (usia 0ndash2 tahun) Periode praoperasional (usia 2ndash7 tahun) Periode operasional konkrit (usia 7ndash12 tahun) Periode operasional formal (usia 12 tahun sampai dewasa)

51 313 Memilih media pembelajaran yang tepat untuk pembelajaran operasi bilangan bulat Untuk operasi bilangan bulat perkalian media yang paling tepat digunakan adalah a sapu lidi

b kerikil c daun pakis d koin

Uraian 52 313 Memilih media pembelajaran yang tepat untuk pembelajaran operasi bilangan bulat

Untuk operasi bilangan bulat negatif kita bisa menggunakan media a uang kertas b kerikil c mistar d koin

Uraian 53 313 Memilih media pembelajaran yang tepat untuk pembelajaran operasi bilangan bulat

Beranda Soal UKG Agus Zainal M (SDN Gudang 2)

Republik_ilalangyahoocoid

Untuk operasi bilangan bulat penjumlahan media yang paling tepat digunakan adalah a uang kertas b kerikil c daun pakis d koin

Uraian 54 314 Memilih media pembelajaran yang tepat untuk pembelajaran operasi bilangan pecahan

Untuk operasi bilangan pecahan kita bisa menggunakan media berikut kecuali a kartu bilangan b garis bilangan c gambar bidang d blok pecahan

Uraian 55 316 Memilih media pembelajaran yang tepat untuk pembelajaran geometri dan pengukuran

Untuk menghitung luas dan keliling bangun datar media yang paling tepat digunakan adalah a Penggaris b Kertas Folio Bergaris

c Neraca d Kalkulator

Uraian

Page 15: 50 SOAL UKG & PEMBAHASAN SESUAI KISI-KISIS.pdf

Beranda Soal UKG Agus Zainal M (SDN Gudang 2)

Republik_ilalangyahoocoid

Untuk operasi bilangan bulat penjumlahan media yang paling tepat digunakan adalah a uang kertas b kerikil c daun pakis d koin

Uraian 54 314 Memilih media pembelajaran yang tepat untuk pembelajaran operasi bilangan pecahan

Untuk operasi bilangan pecahan kita bisa menggunakan media berikut kecuali a kartu bilangan b garis bilangan c gambar bidang d blok pecahan

Uraian 55 316 Memilih media pembelajaran yang tepat untuk pembelajaran geometri dan pengukuran

Untuk menghitung luas dan keliling bangun datar media yang paling tepat digunakan adalah a Penggaris b Kertas Folio Bergaris

c Neraca d Kalkulator

Uraian